MENU
未分類

第94回日本社会学会大会 報告要旨(11月13日(土)9:30~12:30)

報告番号1

社会的世界論の活用――ケーススタディを通じたMacro symbolic interactionの分析に向けて
椙山女学園大学 鎌田大資

1. 目的:社会的世界論(Strauss, Dewey, Shibutaniによるsocial worlds perspective。またBeckerのArt Worldsも同系列)の導入 (報告者の独断による)暫定的定義は以下の通り。具体的な社会、歴史事象の通時的な分析において集団ごと、集団同士の分化、接合における技術やイデオロギーの変遷を解釈し、跡づける考え方。シンボルにかかわる相互作用、集団のアイデンティティ形成、相互作用における正当化、正統化、社会的資源の動員などに着目する。下部構造決定論や社会システム論などの前提を優先的に用いることはない。資源動員論とも近いが、社会運動による社会状況の改善がなされるかどうかという問題意識は共有しない(社会の慣性的保守的圧力によって、一部の参加者が望む変化が実現しない場合も、冷静に一事例として考察する)。普通におこなわれている各時代の歴史記述や分析に近づくことが予測されるが、社会学らしい特色は集団単位の事象の変遷に注目する点にある。宝月誠の近著『シカゴ学派社会学の可能性――社会的石論の視点と方法』(東信堂, 2021)では、イギリスの哲学者Bhaskerの批判的実在論との接合が模索されており、今後は、社会的世界論の視点を用いた研究自体が、ひとつの社会的世界を形成して、ブルデュー系社会学なども含む多様な理論的潮流との接合や分岐のなかで変容していくことが予想される。 2. 方法(日本における)社会的世界の変遷のケーススタディ  日本で社会的世界論を応用した研究は、山口健一の先駆的事例を除きほとんど存在しないが、報告者の経験や参与観察の結果をまとめるだけでも、下記のような分析の系列を列記することができる。 a. 平成30年間の精神保健福祉の変遷――とある精神障がい者患者会の変質と社会復帰から就労支援へのサービス・モデル・シフトの並行関係:ミクロからメゾレヴェルの経験的調査がマクロ・レヴェルの政策的変動を先取りしていた事例 b. 岩田信市の美術家キャリアのエピファニー的解釈――前衛アートから小芝居としての歌舞伎と日本語で上演するコミック・オペラの復活へ:名古屋に移転して参加した歌舞伎、オペラを上演する劇団スーパー一座の活動を通じて得た経験を整理する c. 古都税問題をめぐる交渉文脈の整理――廃仏毀釈による寺社勢力の衰退から拝観事業の発展による観光産業への参画:学士時代に参加した政治社会学的な共同研究 (d. 初期シカゴ学派における量的、質的社会調査の発達にみる市民社会発展史の発展的継続、日本におけるSI学説的、理論的紹介の系譜:社会学説史も大学や政府機関を舞台とした知的社会運動の結果として整理できる 3.結果  各社会的世界の分岐、接合の際には多様な交渉文脈から影響を受けた相互作用的な交渉が生じ、社会的世界やそれを取り巻く環境となる世界の進行に変化をもたらす。 4. 結論  個人レベルの実存的存在契機の分析と相互作用的な交渉文脈の分析、さらにマクロな社会現象へと展開する社会的世界論の分析を導くことで、SIのパースペクティブを現実社会の分析に応用できる。

報告番号2

アジアにおけるウェーバー受容の地政学――二つの二重性との交錯
京都大学大学院 吉琛佳

本研究は、日本と中国におけるマックス・ウェーバーの受容史を例として、二つの社会の近代化期における知識形態を知識社会学的・地政学的な観点から考察する論考である。戦後日本および80年代の中国における知識人の問題関心は、一言で「啓蒙」や「第二の近代化」という課題でまとめられたとはいえ、その同時、各自の時代状況に直面して、「近代」を再想像することでもある。しかし、両社会の知識人が置かれた歴史的、地縁的な布置は、本人が知らざるうちに共通なる思想課題を彼らに課していた。  こうした課題の共通性を考察する際、両社会それぞれで発生したウェーバー受容史は好都合な事例を提示されたのであろう。なぜなら「近代社会」に関するもっとも有力な解釈パターンのひとつを開示したウェーバーの学説は、近代化の道を歩み出したアジア諸国の知識人が誰だって直面しなければならないものであり、それゆえ、かれらの存在的被拘束性を検討する際の共通項を構成したからである。それゆえ、本稿はアジアの知識人が「近代」を面した際の知識形成条件を析出するため、両社会の近代化期におけるウェーバー受容を検討の対象とする。  この研究によって明らかになったように、アジア社会科学のウェーバー受容を形づくったのは、知識形成の前提となる二つの「二重なる」構造である。その一つは近代に内在する二重性である。この二重性は「主体化と客体化との同時的展開」と要約できる。すなわち近代化や資本主義制度の確立と発達は、人間が解放と束縛、支配と服従、主宰者になることと支配を受けること、または自由の獲得と疎外感の深刻化など、互いに相反する存在的状況を同時に経験し推進するプロセスと考えられる。東アジア社会も近代化の推進につれて、こうした普遍的な二重なるプロセスを免れないばかりか、知識形成の基礎なる前提条件として知識人を拘束してきた。  しかし、アジアの知識人はもう一つの二重性、すなわち「植民地的二重性」に直面しなければならない。資本主義の進展は西洋帝国主義の世界範囲での拡張と植民地支配の確立を促した。コロニアリズムは非西洋社会の近代化を推し進めながら、西洋から非西洋社会自身の後進性に関する観念図式の内面化を強いられた。「後進的」と認められた土着的知識体系の正当性が著しく下落したため、非西洋社会における知識人は自分の立ち位置とは異なる西洋の知識体系に基づいて思想形成を余儀なくされた。  近代の二重性と植民地的二重性、これら二つの知識条件の錯綜なる交差は、近代アジア諸社会の知識形態に絶えず影響を与えている。こうした存立条件の対象化によって、アジア社会科学の再帰的な展開への寄与を期待する。

報告番号3

チャールズ・テイラーにおける社会学的世俗化論批判
島根大学 諸岡了介

【1.目的】  チャールズ・テイラーの「世俗化論」を取り上げ、それと宗教社会学における既存の「世俗化論」の関係を吟味することで、彼の社会学的世俗化論批判の姿勢とその含意を明らかにする。 【2.方法】  『世俗の時代』(2007年)を中心としたテキスト読解。 【3.結果】  2007年に、すでに世界的に高名な政治哲学者であったチャールズ・テイラーが『世俗の時代』において彼流の「世俗化論」を示したことは、時流に逆らう印象を与えるものでもあった。宗教社会学の分野では、1960年代から大いに注目を集めた世俗化論は、1980年代からこのかた、新宗教運動の台頭や、イスラーム主義やキリスト教福音派の世界的な成長といった現象を背景に、「時代遅れ」の認識と見なされることが多くなっている。政治哲学の分野では、21世紀に入るとユルゲン・ハーバーマスが「ポスト世俗の時代」を謳い、世俗化した現代世界という認識の「誤り」を印象づけた。テイラーの仕事は、そうした潮流に、唐突に反旗を翻すものであった。  しかしそれでは、テイラーの「世俗化論」は、宗教社会学における既存の「世俗化論」といったいどのような関係にあるのだろうか。たしかに『世俗の時代』には、スティーヴ・ブルースらの世俗化論を取り上げた箇所が含まれるが、宗教社会学の世俗化諸理論を十分に論じているとはいえない。とりわけ、宗教の多元的状況やそれによる「意味喪失」状況を論じたピーター・バーガーの所論は、近現代西洋社会に関するテイラーの認識にもっとも近いものであるはずだが、これを主題的には取り上げていない。バーガーが「信憑性構造の脆弱化」として表現した事態についても、テイラーは「交差圧力」といった独自の用語で説明しなおそうとする。  こうしたテイラーの姿勢の裏には、世俗化を語る宗教社会学の言説空間それ自体に対する批判意識が存していると考えられる。テイラーは、宗教を何らかの社会条件の従属変数と捉える見方や、宗教の中心機能を意味付与機能に求める見方をともに拒否している。テイラーからすると、宗教社会学の言説空間は、超越的契機を抜きにした人間中心的で内在的な世界理解をデフォルトのものとして前提しており、それ自体が、近現代の世俗化した状況の一部にほかならない。社会条件や意味付与機能から宗教を捉える見方は、そうした内在的世界理解に発しているというわけだろう。 【4.結論】  こうした批判意識の下にあるテイラーの世俗化論は、マルセル・ゴーシェの「脱呪術化」論の方向性を承けたもので、マックス・ヴェーバーやバーガーらが先導してきたプロテスタント色の強い世俗化論に対するアンチ・テーゼとして理解しうる。知的な「意味」や抽象的法則に偏した宗教理解・世俗化理解を批判することで得られるもののひとつは、現代における宗教的暴力の可能性という問題に対する視点である。他方、「近代化が宗教に与える効果」といった種類の社会学的説明を排除したテイラーの世俗化論からは、西洋の思想伝統を直接に継承していない非西洋社会における動向は、視野の外に置かれることとなる。

報告番号4

「集合意識」概念の展開可能性
神戸大学 白鳥義彦

【1.目的】  本報告は、デュルケームの社会学における主要な概念の一つである「集合意識」に注目し、その展開可能性を検討することを目的とする。  『社会分業論』において集合意識は、「同じ社会の成員たちの平均に共通な諸信念と諸感情の総体は、固有の生命をもつ一定の体系を形成する。これを集合意識または共同意識とよぶことができる。もちろん、それはただひとつの機関を基体としてもっているわけではない。それは、定義によって、社会の全範囲に広く分散している。〔中略〕個人は過ぎ去るが、集合意識は残る。〔中略〕大都会でも小都市でも、あるいはどんな職業においても、それは同一である。同様に、それは世代ごとに変わったりはしない。むしろ、ある世代とつぎの世代とを結びつける。それだから、集合意識は諸個人のうちにおいてしか実現されないとはいえ、各人の個別的な諸意識とはまったく異なったものである。それは社会の心理的類型である」(田原訳、1971:80-81)と定義、描出されている。こうした集合意識の概念の展開の可能性を、第一にデュルケーム自身の議論について、第二にデュルケーム以外の者による議論について、第三に今日的な社会的事象の分析について、考察していきたい。 【2.方法】  デュルケーム自身の議論については、デュルケームが諸著作において行っている論述をもとに考察を行う。デュルケーム以外の者による議論については、アルヴァックスによる「集合的記憶」の概念や、歴史学のアナール学派による「心性(マンタリテ)」の概念等を念頭に考察を行う。今日的な社会的事象の分析については、意識の共有性という観点から考察を行う。 【3.結果】  デュルケームによれば、社会が社会たりうるのは、その社会を構成する人々の間に何らかの共通性が存在することによってこそである。この共通性については、物質的な側面のみならず、集合意識という意識の側面にも注目することが重要である。また、デュルケームは国家を「集合体の所産ではなくとも集合体にかかわる表象や意思が形成される場である、一種独特の公務員集団である」と定義するが、その文脈において「国家は集合意識を体現するという言い方は、正確ではない。なぜなら、集合意識は、国家の枠内からあらゆる面ではみだしているからである。集合意識は、その大部分が拡散している。あらゆる種類のおびただしく多様な、多くの社会的感情、社会的諸状態がつねに存在しているが、国家が知覚するのは、それらのかすかな反響にすぎない」と述べており、国家やデモクラシーについて考えるに際して、集合意識が重要な概念となっていることがわかる。 アルヴァックスの集合的記憶の概念も、意識や記憶の「共有性」という観点から集合意識の概念との関連性を指摘することができる。アナール学派の心性の概念も、人々の意識に着目するという点から集合意識の概念と関連づけて考察することが可能である。さらにデュルケームによる『世界に冠たるドイツ』の副題に「心性」という語が用いられていたことも想起される。 今日的な社会的事象の分析についても、「ポスト真実」の時代に人々が何を信じるのかということを考察するに際して、集合意識の概念は有効性をもつ。 【4.結論】  このように様々な観点から、「集合意識」概念の新たな展開可能性を考えることができる。

報告番号5

W.E.B.デュボイスと日本――アフリカ-アジア主義から社会主義へ
東海大学 本田量久

【1. 目的】  W.E.B.デュボイスは、白人世界によるアフリカの侵略と植民地支配を論じ、これに対抗すべく汎アフリカ主義運動を主導した。1920年代には、自己決定に基づく新たな世界秩序を構築するためには、アフリカン・ディアスポラとアジアの連帯を重視する一方、日本・中国にアジア防衛を期待した。1930年代に日本が帝国主義的拡張を展開し、日中戦争が拡大しても、デュボイスは日本を擁護し続けた。しかし、第二次世界大戦後、デュボイスは日本に対する期待を弱め、他方で社会主義に世界平和の実現可能性を模索した。本報告では、第二次世界大戦を経て、デュボイスがアフリカ-アジア主義的な非白人の連帯から人種を越えた社会主義的な世界的連帯へと傾倒した背景を論じてみたい。 【2. 方法】  本報告では、19世紀末から1950年代までに刊行されたデュボイスの文献を読み解き、日本のアジア防衛に期待した彼の議論を示すとともに、第二次世界大戦を経て、社会主義に傾倒していった背景を考察する。 【3. 結果】  デュボイスによれば、アフリカ・アジアの安価な労働力や天然資源を搾取し、利益の最大化を求める資本主義的動機が欧米諸国の植民地拡張競争を触発した結果、世界規模の戦争が勃発した。欧米諸国は、アヘン戦争(1840年)以降、アジア進出を展開し、近代化を目指す日本も欧米諸国に続いて中国や朝鮮半島における権益拡大を目指した。  デュボイスによれば、第一次世界大戦後の欧米諸国は「非理性的」「非文明的」「受動的」である非白人には自己決定を認められないという認識を示し、植民地支配の正統性を訴えた。これに対して日本がパリ講和会議(1919年)で人種差別撤廃を提案したことから、デュボイスは日本にアジア防衛の指導的役割を期待した。  1920、30年代のデュボイスは、収益拡大を目指す植民地拡張と戦争をめぐるマルクス主義的な考察とともに、資本主義の原理に還元できない人種主義にも着目しながら、白人世界に対抗すべくアフリカ-アジア主義的な共闘を訴えた。満州事変(1931年)以降、日本がアジア進出を展開すると、欧米諸国は厳しく非難したが、デュボイスは欧米諸国の偽善を指摘しながら日本を擁護している。1936年にはソ連と中国を経由して日本を訪れ、アジア防衛における日本・中国の指導的役割に期待を寄せるとともに、白人世界に対抗すべくアフリカ-アジア主義的な共闘の意義を訴え、更に1940年代にはアジア支配を正統化する日本のイデオロギーにも共感を示している。  しかし、第二次世界大戦後、日本に関するデュボイスの言及は減り、欧米諸国に対する日本との共闘を訴えることはなくなった。第二次世界大戦以前は非白人の連帯を訴えたが、資本主義が戦争をもたらす主な原因であるという認識を強め、社会主義に基づく新たな世界秩序の構築を模索した。1950年前後のデュボイスは、原爆禁止といった人類の存亡に関わる普遍的利害に関心を強め、世界各地で社会主義ネットワークを拡大しながら平和活動を展開した。 【4. 結論】  デュボイスは、日本こそが白人世界の侵略に対抗し、アジア防衛に指導的役割を果たすと強く期待したが、第二次世界大戦後、かつて人種差別的として不信感を抱いた労働組合とも協調しながら、資本主義の超克と世界平和の実現を訴えるようになった。

報告番号6

バーバラ・アダムの時間論におけるG・H・ミードの影響――多元的時間論を中心にして
慶應義塾大学大学院 末田隼大

【目的】 本報告の目的は、バーバラ・アダムの時間論を、G・H・ミードの時間論・社会論との関係から検討し、時間の社会学におけるアダムの議論の位置づけを問い直すことにある。近年、興隆を見せている時間の社会学において、分野として従来から抱えている問題として、知見の体系性に乏しく、個々の理論・実証研究が孤立している点が挙げられる。本研究が対象とするアダムは時間の社会学の体系化を目指した理論家の1人である。アダムは、イギリス人の社会理論家であり、時間の社会学における主要な学者と目されている。彼女の時間論が提示された『時間と社会理論』は学説の系譜における数少ない結節点の一つである。もっとも、本著において展開される議論を理論的・学説史的に検討した課題は少なく、時として雑多な先行研究の集積と非難されることさえある。本報告は、いかなる理論的見地からアダムが知見を整理しているのかを検討することで、彼女の時間論の特質を明らかにし、その上で時間の社会学における可能性を考察すること試みたい。 【2.方法】 本報告は上述の『時間と社会理論』を分析する。ただし、本著は多岐にわたる論点を含んでいるので、今回は多元的時間論に焦点を絞る。多元的時間論とは、客観時間を(無自覚に)唯一の時間とみなす一元論的時間論や線形的時間/円環的時間などの二元論的時間論のオルタナティブとして1980年代以降構想された時間論であり、アダム時間論の重要な論点の一つである。 【3.結果】 アダムは時間の社会学の学説を整理するに当たって、ミードの時間論の枠組みを院に陽に採用する。ミードはニュートン・デカルト的な絶対時間ではなく、具体的な出来事や変化のみが時間を織りなしているという出来事時間を主張したことは広く知られている。アダムの時間論の特質は、この出来事時間の概念と、ミードの社会性の概念を交差させたことにある。アダムはミードの社会性を、社会や自然の要素が相互に作用・調整するネットワークとして捉える。この立場に立つとき、自然時間や社会時間などの区別は意味をなくし、時間計測とは時間を示す様々な出来事と主体の複雑な関係性と枠づけられる。 【4.結論】 多元的時間論の理論的基盤として、ミードに注目する論者は他にもいるが、ミードの社会性の見地からその時間論を考察したことで、アダムは時間という知識の範囲を広げると同時に、それらの関係性にも議論の射程を伸ばしている。アダムは後に、対象の時間や時間性を多面的かつ複合的に分析する「時間景観」という分析概念を提唱するが、彼女の多面的時間論はその理論的基盤の一つと言える。 【参考文献】 Adam, Barbara, 1990, Time and Social, Cambridge: Polity Press.(伊藤誓・磯山甚一訳,1997,『時間と社会理論)法政大学出版局.) ————, 1998, Timescapes of Modernity, London: Routledge.

報告番号7

フィールド調査の系譜――桜田勝徳「漁村研究」と「網子」
早稲田大学 武田尚子

1 関心の所在:日本のフィールドワークの源流を探る  フィールドワーク、いわゆる現場調査は質的調査に不可欠であるが、「フィールドワークの源流」というと、アメリカのシカゴ学派のエスノグラフィーを連想したり、日本の村落社会学・家族社会学・労働社会学の領域で分厚く蓄積されてきた数々のモノグラフを思い浮かべるかもしれない。また、民俗学の柳田国男や宮本常一を思い出すこともあるだろう。本報告は日本の「フィールドワークの源流」の一人として、桜田勝徳の「漁村研究」の意義について述べる。桜田のフィールド調査に基づく成果の一つは「網子」の発見である。「網子」への着眼と意義、研究視点に内包されている可能性について考察する。 2 桜田勝徳とフィールドノート  桜田は日本民俗学の第二世代に該当する。慶応大学文学部で学び、卒論執筆では文学部講師だった柳田国男の指導を受けた。柳田の『明治大正史・世相篇』執筆の助手(資料収集)を務め、自分の関心に即して昭和6年に漁村調査に着手、昭和9年に著書を刊行した。その前年、渋沢敬三の面識を得て、昭和10年にアチックミューゼアム(日本常民文化研究所)に研究員として入所した。昭和9年、柳田は桜田に宮本常一を紹介、のち宮本もアチックに入所した。昭和52年に桜田が没した後、有賀喜左衛門が編者代表を務めて『桜田勝徳著作集〔全7巻〕』が刊行された。全集第1巻巻頭は有賀の「総序」、巻末は宮本の「解説」である。桜田が遺した200点近い調査資料(調査ノートほか)は慶応大学古文書室に所蔵されており、昭和戦前期における村落調査の極めて重要な記録・資料群である。本報告は、この慶応大学所蔵「桜田勝徳調査研究資料アーカイブ」を用いて、桜田のフィールド調査の着眼点と成果について考察する。 3 フィールドでの発見:網子 「桜田勝徳調査研究資料アーカイブ」仮総目録に基づくと、桜田の調査の焦点の一つが「網」だったことがわかる。瀬戸内海の鯛網漁業について、桜田が兵庫県の家島で行った聞き取り調査のノートが保存されている。鯛網の資本すなわち網元を漁民たちは口語で「ムラギ(村君)」という言い方をしていた。桜田はこの言葉に関心を持ち、これは資本を有する網元「村君」と雇われる網子の社会関係の有り様を示唆しており、ここに着眼した桜田は他地域での漁村研究の成果を踏まえて、網親・網子の社会関係の本質は、厳しい身分関係ではなくて、共同労働を能率よく実行していくための「仲間」関係であることを見出した。 4 研究集団とフィールド調査:アチックミューゼウム  桜田の「網子」に関する知見は、同時期に有賀喜左衛門が岩手県二戸郡荒沢村石神で見出した「名子」が親方に提供する無償労働の意味、すなわち農村における親方子方関係の「生活の全体的相互給付」という知見と双璧の貴重な研究成果であると思う。有賀と全く同時期に、桜田は漁村を対象に網集団に着目し、有賀の農村の「名子」とは異なる、漁村の親方子方関係の「網子」の社会的性格を掘り起こしていった。有賀の農村フィールド石神は、そもそもは昭和9年に渋沢敬三が最初に訪れ、その後に有賀と土屋喬雄が調査に入ったものである。昭和戦前期における「名子」「網子」という農村・漁村の社会関係を掘り起こした双璧の研究は渋沢が創ったアチックミューゼアムという調査環境・研究集団を基盤に生み出されてきたものであった。

報告番号8

調査員が回答内容に与える影響の自記式モードとの比較による検討
金沢大学 小林大祐

1.目的 調査票調査において,センシティブな内容を質問する場合,調査員が介在する他記式モードにおいて,測定誤差がより大きくなることは,多くの研究で明らかにされてきた。ただ,調査員の存在によって生じうる回答内容への影響は、必ずしも均質に作用するものではなく,調査員の間の差異に注目することもまた必要である。これまでも「目に見える調査員の特徴」の影響に注目した研究においては,調査員の人種や年齢そして性別が,それらの特徴に関わる質問の回答傾向に影響することが報告されている(調査員の性別に関しては、Kane and Macaulay 1993; Huddy et al. 1997; 小林・前田 2018など)。 ただし,調査員の属性の違いで回答内容に差が観察されたとしても,意識変数ではそのどれが「真の値」に近く,どれが偏りの生じた結果なのかは,多くの場合なんらかの理論仮説に基づいた解釈によって推測されるよりなかった。例えば,男性調査員よりも女性調査員に対して男女平等をより肯定する傾向が見られた場合,女性を前にして男女平等を否定しにくいため肯定的な方向に回答の偏りが生じると解釈され,男性調査員に対する回答の方がより「真の値」に近いと見なされるのである。しかし,当然ながらこのような解釈には常にアドホックとの批判がつきまとってきた。 本報告では,自記式モードと他記式モードを併用した調査データを用いることでこの問題に答える。他記式と自記式が併用されたデータの場合,前述の例だと,調査員が「男性」,調査員が「女性」,調査員が「いない」という3パターンの比較が可能になることで、調査員の影響がない場合を基準として,調査員の性別の影響を解釈できるのである。 2.方法 統計数理研究所が2012年に実施した「国民性に関する意識動向2012年度調査」の2次分析によって,この問題についての検証を行う。この調査では面接と留置の調査モードが併用され,調査員情報についても利用することが可能である。この調査で詳しく聞かれている政治意識や投票行動などについて,回答内容への調査員の影響を分析する。 3.結果と結論 まず,多くの項目で留置と面接で回答分布に有意な差が確認され,必ずしもセンシティブとはいえないような質問であっても調査員の存在によって影響を受けている可能性が示された。次に,調査員の性別による,回答分布の差についても検証を行った結果,幾つかの項目で男性回答者は,男性調査員の場合に回答が偏る傾向が見られ,男性は男性調査員を前にすることで、「見栄を張る」回答をしやすくなる可能性が示唆された。ただし,そのような傾向を示した項目に明確な特徴は無く,この傾向をどのように解釈すべきかについては,引き続き慎重に検討する必要がある。 謝辞 本研究はJSPS科研費20K02110, 18H03649, 19H00609の助成を受けたものです。 Huddy, L. et al., 1997, The effect of interviewer gender on the survey response, Political Behavior 19(3) :197 -220. Kane E.W. and Macaulay L.J., 1993, Interviewer gender and gender attitudes, Public Opinion Quarterly 57(1) :1-28. 小林大祐・前田忠彦, 2018, 「調査員の年齢・性別が性別役割分業意識の回答に与える影響について:SSP2015データを用いた分析」, 日本社会学会第91回大会,(2021年6月16日取得,https://jss-sociology.org/research/91/file/37.pdf).

報告番号9

高等教育研究は学問分野か研究領域か?
嘉悦大学 経営経済研究所 宇田川拓雄

1.目的  日本では大学進学率が5割を超え高等教育の大衆化の時代に入った。高等教育の実践や理論化に関わる高等教育研究という新しい領域が生まれた。高等教育研究は学問分野なのだろうか研究領域なのだろうか。本報告では高等教育究をフィールド科学の視点で分析し、高等教育研究は一つの学問分野として扱うよりも、各学問におけるSoTL*として扱うのが妥当であることを主張したい。(*Scholarship of Teaching and Learning:教員の授業と学生の学習の方法と過程を学術的に探求し公表することを通じて、教授と学習の実践を進歩させる体系的な研究で教育学とは異なる) 2. 方法  文献研究により高等教育研究出現の経緯と現状と問題点を指摘する。主に参照した文献:高等教育関連学会(大学行政管理学会、大学教育学会、高等教育学会、日本評価学会)の規定、学会誌、大会要旨集。高等教育センター紀要、アメリカ社会学会の部会Section on Teachingng and Learnign in Sociologyの規定、学会誌。 3. 結果  高等教育研究のための主要学会が1990年代後半から2000年にかけて作られた。学会誌や紀要などを見ると最初は授業担当者による事例研究の報告や論文が多かった。次第に高等教育研究の専門家や高等教育センターなどの専任研究者が現れ、抽象的な理論や精緻なモデルを用いた研究を行なうようになり、高等教育研究を学問として確立しようという動きが現れた。学問的な研究の水準が上がり、自分の専門分野を持ち副業的に高等教育研究をしている研究者には研究業績が出しにくくなっている。  実証科学としての社会学の視点に立てば高等教育研究はフィールド科学研究と理解できる。フィールド科学では社会的現実の記述が重要な目的となる。物理現象や実験室状況と異なりフィールド科学では何が起こっているのか、何が重要な要因なのか、何が解明すべき問題なのかは必ずしも自明ではない。そのような段階では現実を明らかにしようとする探索的研究を行なう。次の段階ではそれを科学的客観的に記述する記述的研究を行なう。学術的位置づけがはっきりした段階では厳密に定義した仮説を用意し、その検証を目的に研究に取り組む説明的研究(仮説検証型研究)が行なわれる。高等教育研究のコアを抽象度の高い理論や分析モデルを前提とした仮説検証型研究と考えると、探索型研究や記述型研究は科学的でないと判断される恐れがある。高等教育研究をいわば副業的に研究している他の学問分野の研究者の研究は「単なる事例研究」として評価が低くなるだろう。 4.結論  高等教育研究の専門家にとってそれが確立した理論と概念と厳密な論理構造をもった独立した学問分野であることは好ましい事である。しかし多くの高等教育研究者は学生の教育指導と学習支援目的でこの領域に関わっている。高等教育研究は様々な学問分野の研究者が関与する共通の研究領域であって、独立した学問への発展もありうるだろうが、同時に個々の専門分野ごとの特徴を持ちつつ教授と学習の方法の実践進歩を共通課題とするSoTLという研究領域としてして市民権を獲得して行くのではないだろか。

報告番号10

「日本人の読み書き能力調査(1948)」における学歴の効果について――戦後間もない時期の社会調査再訪
情報・システム研究機構 統計数理研究所 前田忠彦

【1.目的】  1948年実施の「日本人の読み書き能力調査」(以下読み書き能力調査)は,全国の15歳~64歳までの男女を母集団とする確率標本を対象として行われた大規模な学術的社会調査としては,戦後最も早期のものである。この調査の歴史的意義はいくつかあるが,社会調査史上では,戦後に導入された統計的な確率的抽出法の標準的な設計(層化多段無作為抽出)の先鞭をつけた貢献が大きい。 その素データは失われ,現在では調査報告書に結果を見ることしかできないが,企画段階から調査の実施,分析報告の段階まで,現代でも報告書から学ぶところは多いと考えられる。調査設計上の一つの特徴として,「読み書き能力」という明確な目的変数を持ち,それに関連する要因を検討するという明瞭な結果検討の枠組が用意されている点を指摘できる。本報告では,この調査がどのような構想の元で設計され,実際にどのような成果を挙げたかを報告書に示された結果から跡づけることを目的とする。なお横山・前田他(2021)で生年の効果を検討したことを踏まえ,本報告では主に対象者の学歴が読み書き能力調査に与える効果を改めて検討する。 【2.方法】  大別して8種類の出題内容計90問よりなり,正答に対して1点を与えた加算得点が90点満点の「読み書き能力得点」となる。報告書では,居住地の市郡別にいくつかの対象者属性等別の得点分布(原則5点刻み)が示された表があり,ここでは学歴別の得点分布を示した第38表を分析する。平均得点は下位学校種部分で急激に上昇し,上位学校種箇所では伸びが鈍化する上に凸の曲線を描く。  分析方法は,度数分布表の階級値を素得点と見なした時の「読み書き能力得点」を目的変数とする重回帰分析である。学歴(旧制)は,大別すると,0.学歴なし,1.尋常小学校,2.高等小学校,3.青年学校,4.実業学校,5.中等学校,6.高等・専門学校, 7.大学,となっている。1~3には卒業と中退,4以上には在学中,中退,卒業の小区分がある。 ここでは,これらの学校種別,中退・在学中か卒業の別をダミー化したものに加えて,市郡の別も説明変数として用いる。モデルのバリエーションについては,報告時に詳述する。 【3.結果】  学校種別・市郡・卒業ダミーのみを説明変数とした場合の自由度調整済み重相関係数は0.474, 学校種別では上位の得点が高く,効果量指標で見ても他の要因に比べて桁違いに大きい。郡部に比べて市郡の得点が高いという効果も高度に有意である。更に市郡×学校種別の交互作用を加えると同0.475となり,増分の値として大きくないものの有意で,郡部での下位の学校種で得点低下が市部より大きい方向の係数推定値が得られる。 【4.結論】  上記の結果は,報告書にも概ね考察されたものであり,新しい発見と言えるものではない。しかしながら,この結果を詳細に検討することを通じ,同調査の精度の高さが改めて確認されるとも言える。特にこの調査は非識字-識字層を分ける大きな区分として尋常小学校卒前後程度の教育程度で典型的であるような能力水準で,識別力の高い試験となっていたと考えられる。現代において同種の試験,たとえば機能的識字能力を測定するための精度の良い調査を設計する際の参考としうる内容と言える。

報告番号11

日本・韓国・台湾における若年雇用と初婚の関係――パネルデータを用いた家族の分析(1)
中京大学 松田茂樹

1.目的  東アジアは、世界で最も少子化がすすむ地域である。日本、韓国、台湾は合計特殊出生率が人口置換水準を大きく下回っており、少子化の克服が社会的な課題である。東アジアの少子化の特徴は、未婚化が急速に進行しており、結婚制度が強固であるために未婚化が出生率低下に直結することである。未婚化には各種要因が関係しているが、中でも主要な要因の1つが若年雇用とみられる。国全体は経済成長しているが、若い世代がその恩恵に浴せず、不安定雇用は広がり、賃金が伸び悩むという状況が生じている。これには、経済のグローバル化、産業構造の変化、生活コストの上昇、急速な高学歴化が、関係している。以上をふまえて、本稿は、日本、韓国、台湾における若者の雇用形態と収入が、初婚イベントの発生に与える影響を解明する。 2.方法  使用したデータは、日本・東大社研パネル(JLPS-Y)、韓国労働パネル(KLIPS)、台湾・家庭動態調査(PSFD)である。各調査の2000年代後半から開始または追加された20代~30代前半の男女個人のサンプルを分析に用いた。被説明変数を初婚イベントの発生、説明変数を雇用形態、年収、学歴等とした離散時間ロジット分析を行った。 3.結果  日韓台とも、非正規雇用者や低所得であることが初婚ハザード率を低くする効果は、もっぱら男性について確認された。3カ国とも、男性低所得者は初婚ハザード率低い。日韓では、男性非正規雇用者において初婚ハザード率が低い(台湾は変数なし)。日韓の女性においても、非正規雇用者は初婚ハザード率が低い傾向がある。3カ国とも、男性は高学歴者において初婚ハザード率が高い。 4.結論  本分析の結果、経済力の低い男性が結婚しにくいことが、いずれの国でも共通してみられた。3カ国は女性の就業率も異なるが(例えば、台湾は女性が結婚・育児で離職することは少ない)、結婚市場においては男性側のみに経済力が求められる状況はいずれの国も同じであった。この結果は、これら国々の家族規範(儒教規範)も、人々の結婚行動を強く規定している可能性を示唆する。また、結果をふまえると、若年の非正規雇用者のままでも所得を増やすような政策をしても、未婚化を止める力は限られるとみられる。 附記 本研究はJSPS科研費18H00936の助成を受けたものである。東京大学社会科学研究所附属日本社会研究情報センターSSJデータアーカイブ、Korea Labor Institute、台湾・中央研究院から個票データの提供を受けた。

報告番号12

韓国におけるワーク・ライフ・バランス政策と出産行動の関係――パネルデータを用いた家族の分析(2)
桜美林大学 裵智恵

【1.目的】  2020年時点において、韓国の総出生児数は約27万2,400人、死亡者数は約30万5,100人となり、いわゆる「人口デッドクロス(dead cross)」を記録した。また、2001年以降継続して1.3以下の極低出生率を経験しており、2020年の合計特殊出生率は歴代最低値の0.84人であった。  このような状況から、少子化問題は昨今の韓国社会におけるもっとも重要なアジェンダであると言える。2017年に始まった文在寅政権は、これまでの政策的努力が、韓国社会が直面する深刻な少子化問題を解決するのに有効ではなかったという問題認識から、少子化政策のパラダイム転換を宣言した。それは、合計特殊出生率や出生児数の上昇を目標とする国家主導的な既存の出産政策から、国民個々人の生活の質を改善し、個人の選択を尊重するという、個人中心の政策への転換を意味する(류 2018)。この新しいパラダイムにおいて核心的推進課題のひとつとして強調されているのが、仕事と家庭生活の両立支援である。  本研究では、こうした韓国の現状を踏まえ、仕事と家庭の両立を支援するためのワーク・ライフ・バランス政策が女性の出産行動に与える効果を実証し、分析結果から得られた政策的含意について検討したい。 【2.方法】  分析に使用したデータは、韓国女性政策研究院が実施した「女性家族パネル」である。具体的には、第1次(2007年)から第7次(2018年)を用いた。分析対象は、調査時点で賃金労働者として雇われている20歳から45歳までの有配偶者である。マルチレベルロジスティクスモデルを利用し、職場のワーク・ライフ・バランス政策が有配偶女性の出産行動に与える影響を確認した。 【3.結果】   分析の結果、職場におけるワーク・ライフ・バランス関連政策の数が多いほど、有配偶女性の出産可能性が高くなることがわかった。特に、育児休業、産前産後休暇のように出産や育児と直接関係がある支援政策の存在は、女性の出産行動に肯定的な影響を及ぼす。また、勤労時間が短いほど出産の可能性が高くなることも確認できた。 【4.結論】   本研究の分析結果によると、ワーク・ライフ・バランス関連政策が充実しているほど、労働時間が短いほど、有配偶女性の出産行動が発生しやすい。こうした結果は、女性の仕事と家庭生活の両立を支援することが、少子化問題を解決するための有効な手段のひとつになる可能性を示唆する。したがって、有配偶女性の出産行動を促すためには、特に女性の出産と育児に直接関連する政策を通じて職場の環境を整え、女性の出産と育児を支援する必要がある。 【参考文献】 류양지,2018,「저출산 정책 방향 및 핵심 과제」『육아정책포럼』56(0):30-37. (リュヤンジ,2018,「低出産政策方向及び革新課題」『育児政策フォーラム』56(0): 30-37.) 【附記】  本研究はJSPS科研費18H00936の助成を受けたものである。また、韓国女性政策研究院から個票データの提供を受けた。

報告番号13

追加出生についての社会階層論的検討――パネルデータを用いた家族の分析(3)
国立社会保障・人口問題研究所  藤間公太

【1.目的】  近年,少子化を1つの背景として,「多子世帯」に対する支援が政策的な課題となっている.ここで言及する多子世帯とは,「18歳未満の子どもを3人以上扶養している世帯」を指す.たとえば内閣府の「平成30年少子化対策白書」においては,「3人以上子供が持てる環境の整備」として,「多子世帯の経済的負担の軽減」などの施策が挙げられている.背景には,経済的に厳しい状況に置かれた多子世帯が増えているという問題意識がある. 日本国内においては,追加出生の規定要因についての研究は主に人口学の領域で蓄積がなされている.たとえば第3子出生の条件を検討した先行研究では,夫婦それぞれの社会経済的要因や既往出生児の性別組み合わせ,男性の育児参加度や三世代同居などについて,第3子出生との関連が指摘されてきた(坂井 1989; 小島 1995; 福田・守泉 2015; 加藤・福田 2018).  その一方で,追加で子どもを持つという選択に社会経済的な要因がどのようなメカニズムで影響するのかについてさらなる検討は十分に行われていない.本研究では,ブリーンらの相対リスク回避モデルとベッカーの子どもの量と質の相互作用モデルの2つに依拠して仮説を構成する.相対リスク回避モデルは,世代間移動における階級の下降移動回避が,階級間での教育選択の相違を生み出すと論じる.このように,世代間での地位継承のために,より多くの教育投資を必要とする階級とそれほどの教育投資が必要とない階級とに分化する.出身階級間での教育投資の違いは,ベッカーのモデルで定式化される子どもの量と質の認知に影響する.すなわち,子育ての質(quality)を重視する親は子どもの数(quantity)を抑制すると考えられる.  こうした観点から本報告では,社会階層論の視点を用いて,追加出生の規定要因を実証的に検討することを目的とする. 【2.方法】  統計法第33条に基づき提供を受けた「21世紀出生児縦断調査」の第1回調査(2001年)から第15回調査(2016年)の個票データを用いた統計的分析を行う. 【3.結果】  基礎的な分析として,生存時間分析の手法を用いて,パネルデータの観察期間内における第2子~6子までの追加出生のハザード率が,母親の学歴によって有意に異なるかを確認した.カプランマイヤー生存曲線を学歴別に描き,ログランク検定を行った結果,どの追加出生についても母親の学歴による統計的に有意な差が認められた.具体的には,第2子に関しては高学歴ほど出生のハザード率が高い一方,第3子以降では,母親の学歴が中卒であるケースほど出生のハザード率が高かった. 【4.結論】  世代間学歴移動に関して相対リスク回避モデルと子どもの量と質の相互作用モデルを仮定して予備的な分析を行なった結果,おおむね仮説を支持する結果を得た.低階層の親は,下降移動を回避するために,高階層の親と同様の水準で教育投資を行う必要がない.すなわち,低階層の親は,子育ての質の水準が高階層の親ほど高くないため,多くの子どもの出生が大きな費用をもたらすとは認識されておらず,それが追加出生のハザード率の高さをもたらしていると解釈できる. 【謝辞】 本研究は,日本学術振興会科学研究費補助金 (18H00931,18H00936)の助成を受けた研究成果の一部である.21世紀出生児縦断調査の個票データの使用にあたっては,厚生労働省の許可を受けた.

報告番号14

中国における親の長期的な不在経験と普通科高校進学――傾向スコア分析による検討
慶應義塾大学 夏天

【1.目的】 中国では国内移動人口の増加と戸籍移動の制限に伴い,親との長期的な別居を経験する「留守児童」が数多く存在している.このような親の就労目的による国内移動に伴う長期的な不在の経験(「留守児童」経験)が子どもの学業成績に対する不利は先行研究で指摘されているが(Zhou et al. 2014),高校進学などのより長期的な影響については,未だに明らかにされていない.また,「留守児童」経験のあるものとないものの間では,出身地域や親の学歴などの出身家庭の属性の偏りがあるため(Xu and Xie 2015),両者を単純に比較するだけでは,差異が過大に検出される危険性がある.本報告では,傾向スコア・マッチング法を用い,セレクションバイアスを統制し,「留守児童」経験の普通科高校進学に対する効果を検討する. 【2.方法】 分析の際には,親の就労目的による国内移動の情報と高校類型別の進学状況の情報が必要であるため,この情報が含まれている北京大学社会科学調査センターによる縦断調査「China Family Panel Studies」(CFPS)データを用いる.母集団は全国の親族世帯であり,調査対象者は当該世帯内のすべての個人である.層化多段無作為抽出によって標本抽出が行われている.分析対象は,wave5時点で年齢が16-23歳の男女で,中学校を卒業し,父親および母親と家計を共にしており(ひとり親世帯などを除外),使用する変数に欠損値がないケースである(n=1,638). 【3.結果】 傾向スコア・マッチング法を用いて検討した結果,子ども期に親の長期的な不在を経験した者は,普通科高校進学に不利であることが確認された.性別と「留守児童」経験の交互作用の検討も行なったが,有意な効果は見られなかった. 【4.結論】 傾向スコア・マッチング法を用いた分析においても,親の長期的な不在経験が高校進学に対するの不利な効果を確認された.同じ傾向スコア分析を用いたXu and Xie(2015)は,「留守児童」経験が子どものウェルビーイングに及ぼす有意な効果は確認できないと指摘しているが,本報告の結果からは,親の不在の経験は子どもにとって長期的な影響があるといえそうだ. 付記 The data are from China Family Panel Studies (CFPS), funded by Peking University and the National Natural Science Foundation of China. The CFPS is maintained by the Institute of Social Science Survey of Peking University. 文献 Zhou, M., R. Murphy and R. Tao, 2014, “Effects of parents’ migration on the education of children left behind in rural China,” Population and Development Review, 40(2): 273–92. Xu, H and Y. Xie,2015, “The Causal Effects of Rural-to-Urban Migration on Children’s Well-being in China,” European Sociological Review,31(4):502-19.

報告番号15

就労女性の不妊治療状況――「女性労働者の労働実態及び男女平等・健康実態調査」より
成蹊大学大学院 松﨑貴子

【1. 目的】  本報告は, 不妊治療と仕事との両立を困難としている就労女性の支援に繋がる課題をみていくことを目的としている.この調査   の特徴は,不妊治療のみではなく,女性の労働者の労働実態,男女平等,健康実態調査であることから多角的な観点で労働環境を分  析できることである.  不妊治療をしている件数は増えており, 全出生数の6.7%になる. しかも, 不妊治療を行っている35~44歳は, まさに就労してい   る女性がキャリアを積む年代ともいえる.  そのうえ,不妊治療は, 女性の月経周期にあわせるため,病院の治療と仕事との時間の調整が必要になってくる.また, 女性の身体  の治療になるため身体に負担がかかり,そのことに加えて,ゴールの見えない治療であることから, 精神的、肉体的にも仕事との  両立が困難になってくる.(乙部由子2015)  就労女性の不妊治療の状況を多角的労働環境の観点から分析していなかったことから, その視点で分析した. 【2. 方法】  本報告で使用するデータは, 2020年4月から7月までに実施した全国労働組合総連合女性部の「女性労働者の労働実態及び男女  平等・健康実態調査」である.この調査は,全国労働組合総連合女性部が、加盟組織から組合を通して調査用紙を配布・回収を  行ったものである.42,000枚配布し, 集約数は,21単産, 47都道府県から7,829人(回収率18.6%)である.この調査の企画の時   に,不妊治療の項目を入れてもらい,個別の調査データの貸出を受けた.この調査の内容は,雇用形態,収入,労働時間,職種,地域,ポ  ジティブアクション,年休,介護,雇い止めの不安,健康状態,睡眠時間,ハラスメント等と多岐にわたっている.そのデータをクロス  させ分析検討した. 【3. 結果】  まず,雇用形態と受診状況との間に関連があると仮説をたて検討を行った. 雇用形態別に、受診する人数が異なるかどうかを検   定するカイ2乗検定をおこなった。雇用形態は「正規」と「非正規」に分けた.また、受診状況については, 「受診している」  「受診していない」に分けた.「受診している」の中には, 「過去に受診した」「受診している」「受診する予定」を入れた.  そして, 雇用形態と受診状況との間に関連があるかを検討するために、カイ2乗検定を行った結果,有意ではなかった.(X²=  0.526, df=1, n.s.)  次に, 女性の夫の雇用形態「正規」「非正規」「無職・不在」と受診状況との間に関連があるかを検討するために、カイ2乗検   定をおこなった結果, 有意であった.(X²=300.099,df=2 ,p<.0.01)しかし,夫の「正規」「非正規」と受診状況との間に関  連があるか検討したところ有意ではなかった.(X²=0.194,df=1, n.s.) 【4. 結論】  女性の雇用形態を「正規」「非正規」別に, 受診状況「受診している」「受診していない」と関連があるとして、仮説をたてた   ところ、有意な結果は得られなかった.つまり, 女性は, 雇用形態に左右されることなく約1割程度受診しているといえる.しかし,  夫の就業状況と受診状況との関連をみると有意であった.夫の不在,無職の場合は受診していないことは明らかである.だが,夫の  「正規」「非正規」別をみると,女性と同様に受診は雇用形態に左右されないといえる.    文献  乙部由子(2015)『不妊治療とキャリア継続』(株)勁草書房

報告番号16

離島における住民の地域おこしへの関心に社会関係資本が与える影響――北海道礼文島の島内・島外出身者の差異に着目した計量分析
大阪市立大学大学院 片桐勇人

【1. 目的】 少子高齢化・人口流出等を背景として、過疎地域の持続可能性が問題となっており、地域おこしの必要性が叫ばれている。地域おこしは住民の主体性が重要であると先行研究は指摘している。本研究は、住民が抱く地域おこしへの関心に関連する要因を探ることを目的とする。住民の地域参加など地域づくりを分析した先行研究は、社会関係資本論を用いたものが多い。たとえば三隅(2017)は過疎地域の地域づくりにおける地域住民同士の信頼(以下「地域への信頼」)の重要性を指摘している。  発表者は2016年4月から2年間、北海道礼文島で地域おこし協力隊として地域おこし活動に従事し、2019年に島を再訪して住民への聞き取り調査を実施した。聞き取り調査では、地域への信頼を利用して地域おこし活動に取り組む島内出身者と、地域への信頼に頼らず自らの専門知識と島外のネットワークを活用して地域づくりに取り組む島外出身者という対称的な事例がみられた。そこから、先行研究では地域レベルの社会関係資本としてすべての住民にとって一様なものであると仮定されてきた地域への信頼が、実際には地元住民と移住者で解釈の仕方、地域おこしに対して持つ効果が異なるのではないかという着想を得た。そのため以下の2つの問いを立てた。 RQ1) 離島地域の住民において、地域おこしへの関心に影響を与える社会的要因はなにか? RQ2) RQ1の答えのひとつが地域への信頼であった場合、住民が持つ地域への信頼が住民の地域おこしへの関心に与える影響は、島内出身者・島外出身者で異なるか? 【2. 方法】 2017年に実施した質問紙調査「平成29年度礼文町「地域おこし」をめぐる地域住民の意識調査(Ⅱ期)」の個票データを使用する。分析方法は、「地域おこしへの関心(1:最小-4:最大)」を従属変数、「地域への信頼(1:最小-4:最大)」を独立変数、年齢、男性ダミー、島内出身者ダミー、第一次、第二次、第三次産業ダミー、香深村(礼文島の旧村)ダミーを統制変数とした順序ロジスティック回帰分析を行う。また、島内・島外出身者別に地域への信頼が地域おこしへの関心に与える影響の差異を分析するため、地域への信頼と島内出身者ダミーの交互作用を投入する。 【3. 結果】 調査データを分析した結果、 (1)住民が持つ地域おこしへの関心には、住民の地域への信頼という社会関係資本が正の影響を与えていること、(2)回帰式における島内出身者ダミーと地域への信頼の交互作用項と主効果の分析により、住民が持つ地域への信頼が地域おこしへの関心に影響を与えるのは島内出身者のみであり、島外出身者にはそのような効果は見られないことが明らかになった。 【4. 結論】 以上より、地域おこしのための重要な社会関係資本として従来捉えられてきた地域への信頼が、地元住民か移住者かによって地域おこしへの関心に対して持つ効果が異なる可能性があり、その性質についてさらなる研究が必要であると本稿は結論づけた。 引用文献 三隅一人, 2017, 「地域社会における信頼形成の社会理論」三浦典子ほか編『地域再生の社会学』学文社.

報告番号17

村落社会における空き家の意味づけとは何か――新潟県佐渡市の集落を事例として
神戸大学大学院 土取俊輝

【1. 目的】日本の村落社会は、過疎化、高齢化、人口流出といった社会変動にさらされているといわれる。村落研究ではこのような状況を学術が取り組むべき実践的課題であるとみなし、集落の再生や消滅といった、村落社会の目指すべき未来を探る研究が蓄積されてきた[金子 2015]。一方、様々な社会変動に見舞われた村落社会で起こる現象が、常に解決すべき問題であるとは限らないことも指摘されている。例えば芦田[2015]は、過疎化、高齢化がもたらす現象の一つである空き家が、行政や研究者の間で一義的に定義され、問題であると位置づけられている印象は否めないとし、まずは空き家が本当に問題なのか、どこに問題があるのか等を確認することから始めた方がよいと指摘している。村落社会の未来について考える前に、過疎化、高齢化等によって村落社会に生じる様々な現象の状況を把握する必要があるといえるだろう。このような背景を踏まえて本報告では、新潟県佐渡市の集落を事例として、過疎村落にとっての空き家の意味づけを分析することを目的とする。 【2. 方法】本報告は、2013年から新潟県佐渡市の集落において断続的に行っている聞き取り調査によって得られたデータに基づいて分析を行う。当初計画では、ムジナ信仰をはじめとする民俗宗教の実態を先行研究との対比で把握するつもりで開始した調査であった。過疎化、高齢化の影響は著しく、先行研究で報告されたもののほとんどが簡略化されたり、廃止されたりしていた。そんな中で、現地の現状を象徴するテーマとして事後的に「空き家」の問題が視野に入ってきたのである。したがって、調査方法はインフォーマル・インタビューが中心であるが、インタビューの文脈としては、祖先祭祀や「家」の存続といったようなものと直結ではないにせよ結び付けて遂行されている。 【3. 結果】調査地では、集落を離れて都市に移住した人々が、空き家となった家に仏壇や墓を維持するために通うという現象が観察された(詳細なデータは当日報告する)。通説通りに伝統的なものが解体されたのなら、家は売却し、仏壇や墓は遺棄もしくは処分しても不思議ではないが、彼らはそうはしない。それはなぜか。かつては、新来者が集落の住人としての権利を得るために、廃絶したイエの家屋敷や位牌、墓を引き継ぐという慣行があったとされる[岩本 1986]。佐渡の村落社会において、家屋敷、位牌(仏壇)、墓は単なるモノではなく、イエの象徴なのである。しかし、伝統的な佐渡の村落社会で価値の高い家屋敷、仏壇、墓は、現代において高く評価されることはない。空き家を処分する方法の一つに売却があるが、市場経済において先住者の仏壇や墓がある空き家に値段はつかないので、売るためには仏壇等を処分するしかない。このように、空き家を処分するためには、現在では近代的な市場経済の論理に従う必要があるが、それは伝統的な村落社会の論理を考慮して、高い市場的価値を示してくれるものではない。そのため、調査地では空き家と仏壇、墓は値段のつけられないモノとなっており、そのまま維持する人がいるのだと考えられる。 【4. 結論】佐渡の村落社会において空き家とは、伝統的な村落にとっては価値が高いが、近代的な市場経済においては価値がつけられないという相反する二重の意味を持つものであるといえる。事例の一般化や他地域との比較は、今後の課題としたい。

報告番号18

中国農村における土地資本化と「家」・「村」の変容――安徽省龍潭村の事例
東北大学大学院 周玉琴

【1.目的】 1990年代以降の中国では、経済の高度成長が都市の膨張をもたらしている。それに農民の流失と農村の衰退が伴い、中国的な農民兼業化が始まったと述べてもよい。ここに現代中国における「土地資本化」が進行してゆくことになった。本研究では、農地に対して農民がいかなる態度や行動をとるか、自家農業を今後どのようにしてゆこうとしているのか、を明らかにしたい。その中から、とくに、経営権を集めて農業を継続・展開しようとする農村の新たなリーダー像を模索することが、本研究のねらいである。 【2.方法】 調査対象地に選定したのは、中国長江沿江部安徽省の龍潭村である。問題が発生している中国農村に分け入って、農家生活の実情をつぶさに見ると同時に、農村部の権力主体の間の相互作用の実態を調べてみたい。 日本の場合なら、自然村とも呼ばれることのある「村」を単位として、それを構成する「家」の内部的編成とその相互関係をつぶさに観察してゆくモノグラフ的方法が有効であり可能である。が、中国の場合、村の規模が大きいこともあって、日本農村社会学のようにただちに全戸に調査を実施するわけにはゆかない。しかし、上述のような土地使用の関係をめぐって一定の範囲を画することはできる。そこに関連する重要なアクターへのインタビューが調査研究の主要な方法となる。 これらを柱として、インテンシブな質問紙調査をおこなおうとするものである。加えて、歴史的資料の収集も重要な方法となる。 【3.結果】 そこにいう「土地資本化」とは、厳密な定義はないが、全世文が「農地の特定権利主体が、利益を増やすため、その土地の特定権利を資本として営むというプロセスである」と定義している(全世文 2018:4)。 中国では、農地に関する内部の権力主体、つまり、国家、農民集団、集団経済組織あるいは村委会、農家という四つの側面から、地域権力構造が相互作用することが明らかになっている。村内部の土地交換、経営の受/委託関係が、村民の生活と深く関わっていることも特徴的である。例えば、出稼ぎグループ内部の人間関係・親戚関係などが、土地経営権の移転と関係していることが判明した。また、村内部の土地経営権を集中するリーダー、経営権を提供する側の農民とその生活、などなどの利害関係が存在することも明らかである。 【4.結論】土地資本化の過程において、単に国の制度・政策だけでなく、農家の生活実態の影響が大きく作用している。龍潭村における農家は、政府の農業諸政策に対して主体的に対応し、農地利用の最善を尽くす努力をしてきた。そして、農協経営の新しいあり方を模索してきた。しかし、この間において、そのような努力にもかかわらず、多くの農家の家族員は出稼ぎに出て、あるいは移転するようになった。その結果、近年の実態としては、農民継承者の不在、農地放置、農村リーダーにおける経営の悪化が顕著となった。。その過程における「家」と「村」は、その形態と性格も、変化しつつある。 <参考文献>全世文、胡历芳、曾寅初、朱勇、「論中国農村土地の過剰資本化」、『中国農村経済』、2018年7月、4頁。

報告番号19

希少生物・環境教育を媒介とした地域環境保全活動の展開過程および合意形成:山形県飽海郡遊佐町八ツ面川の事例
岩手県立大学 三須田善暢

【目的】 本報告では、希少生物が棲む農村部河川の保護活動を取り上げ、(1)当初の地元関係農家と環境保護活動家との対立から変転して合意形成に導いた過程・諸要因を、グラウンディドな姿勢で複雑な利害関係の諸要因の絡み合いを記述的に分析し、(2)この運動をすすめてきたキーパーソンの「思想」をあきらかにする。(3)それらと関連して、ここでの環境教育実践の含意を考察する。 【方法】 著者は、これまで山形県遊佐町での農村調査に従事してきた。その過程で、鳥海山の岩石採取に反対する環境保護運動に取り組む農民や関係者と知り合い、その展開過程を調査した(三須田 2018)。その運動におけるキーパーソンは、これまで、鳥海山の岩石採取反対運動以外に、希少生物(イバラトミヨ)およびその生息環境を形成する湧水・それを源とする地元河川(八ツ面川)の調査・保護活動に取り組んでおり、くわえて、それらを活用した環境教育にも関わっていた。しかし、このキーパーソンら環境保護活動家が中心となった地元河川の改修を阻止する運動において、初期の1990年代には地元農家は環境保護活動家に対して猛烈な反対の意向が強かった。地元農民にとっては生き物の希少性よりも農業効率や生活環境の整備の方が重要だったのである。だが、その後キーパーソンである環境保護活動家の粘り強い取り組みと、皇族の存在、および地元生徒を巻き込んだ環境教育という媒介により、キーパーソンらと、地元農家および行政との間に河川保護をしていく「合意」が形成されていく。2007年には農水省主催「田園自然再生活動コンクール」で地元農家が構成する「八ツ面川朝日堰流域水路管理組合」が団体授賞をするなど、今日では自然保護活動家と地元農家を巻き込んでの地域的環境教育実践として高い評価を得るようになったのである。さらに補足すると、この環境教育実践は、「詩」を活用した情操教育(鈴木2013)や、トゲウオのいる地域相互の交流活動へと進展しつつある。 こうした過程を追っていくと、「合意形成」をどのレベルで判断するか、また表面的な連携=合意形成の背後にある複雑な状況を把握することが必須であることが見えて来た。すなわち、展開過程をきれいに理論的に「整理」することは、現実の複雑さを把握しかねることにもなりかねない。そのため本報告では、いわゆるグラウンディド・セオリー・アプローチで採られるような個々の具体的事実に寄り添いそこから理論を構築していく手法を取っている。 【結果・結論】 調査の結果、「合意」を形成しえた背景には、(1)皇族という象徴的資源に地元住民を関与させたこと、(2)キーパーソンである保護活動家の側で、対立する人たちと合意形成を行なっていく手法を意識的に活用したこと、(3)その一つとして、子どもたちの環境教育により、利害が世代を超えるものになったこと、がある。この保護活動はその後、地域間交流と「詩」を活用した情操教育にも進んでいき、その過程でキーパーソンはテイミング論等にみられる独自の思想を構築していく。そうしたユニークさがこの保護活動の特徴といえる。

報告番号20

軍用地コンバージョンの比較研究(1)――ドイツ-バンベルクの難民収容事例
大阪経済大学 難波孝志

【1.目的】  本研究は、軍用地コンバージョン(跡地利用)のプロセスについて、国内の事例や、ドイツ・韓国などとの国際比較を行うことを通じて、国家と地方自治体、基礎自治体、そして地域社会の住民諸組織の関係性を、地域社会の権力構造と自治、そして地域の復興・発展という観点から探ることを目的とする。 【2.方法】  ドイツでは、ハイデルベルク、バンベルク、シュヴァインフルト、ハーナウ、ハン・ミュンデンにおいて、行政担当者等への聴き取り、資料収集を行った。韓国では、ソウル市や米軍基地移転・跡地利用に関する文献を収集した。国内では、青森県、東京都、神奈川県、京都府、奈良県、山口県、福岡県、沖縄県での現地調査および自治体等への聴き取りを行った。2020年度以降、新型コロナウィルス禍の影響もあり、海外での現地調査は、延期せざるを得なくなった。この間、ネットによるデータの収集、Zoomなどのソーシャルメディアによるインタビューを行った。本報告では、特に(1)バンベルク(2)舞鶴(3)佐世保(4)小倉の4つの事例を比較検討する。 【3.結果】  バンベルクでは、2014.2にアメリカ軍が完全に撤退した。2014.11連邦不動産局(BImA)が跡地を引き受け、バンベルク市当局と一緒に再開発計画の策定を開始した。ドイツの軍用地コンバージョンは、住民対話によってその骨格が固められる。土地所有者の利益よりは、地域に住む住民の意向を最大限に取り入れて進行していくのである。2015.6までに、バンベルク市当局主導で、8回のワークショップが開催される。住民参加で設計コンペを実施し、市が連邦政府から土地を買い取り、高級住宅街を含むマスタープラン「TargetConcept2035」が策定された。しかし、2015.8バンベルク市議会が難民到着施設の受け入れを決議する。2015.9.5にはメルケル首相がハンガリーで足止めされていた難民の受け入れを宣言し、2015.9.14最初の難民70人がバンベルクの基地跡地に到着する。ここでは、土地利用について最終決定権を持つ市が、時間をかけて策定してきたマスタープランに対して、国家事業としての難民受け入れを優先させた事例を紹介する。 【4.結論】  2015年から2019年の間に、通算すると15,889人(市の全人口77,373)の難民が元基地を改装したバンベルクの難民施設を利用した。また、この間に約5,000人の難民がバンベルク市の居住者となった。ドイツでは、2016年に制定された連邦の統合法によって、①ドイツ語習得支援、②就労・(高等)教育支援、③ドイツ社会への統合が実施される。また、難民が就労のために大都市に集中することを避け、統合の妨げとなるセグリゲーションを防止するために、3年間は最初に割り降られた州に居住することが義務付けられ、各州は新規庇護申請者に対して居住地を指定することができる。バンベルク市は、2016年マスタープランの機能保持を確認、またコンバージョンのために組織された協議会が、市による迅速な買収とその後の都市開発を連邦政府へ要求している。ただ、基地跡地内の難民受け入れ施設を囲むエリアを、連邦警察訓練および高度訓練センター(BPOLAFZ)が使用を開始した。ドイツにおいて、土地利用について最終決定権を持つ市と、難民受け入れという国家事業とのせめぎ合いは、現在のところ国家事業が優先権を持っている。  なお4報告はJSPS科研費19H01581の助成を受けた。

報告番号21

軍用地コンバージョンの比較研究(2)――旧軍港都市舞鶴を事例として
 杉本久未子

【1.目的】  本報告は、舞鶴市を事例として軍用地のコンバージョンと地域社会の関係を把握するものである。報告者は沖縄県読谷村の軍用地およびそのコンバージョンと地域社会の関係を研究しており、この両地域では軍用地をめぐる地域社会の関係が大きく異なることを再認識した。ここでは、海軍鎮守府の形成・発展に伴って近代的な都市が形成され、戦後は軍港都市転換法(1950年)にもとづき造船業など民間企業への転換や公共施設としての活用、そして自衛隊基地としての利用が進んだ舞鶴市の歴史的展開を確認する。さらに、近年の防衛や安全保障、近代化遺産に対する人びとの関心の高まりが舞鶴市の軍用施設をめぐる行政政策や住民の取り組みにどのような影響を及ぼしているかを紹介する。 【2.方法】  舞鶴市は旧軍港都市の一つとして、研究蓄積の多い都市である。研究に際しては、軍港都市に関する先行研究の整理をまず行った。ついで歴史的推移を把握するために舞鶴市史や企業史、自衛隊史などから地域の歴史を確認した。また、フィールドワーク、舞鶴市役所や舞鶴市民等へのインタビューから行政政策や市民生活の現状を把握している。 【3.結果】  海軍鎮守府が開庁された舞鶴では、軍用目的に合わせて、軍港や鎮守府庁舎、海軍病院など直接の軍関連施設だけでなく、海軍工廠=製造業、鉄道や上水道など=インフラ整備など総合的な都市開発が行われた。戦後は、一部が引揚者のための施設として利用されたほか、海軍工廠などの民間企業への提供、水道施設などの市移管、さらには海上保安庁や海上自衛隊の施設に転用された。その結果1980年時点では、8割を超える旧軍用財産が転用されている。  この転用の結果、造船業、ガラス工業、化学工業、木工業などが成立するが、戦後の時代状況のなかで、次第に沈滞していった。海上自衛隊や海上保安庁は、現在も多くの公務員が就業している。このような状況のなか、赤れんが倉庫群や引揚記念館などの旧軍港都市に関連する歴史的遺産が、新たな観光資源として注目を集め、さらに海上自衛隊の艦船が間近に見られる自衛隊施設そのものが集客性を高めている。   【4.結論】  舞鶴市の軍用地のコンバージョンは、海軍工廠や港湾施設、整備されたインフラを活用して日本海側の産業と流通の拠点づくりを目指すものであったと言えるだろう。しかし造船不況など中心となる産業をめぐる状況の変化がそれを困難とした。むしろ旧海軍施設や海上自衛隊の存在は、赤れんが倉庫群の活用から、文化庁の「日本遺産」や日本イコモスの「日本の20世紀遺産20選」の対象、さらには「艦隊コレクション~艦これ~」まで重要な観光資源となっている。住民の旧軍関連施設等への愛着も強いようである。  海軍舞鶴鎮守府開庁120年の今年、コロナ渦でさまざまなイベントは中止されているが、海軍の遺産と記憶、海上自衛隊の存在は舞鶴市のアイデンティティを形成している。

報告番号22

軍用地コンバージョンの比較研究(3)――小倉陸軍造兵廠の事例
吉備国際大学 平井順

【1.目的】  造兵廠跡は戦争遺跡として保存されやすい条件とされにくい条件が共存している.工場建物を残したまま居抜きで転用することも多く,その工場建物や倉庫が近代化遺産の対象になりやすい.大規模な土地区画整理を実施して元の用途と無関係に生まれ変わる転用であれば建物が残らないのと対照的である.いっぽうで,民間所有の工場は生産効率的に合理的であり続けなければならないが,戦前から続く建物が戦後数十年間劣化陳腐化しないことも難しく,建て替えられることも多く保存されにくい.  この報告では,軍用地の跡地転用後の受容過程及びその変遷を考察する.考察の対象は造兵廠跡とする.具体事例として北九州市の小倉陸軍造兵廠跡を取りあげる. 【2.方法】  東京工廠が関東大震災で壊滅的な被害を受け,そのひとつの移転先として1933年に小倉工廠が開設された.1940年に陸軍造兵廠に改称された.終戦を迎えてこの区域は進駐軍に接収された.1950年代後半に順次接収が解除された.そして軍用地コンバージョンが実施された.住宅地図その他の資料を照らし合わせつつその変遷を跡づける.  小倉のほか国内の陸軍造兵廠としては,名古屋陸軍造兵廠,大阪陸軍造兵廠,東京第一陸軍造兵廠,東京第二陸軍造兵廠,相模陸軍造兵廠があった.造兵廠は軍直轄の軍需工場なので,跡地転用もその多くが民間の製造業へ払い下げられた.旧軍国有地の払下げが戦後の高度成長を支えたという議論の多くは,陸軍造兵廠と海軍工廠の跡地転用にかかるものである. 【3.結果】  小倉は1960年代前半に軍用地コンバージョンが実施された.官公署や文教施設などの公共の建物と,緑化整備された公園と,住宅公団による団地供給と,民間の工場等の用途に転用された.面積3千平方メートル以上の用地を取得した企業はいくつかあったが,現在製造業で存続しているのはひとつの企業のみである.それ以外は撤退もしくは他所に転出した.  名古屋は工業地・倉庫として転用されたが,1980年頃から公園・住宅として再開発された.大阪は車両基地のほか,大阪城公園が整備されるとともに,弁天島地区が大阪ビジネスパークになった.東京の一造は陸上自衛隊駐屯地と図書館と公園等になった.二造は大学及び各種学校と公園,集合住宅等になった.工場はほとんど残らなかった.相模は今なお接収されたままであり,米軍相模総合補給廠として存続している. 【4.結論】  小倉の造兵廠跡はもと軍需工場であったことを活かしてかなり広い敷地が製造業に転用された.しかし,次第にこの跡地一帯は工場の町ではなくなってきた.おおむね1990年代頃を境に集合住宅(分譲・賃貸)が広がる市民が生活する街に変化している.製造業へ転用したあと,数十年経過して住宅や公園に転用される.一部モニュメントが残るものの,元軍用地だった記憶が失われていくプロセスが進展している.

報告番号23

「地域づくり」と「閉じこもり防止」の隙間の検討――岩手県X市Y地区における「つながりづくり」をしない高齢者支援を事例に
岩手保健医療大学 大井慈郎

【1.目的】  本稿は,岩手県X市Y地区で行われている活動Aを事例に,地域づくりによる介護予防事業として実施されている高齢者サロンの問題点を指摘し,それを克服するための取り組みの分析を行う.  近年の高齢者の社会的孤立という問題は,介護予防事業の一部として対策されている.生活機能が低下し閉じこもりになりやすい高齢者を対象として,住民運営の通いの場の充実を図る「地域づくりによる介護予防」として全国に多数の高齢者サロンが設立されている.従来の研究では,地域組織化活動や住民運動からの延長線上に位置づけられ,「地域助け合い型」の活動に注目が集まっている.しかしながら,高齢者サロンの取り組みは,普段から地域のイベントに顔を出す,すでにつながりのある住民たちが集まる場になり,本当に参加が必要な人たちが集まる場として機能していないという問題が指摘される.つまり,新規参加者が「知らない人たち」のなかへの参加を忌避することと,サロン側も「知らない人」の参加を好まない,両面の問題である. 【2.方法】  本研究では事例として活動Aを取り上げる.Aは,2018年7月より「町内の方々が気軽に集まれる場所」を目指した取り組みである.毎週金曜日の13時から16時まで地域の会館にボランティアスタッフが常駐し,幼児から高齢者まで自由に出入りできる形をとっている.特にこれまで地域活動に関わりがなかった人を対象として設定しているため,広報活動は大々的に行っておらず,基本的に発起人の民生委員が個別に声をかけている.利用者は,平均すると1回に1~2人である.  2018年10月からAへ継続した参与観察を実施している.加えて2020年の12月に「公園」を立ち上げた民生委員に対しての個別聞き取り調査(約1時間×3回)に加え,「公園」利用者2名に対して聞き取り調査(それぞれ約30分)を行っている. 【3.結果・結論】  「公園」は従来の老人クラブやサークル活動とは異なり,「近所の喫茶店」のように,各々好きな時間に来て,疲れたら帰ることができる場所を目的としている.「拘束しない」という点が重要なポイントだという.この「拘束」には,時間的拘束と,活動的拘束の2つの要素が入っている.  利用者は,誰かに話したいことがあればAに,ふと立ち寄り,一通り話したら帰宅する.利用者が複数人同じ時間にいるからといって世間話をするわけではない.それぞれボランティアの人と話をしている.仲が悪いわけではないが,それ以上にボランティアの人と仲が良いのである.「利用者同士がつながる場」「助け合う場」ではなく,あくまで「家の外で他者と会話ができる場」なのである.  定年まで地域の活動への参加する機会がなかった場合,そこから単身で既存の地域の人間関係の中に入っていくことは容易ではない.社会的孤立の検討に際し,「地域づくり」を目的とした活動が,つながりの外にいた人にとって参加の難易度を高め,かえって閉じこもり防止につながりにくくなる可能性への視点が重要となる.

報告番号24

グローバリゼーションと地域ガバナンス――陶磁器業をめぐる越境的交流活動の比較分析
早稲田大学 太田有子

【1.目的】 本研究報告では、グローバリゼーションに伴う地域の産業活動の変化を検証するため、特定の地域を拠点に発達した陶磁器業における越境的な活動の影響の分析について報告する。陶磁器業は、海外より生産技術を導入しつつ、世界諸地域に流通し、越境的な活動を通じて発展した産業であることから、地域におけるグローバリゼーションの展開を検証するうえで重要な事例といえる。陶磁器業が越境的活動と共に発展した複数地域の比較分析を行い、既存の関係性および産業活動の変化について検証することで、グローバリゼーションの影響とその地域特性を明らかにする。 【2. 調査方法】 陶磁器業の発展した有田地域、名古屋・瀬戸地域における陶磁器業の生産・流通をめぐる関係性について陶磁器業関連の資料を中心に分析を行った。上述地域では、近代以前より陶磁器業が発展し、特に海外市場への積極的な販路拡大をはじめ越境的な活動の拡大を通じて、持続的に発展した地域であることから、グローバリゼーションの過程と地域の産業活動への長期的な影響について分析するうえで重要な事例である。 【3. 結果】 日本の陶磁器の生産・流通活動が越境的に拡大する過程における既存の関係性・活動の変化とともに地域間の相違点が明らかになった。有田地域では、行政の財政・制度的支援による技術導入が進展したのに対して、名古屋・瀬戸地域では、民間資本を中心に、既存の生産技術を活用した生産体制の変化が見られた。いずれの地域も海外市場対象の生産増が顕著であったが、生産・流通過程に参与する主体や財政的基盤に相違点が見られた。地域間の比較分析を通じて、陶磁器業をめぐる既存の関係が、資本調達、技術導入・設備投資の規模等に影響しており、さらに生産・流通活動が越境的に展開する過程における主たる担い手、海外市場との関係性の形成の点において地域間の相違が確認された。既存の生産・流通をめぐる地域の状況に応じて革新的な技術導入よりも既存の生産関係の緩やかな変化が持続的な発展に結びつく事例や地域間の相互作用による変容が生じる場合も見られた。分析対象地域では、いずれも海外市場対象の陶磁器生産が増加し、多くの製品が海外に流通したが、地域によって独自の関係性が生じており、越境的活動の展開のあり方にも作用していた。 【4. 結論】 陶磁器の生産、流通に関わる資源の管理や分配に関与している様々な社会集団の関係性において、越境的な活動の活発化は、必ずしも既存の生産関係を序列化し、一方的な改編に終始するものではなかった。グローバリゼーションに伴い、地域の産業は既存の生産工程・部門の統合や産地移転などの変化を余儀なくされるものとして理解されているが、その過程は一律ではなかったといえる。グローバリゼーションの地域への影響は、既存の生産活動に対して、画一的に波及するものではく、行政の関与や既存の生産・流通活動をめぐる関係性に応じて地域によって多様な展開を示していたことが明らかになった。

報告番号25

ワイン事業新規参入時における地域・産業コミュニティ参入プロセス分析
学習院大学 深見嘉明

本研究は、これまで原料果樹生産ならびに醸造事業に関与がなかった起業家が、ワイン事業に新規参入するにあたって直面する地域ならびに産業コミュニティへの参入プロセスを分析することにより、近年拡大傾向にある多様な特色を有する醸造・蒸留産業の構造を把握するとともに、近年の日本ワインならびにクラフト醸造・蒸留ビジネスの流行が地域に与える影響を明らかにすることを目的とする。分析を実施するにあたり、2020年以降北海道においてワイン醸造事業に新規参入した事業者を対象とした事例分析を、参入者ならびに関係者に対するインタビューを中心とした質的調査にて実施した。今回の分析対象を含む、近年のワインならびにビールやジンなどの酒類市場新規参入する小規模事業者は、地域において生産された原材料を用い、少量ながら特徴のある製品を供給するというコンセプトの産品は、職人による手工業的な製造手法によって製造されたという「クラフト」という呼称によりカテゴライズされている。「クラフト」のコンセプトは、ビールやジンなど、以前は大企業による大量生産が中心であった酒類において21世紀に入ってから生じたムーブメントであるが、テロワール、エステートなどの用語が用いられてきたワイン市場においては、小規模事業者が原材料生産から醸造までを地域内で完結させるという形態を採用する事業者は以前から多く、近年の新規参入者もそうした志向をもつ場合が多い。対象者は販売を含む6次産業化を目指しており、地域内での圃場(原料果樹生産の畑)と醸造施設建設用地を確保しなければならない。それまで蕎麦などが生産されてきた比較的圃場としては価値が低い土地、耕作放棄地をうまく譲り受けられれば圃場を手に入れるための資金はそれほど大きくないが、原材料生産に適した土地の入手には、所有者ならびに自治体との間に圃場を仲介され、農業委員会において許可を得られるだけの関係性を構築することが求められる。よそ者の参入に対しオープンなコミュニティがある地域を選定すること、農業委員会による許可を得るための根拠となる地元の有力者や行政とのパイプを構築することは、参入のための必要条件である。どれだけよい圃場を確保できたとしても、果樹生産ならびに醸造技術の習得は必須である。親族に醸造事業者がいたり、大学/大学院醸造科を卒業していたりしない、多くの新規小規模参入者には技術を体系的に学べる機会は殆どない。例外として長野県塩尻市役所の塩尻ワイン大学、北海道庁の北海道ワインアカデミーなど自治体が運営する教育プログラムも存在するが、新規参入者に対して定員は希少である。こうした教育プログラムを履修したとしても、土壌や気候、品種による差異が前提となるワイン生産において明文化されていないノウハウの習得が製品の競争力醸成に大きく影響する。そこで同業他社であるぶどう果樹生産農家や醸造事業者と関係性を構築し、作業の補助をしながら技術を習得することが望ましいことが判明した。小規模事業者によるワイン事業新規参入者は、地域コミュニティへの参加と同時に、ワイン産業従事者コミュニティへの参加と、コミュニティ内での積極的な活動が求められる。未経験者のワイン事業に対する新規参入において、複数のコミュニティに同時に関与を開始し、ステークホルダーとの関係性を構築することが求められることが明らかとなった。

報告番号26

社会的企業研究におけるメゾレベル分析の意義――社会的企業の⽣態系に関する実態調査(1)
明治学院大学 米澤旦

【1.目的】 本報告の目的は社会的企業に対するメゾレベル分析の意義を示し、社会起業家の社会ネットワーク調査結果による例証を試みるものである。「社会的企業」は経済的目的と社会的目的を両立させるハイブリッド組織として、日本でも2000年代半ばから注目される一方で、その捉えどころのなさによって、組織の特性についての経験的研究の蓄積が進まなかった。この状況に対して、メゾレベル分析(Granovetter 2017=2019)の意義を示す。 【2.方法】 本調査プロジェクトの中心は、東京都と愛知県の起業家支援団体中心とした社会起業家への質問紙調査の分析と関連する社会的企業のスタッフへのインタビュー調査による。 本調査はこれまでの国内の社会的企業の調査分析において中心となっていた、個別事例の研究、または外在的基準による大規模調査の持つ限界を乗り越える可能性がある。個別事例研究では当該組織にみられた特徴がほかの社会的企業と比較したときにいかなる意味を持つかを示すことが難しく、大規模調査では、文脈や組織アイデンティティの異なる組織をひとまとめにしてしまう。これに対して、メゾレベルに注目した研究では、意味世界を関係者間で共有する組織集団を対象とすることができ、現時点での日本の社会的企業の「生態系」(Borzaga et al. 2020)の状況や文脈を共有した組織集団における個別組織の特性を明らかにすることが期待できる。 【3.結果】 調査から明らかにされたことは、社会起業家の所有するネットワーク資源の複層性(社会的企業ごとにネットワーク資源の種類がばらついていること)、開放性(社会的企業内との他のセクターのネットワークが分断されていないこと)、一時性(社会的企業間のネットワークに頼る期間は時間的に限定されていること)といった性格である。 対象とする社会的企業の集団でも、組織の基本的特性(組織年数や組織規模)や起業家個人の特性に関連する形で活用できるネットワーク資源の種類には一定のばらつきが見られた。例えば、設立初期の団体は同類的なネットワークを活用するが、設立後一定期間を経た団体は外部セクターの団体とのリソースを活用することが示されている。 【4.結論】 本研究が示すことは、ある程度の意味世界を共有する社会的企業の集団を対象としたとしても、それらが構成する「生態系」はあまり強固なものとは言えないことや、意味世界を共有した組織集団のなかでの組織特性による資源活用の傾向性の違いである。これらは日本社会のように社会的企業の法制度が整備されていない社会に特有である可能性があり、同時に進行中の法制度が整備された韓国社会での調査への比較によってよりよく理解できると考えられる。 本研究はトヨタ財団 2018 年度研究助成プログラム(助成番号:D18-R-0122)の成果である。 文献 Borzaga, C. G. Galera, B. Franchini, S. Chiomento, R. Nogales and C. Carini. 2020, Social Enterprises and their Ecosystems in Europe. Comparative Synthesis Report. Publications Office of the European Union. Granovetter, M., 2017, Society and Economy Framework and Principles, (=2019、渡辺深訳 『経済と社会――枠組みと原則』ミネルヴァ書房)

報告番号27

リソースジェネレータによる社会起業家の社会関係資本の把握――社会的企業の生態系に関する実態調査(2)
名古屋大学 福井康貴

1. 目的 本報告の目的は、弱い埋め込みを重視する新しい経済社会学の立場から、社会起業家が構築している社会ネットワークと社会関係資本の分布を検討し、ハイブリッド組織としての社会的企業を再考することである。社会的企業のEMESアプローチは、カール・ポランニーの3つの資源配分原理―交換(市場)、再分配(政府)、互酬(コミュニティ)―のハイブリッドとして社会的企業を概念化し、新しい経済社会学とは異なり、社会関係に埋め込まれた領域として、コミュニティを特権化する傾向がある(Evers and Laville 2004=2007)。しかし、社会的企業が、市場、政府、コミュニティのアクターと構築している社会関係や相互作用の把握は、社会的企業のハイブリッド性や領域間の相互作用を実証的に解明するための有力なアプローチの1つだと考えられる。 2. データ・方法 本研究では、2020年、21年に、愛知県と東京都の起業家支援団体に実施した質問紙調査のデータを用いる。愛知調査は2020年9月〜11月に実施され、調査対象は東海若手起業塾の修了者である(調査協力依頼数46名、有効回答数23名)。東京調査は2020年12月〜21年1月にETIC.の「社会起業塾イニシアティブ」プログラムの修了者を対象に実施された(調査協力依頼数118名、有効回答数40名)。本研究では、社会起業家の社会ネットワークと社会ネットワークから獲得した資源をリソースジェネレータ方式で尋ねた質問を用いる(15のネットワーク・メンバー、12項目の相談・支援)。 3. 結果 資源の獲得率は「商品・サービスの質を改善したいとき」「事業アイデアを考えるとき」の獲得率が9割を超えるなど、多くの資源がいずれかのセクターから獲得されているが、「ロビー活動を行うとき」や「事業の評価方法を知りたいとき」の獲得率は低くなっている。平均資源数を比較すると、資源の獲得先は社会セクター(社会起業家、支援団体のアドバイザー、ボランティア団体・市民運動団体の役員)と市場セクター(ベンチャー・中小企業の経営者・役員、大企業の経営者・役員、金融機関の担当者)に偏っており、行政、政治、地域セクターなどから獲得される資源は少ない。この傾向に関して法人形態による大きな違いはなかった。資源の獲得先について、社会セクターと市場セクターを比較すると、社会セクターから獲得される資源が多いが、「融資・投資について相談したいとき」や「人材育成や職場環境について相談したいとき」など一部の資源では、市場セクターも有力な獲得先となっている。 4. 結論 本報告の分析結果は、少なくとも相談・支援という非貨幣的資源に関して、社会起業家の資源獲得先が、同類である社会起業家や起業家支援団体などに偏っていることを示唆しており、社会的企業の資源配分のハイブリッド性に対して、検討の余地があることを示唆する。社会的課題の解決には、市場、地域、行政、政治、学術といった多様なセクターの社会関係資本を動員する必要があると考えられ、多様な社会関係を創出する場の形成や、セクター間の資源仲介機能の強化といった中間支援を構想する必要がある。 付記:本研究はトヨタ財団 2018 年度研究助成プログラム(助成番号:D18-R-0122)、JSPS科研費(17K13840)の研究成果の一部である。

報告番号28

社会的企業への従業員の入職過程の検討――社会的企業の生態系に関する実態調査(3)
目白大学 井口尚樹

【1.目的】  社会的企業の存続のためには、起業家や社会的投資家だけでなく、実際に事業を営む従業員が欠かせない。従業員がいてこそ、社会的価値が実際に創出されるといえる。  では従業員はどのような人々で、彼らはどのようにして社会的企業に入職するのか。また社会的企業への入職の障害は何か。本発表では社会的企業で働く従業員への聞き取り調査をもとに、これらの問いに答えようとする。 【2.方法】  国内の3つの社会的企業の従業員約10名に、聞き取り調査を行った。3つの社会的企業はいずれも創業後10年以上の団体であるが、規模や事業内容は異なる。対象者のうち新卒で入職した者は数名で、大半は伝統的営利組織から転職した者である。聞き取り調査は基本的に事前に定めた質問項目に沿い、対象者の語りに応じ質問を追加する、半構造化面接法で行った。主な質問項目は、入職の経緯、現在の仕事内容、勤務形態およびそれらに対する評価、ソーシャル・セクターの特徴、個人的ネットワークについてであった。 【3.結果】  対象者の多くは、ソーシャル・セクターで働くことへのこだわりを強く抱いて入職しているわけではなかった。多く見られたのは、Amin(2009)の挙げた、キャリア上の障害などに直面した際、たまたま自身のスキルにも見合い満足できる仕事として社会的企業に入職するという類型である。  社会的企業の情報を入手した経路は、一般の転職者向けサービスや、個人的ネットワークが中心で、ソーシャル・セクターに限定した企業探索がなされていたわけではなかった。  もちろん、社会的価値の創出や社会デザインに携わることができること、勤務形態が自身の希望に沿っていることなど、社会的企業に見られやすい要素は、対象者が就業を継続したいと考える要因となっていた。一方で、対象者たちは、自身の働き方を、伝統的営利組織でのそれから断絶したものとはとらえていなかった。  また対象者からは、「ボランティア」「意識が高い人が行く所」といった伝統的営利組織からの断絶を強調するステレオタイプが、入職の障害(周囲による反対、承認欠如)となることが語られた。 【4.結論】 結果からは、労働条件だけでなく、従業員のパーソナリティまでも含んだ像について、一般に抱かれるイメージと実態の間にズレがあり、そのことが、「ソーシャル・セクター」が目指されづらく、敬遠される場合もあることにつながっていることが示唆される。 付記:本研究はトヨタ財団 2018 年度研究助成プログラム(助成番号:D18-R-0122)の研究成果の一部である。 Amin, Ash, 2009, “Extraordinarily ordinary: working in the social economy,” Social Enterprise Journal, 5(1): 30-49.

報告番号29

日本における労働市場の非流動性、安定志向と若年LGBQのキャリア設計
フロリダ州立大学 上野康司

1.目的 近年アメリカで行われたUenoら(2018)の研究によると、レズビアン・ゲイ・バイセクシュアル・クィア(LGBQ)の若者は、キャリアの達成に関して、LGBQであることが不利にならない、むしろ有利になる、という楽観的で前向きな職業観を持っている。この結果は、アメリカにおける流動的な労働市場や、努力すれば困難を乗り越えて経済的成功を収めることができるというアメリカン・ドリームに基づいた言説を反映していると考えられる。構造的・文化的に異なる社会環境では、LGBQの若者はどのような職業観を持つのか、を日本を例にとって、探った。 2.方法 首都圏または近畿圏在住の若年LGBQ (18-25歳)を対象にインタビューを行った(n=43)。 3.結果 • 全般的に、参加者は悲観的な職業観を持っていた。将来の職場では、異性愛規範が強調され、雇用者・同僚は不寛容であると仮定し、LGBQであることを隠して働くことになると考えていた。同じ職場で長く働くことを前提としていることも、その決意を強めた。 • 職業・雇用者を選択する際に、LGBQへの寛容性よりも、キャリアの安定性を重視する傾向があり、多くの参加者が会社員・公務員になることを希望した。 • 参加者がこのようなキャリア設計をした理由を説明する際、次のような日本特有の構造的・文化的要因に言及した。(1)正規社員の労働市場が非流動的であり、新卒時またはそれから数年間の就職活動の結果が、後の職業上での成功を大きく左右すること、(2)キャリアの安定が、キャリアに関しての満足感・人生の幸福感につながると一般的に言われていること。 • 少数の参加者は、LGBQであることを公言し、他のLGBQの人々の役に立つ仕事に就くことを希望したが、このような計画は、現実性の低い「夢」として提示された。 4.結論 上の結果を、アメリカにおけるUenoら(2018)の研究と比較することにより、日米の違いが明らかになった。LGBQのように、社会的に疎外されている人々が、どのようにしてキャリア設計をするのか、を理解するには、その人々を取り囲むマクロ要因、特に労働市場の特性やキャリア達成に関する言説に注目することが大切である。 文献 Ueno, Koji, Abraham E. Pena-Talamantes, Teresa A. Roach, Amanda N. Nix, and Lacey J. Ritter. 2018. “Sexuality-Free Careers? Sexual Minority Young Adults’ Perceived Lack of Labor Market Disadvantages.” Social Problems 65:305–322. (https://doi.org/10.1093/socpro/spx014.)

報告番号30

東大社研パネル調査とコロナ禍における健康と⽣活――東大社研パネル調査(JLPS)データの分析(1)
東京大学 石田浩

【1.目的】  本研究は、東京大学社会科学研究所が2007年より実施している「働き方とライフスタイルの変化に関する全国調査」(Japanese Life Course Panel Surveys)の若年・壮年対象者が、新型コロナウイルス感染症の拡大によって、健康状態や生活意識がどのように変化したのかについて検証する。 【2.方法】  データは「働き方とライフスタイルの変化に関する全国調査」(JLPS)を用いる。日本全国に居住する20-34歳(若年パネル調査)と35-40歳(壮年パネル調査)の男女を対象として、2007年以後毎年追跡している(若年・壮年継続サンプル)。2011年には同年齢の対象者のサンプルを補充(若年・壮年追加サンプル)し、さらに2019年にはそれまでの調査対象者よりもさらに若い20-31歳の若年者を新たに抽出(若年リフレッシュサンプル)し、毎年追跡している。これらのJLPSの対象者に対して、2020年8月下旬より11月上旬にかけてウェブによる特別調査を実施した。この調査は毎年定期的に行っている調査項目に加え、コロナ禍の生活経験に関する項目を含んでいる。ウェブ特別調査の回収率は対象者全体で63.9%(3740名)であった。本分析では、第14波(2020年)までの調査データとウェブ特別調査を用いる。JLPSの特性を活かして、健康に関連する調査項目と生活意識に関する調査項目について時点間変化を分析する。 【3.結果】  主観的健康観、精神的健康状態、仕事・生活満足度、暮らし向き、将来への希望の調査項目について、2020年1月から3月(第14波)と2020年秋(ウェブ特別調査)の2つの調査時点間の変化を分析した。同様な形で2019年1月から3月の時点(第13波)と2020年1月から3月の時点(第14波)の間の変化についても変化パターンを分析した。2つの変化パターンを比較することで、新型コロナウイルス感染症拡大の時期のインパクトがあったのかを検証した。全体的な傾向から言うとコロナ禍で健康と生活に関わる状況が、大きく損なわれたと結論することはできない。少なくともこれらの調査項目に関しては言えば、変化は限られている。新型コロナウイルスの感染症が拡大する以前から、健康と生活に関連して悪い方向に変化している人々が2割程度存在していたことがわかる。コロナ禍を経た2020年秋頃にも、ほぼ同様の比率の人々が悪化を経験している。 【4.結論】  長期的に継続しているパネル調査を用いることで、コロナ禍以前と以後の人々の健康と意識の変化のパターンを比較することが可能になった。全体的な傾向としては、コロナ禍以後に大きな変化があり、感染症の拡大を受けて健康や生活状況が大きく悪化したという明確な傾向はみられなかった。 【謝辞】 本研究は、日本学術振興会(JSPS)科学研究費補助金・特別推進研究(25000001, 18H05204)、基盤研究(S)(18103003, 22223005)の助成を受けたものである。東京大学社会科学研究所(東大社研)パネル調査の実施にあたっては、社会科学研究所研究資金、株式会社アウトソーシングからの奨学寄付金を受けた。パネル調査データの使用にあたっては東大社研パネル調査運営委員会の許可を受けた。

報告番号31

COVID-19のリスク認知と感染予防行動――東大社研パネル調査(JLPS)データの分析(2)
東京大学 大久保将貴

【1.目的】 Akesson et al. (2020)はCOVID-19とリスク認知について分析をしており、以下の知見を得ている。第1に、多くの人がCOVID-19の感染リスクを実際よりも過大に見積もっているという知見である。未知の脅威に対しては、リスクを過大に見積もることが知られており(Slovic 2000)、こうしたメカニズムが作用していると考えられる。第2に、COVID-19に対するリスク認知が高い人ほど、ソーシャルディスタンシング行動をとっていないという知見である。この結果は、一見すると意外かもしれない。なぜならば、COVID-19に対するリスク認知が高いのであれば、COVID-19に感染するリスク要因を回避するため、よりソーシャルディスタンシング行動を心がけると考えられるからである(この考え方の前提には、ソーシャルディスタンシング行動がCOVID-19の感染確率を低くするという信念がある)。Akesson et al. (2020)は、COVID-19に対するリスク認知が高い人ほどソーシャルディスタンシング行動をとっていないという現象を、「宿命的効果(fatalism effect)」と呼んでいる。すなわち、COVID-19の感染力が強いと認知すると、ソーシャルディスタンシング行動をとったとしてもCOVID-19に感染する確率を高く見積もる可能性がある(例えば手を頻繁に洗うなどの対策をしていてもウイルスに感染するというように)。個人がどのような対策をしてもCOVID-19に感染する確率が高いと考えるようになると、ソーシャルディスタンシング行動を避ける可能性がある。こうしたリスク認知とソーシャルディスタンシング行動の間のメカニズムは、ひとつの合理的な「宿命的効果」だと指摘されているのである(Akesson et al. 2020)。 【2.方法】  本稿では、Akesson et al. (2020)を踏まえたうえで、(1)COVID-19に対するリスク認知が人々の属性でどのように異なるのか、(2)COVID-19に対するリスク認知がCOVID-19に対する備えや日常の行動にどのような影響を与えるのか、の2点について分析する。 【3.結果】 東大社研2020年ウェブ特別調査データを用いて分析を行った結果、以下の点が明らかとなった。第1に、多くの人(約87%)が実効再生産数(COVID-19に対するリスク認知)を過大に見積もる傾向がある。第2に、実効再生産数の過大な認知は様々な属性に規定されている。具体的には、男性は女性に比べて、大卒は非大卒に比べて過大認知をしない傾向、年齢が高い人は低い人に比べて過大認知をしない傾向、販売・サービス職と生産現場等の職業は専門・管理・技術職に比べて実効再生産数を過大に認知する傾向がある。こうした実効再生産数を過大に見積もる人は、換気をしたり、外食を控える傾向がある。 【4.結論】 本稿の分析結果は、リスク認知と行動の関連は社会によって異なる可能性を示唆する。 【謝辞】本研究は,日本学術振興会(JSPS)科学研究費補助金・特別推進研究(25000001,18H05204),基盤研究(S) (18103003, 22223005)の助成を受けたものである.東京大学社会科学研究所(東大社研)パネル調査の実施にあたっては,社会科学研究所研究資金,株式会社アウトソーシングからの奨学寄付金を受けた.パネル調査データの使用にあたっては東大社研パネル運営委員会の許可を受けた.

報告番号32

コロナ禍での運動習慣と働き方――東大社研パネル調査(JLPS)データの分析(3)
武蔵大学 林雄亮

1.目的 コロナ禍では運動不足を背景とした健康二次被害が懸念され、特に中高年齢者の生活習慣病等の発症や体力・生活機能の低下リスクが指摘されている。スポーツ庁の発表によれば、意識的に運動・スポーツに取り組むことは、健康の保持・増進だけでなく、ストレス解消、自己免疫力を高めてウイルス性感染症を予防することにも役立つという。本報告ではコロナ禍における運動習慣の実態と、その規定因として主に働き方の影響に着目する。 2.方法 本報告で用いるのは、東京大学社会科学研究所が2007年から実施している「働き方とライフスタイルの変化に関する全国調査(JLPS)」の若年・壮年パネル調査データである。2020年には通常の調査に加えて、コロナ禍での仕事や生活の様子に関する質問を多く含むオンライン特別調査(9〜10月頃)が実施され、コロナ禍以前からの人々の変化を追うことができる設計となっている。 3.結果 ①コロナ禍以前の2019年初旬、②国内でも感染が拡大し始めた2020年初旬、③2020年の1回目の緊急事態宣言下(4〜5月頃)、④2020年の特別調査実施時点(9〜10月頃)の4時点において、日常的に運動をほとんどしない人の割合は男性で①51.1%、②47.9%、③55.7%、④47.0%、女性で①66.8%、②61.6%、③68.7%、④62.6%であった。すなわち男女ともに②と③の時点間で運動をしない人が増加していた。しかしこの2時点間の遷移表を確認すると、運動頻度が上昇した人の割合は男性で21.8%、女性で18.0%、低下した人の割合は男性で24.5%、女性で22.7%であり、運動頻度が全体的に低下したわけではなかった。またこの間に新たに運動を始めたのは男性で9.3%、女性で10.7%存在していた。 ③の時点の運動頻度を従属変数とし、年齢、学歴、従業上の地位、リモートワークに関する変数を独立変数とした多変量解析を、性別、配偶者の有無ごとに実行したところ、配偶者のいない男性では有意な効果を示す独立変数はひとつもなく、配偶者のいない女性では正規雇用に比べて自営業・家族従業者が正の有意な効果を持っており、加えて緊急事態宣言下でのリモートワークの経験も正の有意な効果を持っていた。リモートワークの正の効果は既婚の男女でも確認され、さらに既婚女性でのみ、配偶者のリモートワークも正の効果を持つことが明らかになった。 4.結論 コロナ禍における運動習慣の変化は、運動頻度が上昇する者、低下する者がそれぞれ存在するなかでの総合的な結果であった。この時期の運動頻度に対して年齢や学歴、従業上の地位はほとんど影響を持たないのに対し、リモートワークの経験という新しい仕事のあり方が影響を持っていることは重要な知見である。さらに既婚女性の運動頻度には夫のリモートワークの経験が影響しており、夫本人の運動習慣だけでなく家庭の運動習慣を向上させるきっかけとなっていることが示唆された。 謝辞 本研究は、日本学術振興会(JSPS)科学研究費補助金・特別推進研究(25000001, 18H05204)、基盤研究(S)(18103003, 22223005)の助成を受けたものである。東京大学社会科学研究所(東大社研)パネル調査の実施にあたっては、社会科学研究所研究資金、株式会社アウトソーシングからの奨学寄付金を受けた。パネル調査データの使用にあたっては東大社研パネル運営委員会の許可を受けた。

報告番号33

コロナ禍における社会的孤立リスクの格差――東大社研パネル調査(JLPS)データの分析(4)
東京大学 石田賢示

1.目的  2020年3月11日にWHOが(パンデミック)に至ったと宣言したCOVID-19(新型コロナウイルス感染症)は、日本社会においても生活のあらゆる側面に重大な影響を及ぼした。海外のロックダウンとは異なり強制的な行動制限は日本ではおこなわれなかったが、外出自粛への強い呼びかけに多くの人々が応じた。経済活動だけでなくさまざまな社会的活動・交流の機会も激減した結果、社会的孤立や孤独が世界的に問題視されている。  さまざまな速報的知見が蓄積される一方、COVID-19の流行前と比べて孤立のリスクがどの程度拡大したのか、その程度について社会的背景による格差が存在するのかについては研究が少ない。これらの点を検証するためには、コロナ禍前後の状況を把握できるパネル調査が必要となる。そこで本報告では、孤立の指標として社会ネットワークサイズに着目し、その推移と孤立リスクの背景を探ってゆく。 2.方法  分析には、JLPSデータを用いる。2007年のWave1から、2020年1月から3月にかけて実施されたWave14までのデータに加え、2020年8月から11月(回答時期のほとんどは9月から10月)にかけて実施したウェブ特別調査データ(対象者全体での回収率は63.9%)を用いる。アウトカム変数は社会的孤立か否かの2値変数であり、Wave3、Wave12、Wave13、Wave14、ウェブ特別調査の5時点で使用可能である。対面、通話、メール・メッセージアプリのすべてについて、接触・連絡をもつ人数が0人である場合を孤立とみなした。共変量として、就業状態や世帯構成、学歴、居住都市規模などを用いる。推定方法は線形の固定効果モデルである。 3.結果  2009年から2020年1-3月までの孤立割合はほぼ一定に推移していたが、ウェブ特別調査が実施された8月から11月にかけて2倍程度に急増した。対象者の属性にもとづき観察期間内での孤立の予測確率を求め、リスクの高い群、低い群に分けて孤立割合の推移をみたところ、孤立割合の上昇は高リスク群でより顕著であった。調査時期と孤立リスクの交互作用効果を線形固定効果モデルで検証したところ、2020年8-11月時期に格差が拡大していることが明らかとなった。 4.結論  以上の結果は元々孤立しやすかった人がより孤立しやすくなるという負のスパイラルを示唆している。今後の調査の蓄積により、拡大した孤立リスクの格差がどのように推移してゆくのか、また社会的孤立が他の生活状況にどのように影響しているのかを検証することができるようになる。 【謝辞】本研究は,日本学術振興会(JSPS)科学研究費補助金・特別推進研究(25000001,18H05204),基盤研究(S) (18103003, 22223005)の助成を受けたものである.東京大学社会科学研究所(東大社研)パネル調査の実施にあたっては,社会科学研究所研究資金,株式会社アウトソーシングからの奨学寄付金を受けた.パネル調査データの使用にあたっては東大社研パネル運営委員会の許可を受けた.

報告番号34

学校外教育は大衆化したのか――東大社研パネル調査(JLPS)データの分析
東北大学大学院 眞田英毅

【1.目的】 本研究の目的は,中学校時代の学校外教育経験の規定要因をコーホート間で比較することにある.少子化が叫ばれる現代において,1980年代から急激に事業を拡大させたのが学校外教育である.中でも学習指導をメインとする学校外教育(とりわけ塾・予備校)は「影の教育」とも呼ばれ,受験勉強のために多くの親が子どもを通わせてきた.これは塾の大衆化とも呼ばれ,いまでは高校受験を控えた中学3年生の約6割が通塾しているという.このような学校外教育の拡大は,誰が学校外教育を利用するのかといった研究の発展にもつながり,学校外教育の利用と出身階層の正の関連などが確認されている.しかし,長期的な学校外教育経験と出身階層との関連の推移は明らかにされておらず,学校外教育の大衆化が階層間格差の拡大あるいは縮小を促したのかは明らかではない.本研究では,このような学校外教育の大衆化について,コーホート間の比較が可能な全国調査データを用いて分析をおこなう. 【2.方法】 本研究で用いるデータは,東京大学社会科学研究所パネル調査プロジェクトが実施している「働き方とライフスタイルの変化に関する全国調査」(Japanese Life Course Panel Surveys―JLPS)である.この調査は,2006年12月時点で20−34歳の若年層と35-40歳の壮年層を対象とし,2007年から毎年同一の回答者に質問紙調査を行っている.また,2018年時点で20−31歳の新しいサンプル(若年リフレッシュサンプル)を抽出し,元のパネル調査と共通の質問紙で調査を実施している.本研究でもこれらの利点を活かし,コーホート間の比較を行う.分析手法は中学校時代の塾・予備校の経験の有無を従属変数とした二項ロジスティック回帰分析を用いる. 【3.結果】 2変数の関連から確認する.対象者を3つのコーホートに分けた場合,通塾経験は一番若いコーホートが約60%と高く,年齢の高いコーホートでは順に割合が少しずつではあるが低くなる.この傾向に留意しつつ,多変量解析を行い,規定要因について確認した.コーホート別に分析を行った結果,若いコーホートでは学歴の正の効果はみられない一方,家庭の暮らし向きや文化資本の正の効果は維持された. 【4.結論】 本研究は,学校外教育の中でもとりわけ塾・予備校に着目し,その規定要因についてコーホート間の比較を行った.結果,コーホート間で出身階層の効果は一部みられなくなったものの,家庭の暮らし向きや所有財の効果は一貫してみられた.これより,塾・予備校の大衆化は親学歴にもとづく格差を縮小させているが,階層の効果が消えたとはいいにくい.よって,学校外教育は大衆化というよりはむしろ高階層が階層の再生産に向けて利用する保険のようなものになりつつあるのかもしれない. 【謝辞】 本研究は,日本学術振興会(JSPS)科学研究費補助金・特別推進研究(25000001, 18H05204),基盤研究(S)(18103003, 22223005),特別研究員奨励費(20J10278),および東北大学人工知能エレクトロニクス卓越大学院プログラムの助成を受けたものです.東京大学社会科学研究所(東大社研)パネル調査の実施にあたっては,社会科学研究所研究資金,株式会社アウトソーシングからの奨学寄付金を受けました.パネル調査データの使用にあたっては東大社研パネル運営委員会の許可を受けました.

報告番号35

女性の働き方と意識の変容――東大社研パネル調査(JLPS)データの分析(6)
東京大学 黒川すみれ

【1.目的】 本研究では、パネルデータを用いて多様化する女性の働き方を記述するとともに、働き方の変化が階層帰属意識などの意識とどのように関連していたのかを検討する。1990年代のバブル崩壊による不況や潜在的な労働力不足の懸念が契機となり、女性はさまざまなかたちで労働市場に参入した。女性の基幹労働力化が重視され、総合職のフルタイム労働に従事する女性や、派遣社員などの非正規雇用で就業する女性が増え、女性が歩むキャリアは多様化した。また、女性のキャリアは結婚・出産といったライフイベントとも依然として大きく関わっており、女性個人の属性や地位だけでなく、家庭の状況とも大きく関わっている。さらに、職業はさまざまな意識変数の規定要因となっていることも知られており、働き方が個人の意識に及ぼす影響も多く検討されてきた。本研究ではパネルデータの特性をいかし、直近の14年間の女性の働き方を概観したうえで、キャリアの形成と同時進行する家庭要因や意識の変遷との関連を検討する。 【2.方法】 使用したのは東京大学社会科学研究所が実施する「働き方とライフスタイルの変化に関する全国調査(JLPS)」であり、2007年から2020年までの若年・壮年パネルデータを用いる。職歴データの系列分析を行うことで、14年間の女性の働き方を雇用形態に着目しながら記述し、キャリアパターンを抽出する。記述したキャリアパターン別に世帯年収や配偶者年収の推移を比較することで、女性の働き方が家庭状況とどのように連動していたのかを検討する。また、意識についてもキャリアパターン別にその推移を比較することで、意識と行動(働き方)がどのような関連を持っていたのかを確認する。そのうえで、これまでの働き方が現在の意識に及ぼす影響を回帰分析により検討する。 【3.結果】 分析の結果、女性の働き方は6~7つに類型化でき、同じ働き方を14年間継続しているパターンや、雇用形態の転換(正規雇用から非正規雇用への転換)、就業の中断・開始を経験しているパターンなど、女性の多様な働き方の具体的内容を記述した。女性の働き方は配偶者(夫)の年収とゆるやかな関連があった。配偶者年収の水準がある程度高く安定していると女性の働き方の変化は少なく、同じ就業形態を継続するが、配偶者年収が下降したり上昇が止まるという動きがあると、女性の働き方に変化が生じる傾向があった。また、階層帰属意識などの意識に対しては、過去14年間に無業からパート・アルバイトとして就業を開始したパターンや、パート・アルバイトとしての就業を継続してきたことが、階層帰属意識を低める有意な効果を持っていた。 【4.結論】女性の働き方は多様かつ流動的であり、特に有配偶女性は家計状況の調整のために働き方を変えていることが改めて確認された。また、働き方によって意識が大きく連動することは確認されなかったが、過去の働き方が現在の意識に影響を及ぼしていた。 【謝辞】本研究は,日本学術振興会(JSPS)科学研究費補助金・特別推進研究(25000001,18H05204),基盤研究(S) (18103003, 22223005)の助成を受けたものである.東京大学社会科学研究所(東大社研)パネル調査の実施にあたっては,社会科学研究所研究資金,株式会社アウトソーシングからの奨学寄付金を受けた.パネル調査データの使用にあたっては東大社研パネル運営委員会の許可を受けた.

報告番号36

親からの支援が及ぼす若者の離家・再同居への影響――東大社研パネル調査(JLPS)データの分析(7)
東京大学 俣野美咲

1.目的  本報告の目的は、親からの支援が及ぼす若者の親世帯からの独立への影響について検討することにある。1990年代以降、先進諸国では、若年層の間で親元からの離家の遅れや離家後の再同居が増加している。その背景には、高等教育進学率の上昇や未婚化・晩婚化の進行、若年労働市場の逼迫などの社会状況の変容がある。こうした状況の中で、欧米諸国の研究では、若者の親元からの「巣立ち」において親からの支援が重要な要因であることが明らかにされている。日本社会においては、親からの支援が及ぼす若者の離家や再同居への影響について、これまで検討されてこなかった。そこで本報告では、親からの支援を受けることが若者の離家や再同居の経験にいかなる影響を及ぼすかを検証する。 2.方法  分析に使用するデータは、東京大学社会科学研究所が実施するパネル調査「働き方とライフスタイルの変化に関する全国調査」(東大社研パネル調査)のWave1〜14(2007〜2020年)のデータである。分析対象は、35歳未満で少なくとも父母のいずれかが健在のケースであり、親からの経済的/非経済的支援が離家イベントおよび再同居イベントの発生に及ぼす影響について検討する。 3.結果  分析の結果、親からの経済的支援は、男女ともに離家の確率を高め、再同居の確率を低くすることが示された。また、非経済的支援の影響は性別や支援の内容によって異なっており、女性において親から人間関係の相談や仕事の紹介などの情緒的支援・実践的支援が離家を促進することも明らかとなった。 4.結論  親からの経済的支援は、家賃や生活費などの暮らしにかかる金銭的コストの低減につながり、親元を離れやすくさせる。さらに、家を出た後も独立した暮らしを維持しやすくなり、親の家に戻る可能性が低くなる。また、親からの情緒的・実践的な支援も独立を促す場合がある。このことから、日本社会においても欧米と同様に親からの支援が若者の親世帯からの独立において重要な要因となっていることが明らかになった。  しかしこのことは、若者の住まいの独立の機会における出身階層間格差の存在を示唆する。公的な支援が乏しい中では、親からの私的な支援が重要となるが、誰もが同じように親から支援を受けられるわけではなく、親の社会経済的資源の多寡によって支援の受けやすさは異なる。したがって、若者が私的支援を活用することで出身階層間での格差が生じる可能性がある。 【謝辞】本研究は、日本学術振興会(JSPS)科学研究費補助金・特別推進研究(25000001,18H05204),基盤研究(S)(18103003, 22223005)の助成を受けたものである。東京大学社会科学研究所(東大社研)パネル調査の実施にあたっては、社会科学研究所研究資金、株式会社アウトソーシングからの奨学寄付金を受けた。パネル調査データの使用にあたっては東大社研パネル運営委員会の許可を受けた。記して感謝申し上げる。

報告番号37

軍事基地を起因とした有機フッ素化合物汚染はどのように乗り越えられるか――環境アクティヴィズムによる知識生産実践と地位協定
専修大学 森啓輔

【1.目的】  本報告では、環境問題と軍事問題が交錯する中で、有機フッ素化合物(PFASs)汚染が問題化されていく過程を社会学的に明らかにする。考察対象は、2016年から2020年までの沖縄のPFAS汚染をめぐる、環境活動家や地元メディアによるPFASに関連する知識の生産と流通過程である。  PFASは、非水溶性の化合物で防水塗料や消火剤として広く使用されてきた。一方でその性質上、環境中や人体内でほとんど分解・蓄積されないため、がんや発達障害、免疫機能障害、不妊症などとの関連性が指摘されている(US EPA OCSPP 2015)。軍事基地の航空機や工場の火災に対してPFASを含む消火剤が使用され、消火訓練が定期的に行われてきた。  これらは、科学社会学で述べられる二つの科学的知識、すなわち(1)環境規制当局が、化学物質の環境への影響に関する企業の研究を開示することなく、新しい化学物質の使用を認可する「見えざる科学 unseen science」の問題と、(2)関係する地域社会や社会運動のニーズがあるにもかかわらず、規制プロセスが具体的な環境汚染の状況に合った科学的評価を提供しないという「成されない科学undone science」の問題が議論されてきた(Cordner et al 2019; Richter 2018)。  軍事基地に起因するPFAS汚染は、SOFAによる米軍基地運用の政治的遮蔽戦略(川名2020) がこれに加わることで複雑化する。日本の安全保障行政は米軍の作戦に関連する情報をほとんど開示しない。その結果軍事基地に起因するPFAS汚染は、受入国の基地周辺住民にとって大きなリスクとして浮上する。このような状況において、環境アクティヴィズムはどのようにして現状の障壁を乗り越えようとしているのか。 【2.方法】  報告では第一に、2016年から2020年までのPFAS問題の経過を辿りながら、沖縄のPFAS汚染対策にどのような構造的障壁が存在するかを分析する。第二に、環境活動家がPFAS問題を軍事環境汚染として対峙する中で、活動家がどのような戦略を動員して情報源を獲得し、それらを公開してきたのかをインタビュー調査によって明らかにする。 第三に、地元の新聞がPFAS問題をどのように報道したのかを、定量的なテキスト分析によって明らかにする。 【3.結果と結論】  沖縄におけるPFAS汚染の問題化は、沖縄県企業局による北谷浄水場の水源地の汚染から始まった。軍事的な環境汚染問題に取り組んできた環境アクティヴィストらは、必然的に知識生産活動の焦点をPFAS問題に合わせることになった。 その戦略は以下の通りである。(1)政治的遮蔽の突破、(2)言語的障壁の突破と知識流通の促進、(3)直接ヒアリングによる行政ネットワークの追跡、(4)地元マスメディアとの連携。また地元の新聞紙も、環境アクティヴィストらの知的生産をローカルレベルで流通させる役割を果たしてきた。  地位協定の遮蔽戦略は、「見えざる科学」と「成されない科学」と絡まり合いながら地域の環境汚染の解決を困難にしている。他方で、環境アクティヴィストらの戦略は、地位協定の存在にもかかわらず、科学的な知識生産を通した汚染原因の突き止めは可能であることを示している。

報告番号38

紀伊半島の原発反対運動の展開――熊野市の原発反対運動の参照点として
神戸女学院大学 景山佳代子

【1.目的】 本報告の目的は、全国で集中的に原発計画が立てられた1970年前後に、原発のおことわりに成功した、紀伊半島での住民反対運動の特性を明らかにすることにある。 1971年10月に原発計画が公表された三重県熊野市では、計画公表直後に住民の反対運動が生まれ、わずか4ヶ月で原発反対の市議会決議につながった。ただその後も原発計画は執拗に浮上し、16年間で5度の市議会決議を経てようやく終止符を打った。 原発反対運動は往々にして長期化し、運動の切り崩しに合う中で住民たちが対立し、地域社会に修復困難な傷跡が残される。ところが、熊野市の住民運動はそうした亀裂を「生まなかった」ということが一つの特徴としてあげられる。なぜ熊野市でそのような住民運動が展開できたのかを、熊野市に先んじて行われていた紀伊半島の原発反対運動との関係性において分析していく。 【2.方法】 熊野市で原発計画が公表される以前に、同じ紀伊半島の三重県・芦浜、和歌山県・日高、古座、那智勝浦でも原発計画が浮上し、反対運動が展開されていた。とくに芦浜が候補地とされた1963年は、まだ原発のことはほとんど知られておらず、芦浜原発反対運動は日本での本格的な反原発運動の嚆矢とされる。三重県および和歌山県で繰り広げられた原発反対運動の経緯とそのあり方をたどり、熊野市での、住民を「分裂させない」反対運動という初期設定が、いかにして可能になっていったのかを検討していく。 【3.結果】 芦浜原発の反対運動は、三重県下のほとんどの漁協が参加する原発反対漁民大会が開かれるなど、三重県漁民たちに原発とはなにかを広く知らせる機会となっていた。と同時に、「板子一枚下は地獄」という海の上で、何百年も助け合いながら生きてきた漁民たちの人間関係が、原発のために破壊されていく様を、三重県内外の漁民たちは目撃することにもなった。これは熊野市の漁民たちにも大きなインパクトをもって受け取られた。 一方、和歌山県の古座や那智勝浦の原発計画では、隣接自治体である太地町が主導し、熊野灘沿岸の漁協に反対運動が広がっていった。さらに「子供を守る母親の会」という女性を中心とした運動は、熊野市に隣接する新宮市まで広がり、原発のおことわり成功につながっていた。 熊野市の住民たちは、芦浜での原発反対運動、そして和歌山での反対運動から学び、自分たちの運動を設定していった。 【4.結論】 熊野市は、国や県といった行政の「中心」からみれば、「陸の孤島」のようにみえる。しかし、市町村といった既存の行政区分ではなく、すでに中世にその重要な地域基盤が形成されていた「熊野」という活動圏において、熊野市の住民運動を眺めてみれば、それが他地域から隔絶された「閉じた」運動ではなく、熊野市の外との「開かれた」関係性をもった運動であったことが見えてくる。 紀伊半島の熊野灘沿岸地域で連綿と培われてきた「熊野」という歴史的・経済的・社会的なつながりが、原発候補地となった自治体に限定されない原発反対運動の広がりと学びを可能にしていた。

報告番号39

「発話困難な重度身体障がい者」が介助者と協働で行う当事者研究 ――介助関係への影響と社会変革の可能性
日本学術振興会特別研究員PD 天畠大輔

【1.目的】 発表者は発話困難な重度身体障がいの当事者である。自ら発話することはできないが「あ、か、さ、た、な話法」を用いて意思を伝え、介助者との協働による当事者研究で博士論文を執筆した(天畠 2020)。当事者研究とは、先行研究によると、依存症自助グループの当事者の内面に焦点を当てた活動と、当事者運動の社会を変えようとする活動の両方の要素が備わっている(熊谷 2020)。一方、重度身体障がい者の自立生活は、介助者との関係性の“よどみ”が生じる「その後の不自由」と呼べる構造的な問題がある(渡邉 2021)が、それと付き合うための言葉は蓄積されていない。 そこで本研究では、介助者と協働で行う当事者研究の経験を振り返ることで、当事者研究が介助関係に与えた影響や条件を明らかにし、介助者とともに当事者研究を行うことの新たな意義を示すことを目的とする 。 【2.方法】 本研究では自己エスノグラフィ(沖潮 2013)を参照枠とし、発表者が自己の経験を語り、引き出し、対象化して分析する。沖潮の方法と異なるのは、語りの分析だけでなく、テキスト化も発表者と介助者の協働で行なった点である。特に介助関係に焦点を合わせ、障がい当事者と介助者、各々の個別の観点だけでなく、両者の視点を掛け合わせた分析を行う。 【3.結果】 第1に、発表者が当事者研究に至るまでの経緯から、発表者と介助者への効果が明らかになった。発表者は論文執筆チームを結成し当事者研究を行うことで介助関係を継続させることができた。一方、介助者は発表者との介助関係を見直す契機となり、介助関係問題に向き合う関係が生まれた。 第2に、外在化と免責のプロセスがわかった。発表者は当事者研究を通して問題の設定を外在化し、介助者はその問題に深く関わることで当事者意識が芽生え、最終的には両者が協働で問題の責任を引責する関係へと変わった。 第3に、当事者研究を可能にした条件として、安全・安心な場と文章化支援に専念する遠隔地の介助者の存在が浮き彫りになった。 【4.結論】 「発話困難な重度身体障がい者」が介助者とともに行う当事者研究は、テキストとして残すことで、お互いの関係性を再帰的に振り返ることができる。また、当事者研究を論文化し発信することは、個人の困難がいかなるものであり、その支援の合理性を考える際の指針を提供するものになる。本研究では、そのプロセスを詳細に示すことができた。 当事者研究は「個人モデルから社会モデルへの転換」を最小単位で実現し、社会規範を変える糸口を秘めている。以上により、当事者研究の新たな意義として、テキスト化による介助関係の突破口と、社会変革という可能性を見いだすことができた。 熊谷晋一郎,2020,『当事者研究――等身大の〈わたし〉の発見と回復』岩波書店. 沖潮(原田)満里子,2013,「対話的な自己エスノグラフィー――語り合いを通した新たな質的研究の試み」,『質的心理学研究』12: 157-75. 天畠大輔,2020,「『発話困難な重度身体障がい者』の論文執筆過程の実態――思考主体の切り分けがたさと能力の普遍性をめぐる考察」,『社会学評論』71(3): 447-65. 渡邉琢,2021,「自立生活、その後の不自由――障害者自立生活運動の現在地から」,『現代思想』49(2): 164-180.

報告番号40

1960年代末ニューヨーク、オルタナティブ・スペース・ムーブメントの分析――アート業界における制度批判の社会運動
大阪市立大学 笹島秀晃

【1.目的】 本報告では、1960年代末ニューヨークで発生した、アーティストを中心とするアート業界内の制度批判の運動であるオルタナティブ・スペース・ムーブメントを、「アートと社会運動」という観点から検討する。 オルタナティブ・スペース・ムーブメントとは、1960年代アメリカのヴェトナム反戦運動や公民権運動の時代、多様なマイノリティによる異議申し立ての文脈のなかで、ニューヨークで活動するアーティストが作品展示機会の公平性や、また選出過程の透明性をもとめて、主要ミュージアムに対して異議申し立てを行った活動の総称である。当時、メトロポリタン美術館やニューヨーク近代美術館で展示される作品は白人男性のものに大幅に偏り、理事会などの組織運営も富裕な特権層からなることが批判された。運動はミュージアムに対する制度批判から始まり、アフリカ系、プエルトリコ系それぞれのコミュニティにおける展示空間の創設、女性アーティストの作品のみを扱う展示空間の創設など、その範囲は幅広い。 アートと社会運動の関係性については研究が蓄積されつつあり、社会運動の発生、活動の進展のなかで担うアート役割が分析されてきた。アメリカ・公民権運動におけるプロテストソング、1980年代後半のエイズ・アクティビズムの代表であるAct UpにおけるGran Furyによる一連のポスターはその典型的な事例である(Reed 2005)。アートと社会運動の関係について概括的に論点を整理したRon Eyermanは、運動におけるメンバーの動員やフレーミング、メンバー外の社会に対する理念や感情の発信などの点に注目して社会運動におけるアートの役割についてまとめている(Eyerman 2015)。 【2.方法】 本報告では、こうしたアートと社会運動に関する既存の理論的枠組みをもとに、オルタナティブ・スペース・ムーブメントにおけるアートと運動の関係性を分析する。具体的に4つの運動を取り上げる。アーティストの権利を重視しつつヴェトナム反戦運動も展開したArt Workers Coalitionとthe Guerrilla Art Action Group、アフリカ系アメリカ人のアートを中心としたThe Black Emergency Cultural Coalition、そして女性アーティストの展示の権利を主張したWomen Artists in RevolutionとAd Hoc Women Artists’ Committeeである。これらの運動は、明確なメンバーシップなど組織化の点では不十分であったが、相互に連携しつつ制度批判のデモ活動を行った。報告では、運動の争点、組織化の過程、他の組織との連携を含む運動の制度化など、基本的な事実認識を一次資料・二次資料にもとづき記述しつつ分析する。 【3・4 結果と結論】 オルタナティブ・スペース・ムーブメントは、展示機会の公平性をもとめる運動であったが、コミュニティの表象や職業としてのアートの意識の高まりなど、多様な争点があった。さらに、アーティストによる社会運動であったが、動員やフレーミングといった運動の組織化の過程には、アート作品がほとんど使われていないことが明らかになった。 文献 Ron Eyerman, 2015, “The Art of Social Movement,” D. D. Porta and M. Diani eds. The Oxford Handbook of Social Movements, Oxford University Press. T.V. Reed, 2005, The Art of Protest: Culture and Activism from the Civil Rights Movement to the Streets of Seatle, University of Minnesota Press.

報告番号41

原発被災地で再び漁業を始める理由――福島県浪江町請戸漁港の事例
東北学院大学大学院 庄司貴俊

1 目的  本報告では,原発被災地域である請戸漁港を事例に,原発から至近距離に位置し,かつ当該地区に住まうこともできなくなったにもかかわらず,なぜ当該地区で再び漁業を始めたのか,その理由を明らかにする.2011年に福島県で発生した原発事故は,多くの住民から故郷や生業を奪うことになった.本報告が対象とした地区は,津波により壊滅的な被害を受け災害危険区域に指定された.さらに,原発から20km圏内に位置していることから警戒区域にも指定された.  当然,地区の生業である漁業はおこなえる状況ではなかった.震災前90隻あった漁船は津波によりそのほとんどが消失した.請戸港は福島県内では有数の好漁場をもち活気にあふれていたが,震災の日を境にその姿は大きく変わった.それでも事故の発生から6年が経過した2017年に原発に関する区域が解除されると,漁船を地区に入港できるようになり,漁についても制限がありつつもおこなえるようになった.結果的に,27隻の漁船が請戸で再び漁業を始めた.  注目したいのは,事故後の漁船の数が表すように,再開にあたり生活のためという経済的理由は必ずしも該当するわけではない点である.むしろ,①先行きが不透明な状況,②風評の問題,③船を失った人は再建のために大きな経済的負担をしなくてはならない,④遠方に避難した場合はそこから通わなければならない,といったように撤退する理由については多くあげられる.それにもかかわらず,なぜ再開したのだろうか.本報告では以上の問いについて考察する. 2 研究方法  対象地区の住民に聴き取りを行い,調査で得た知見をもとに考察する. 3 結果  調査の結果,漁業の再開により2つの関係が構築されることが判明した.ひとつは海との関係である.漁という目的のために海と関係を築くことで,たとえば海に出ると「スカーっとする」といったように気持ちの面で大きな変化があった.くわえて、海に出た際に良い漁場を探すこともしている.日々海の変化を感じ取り対話するように良質な漁場を探索することは,漁師にとって楽しみを覚える瞬間でもあった.しかし,組合が決めた海に出る日を守るなど,たとえ漁師でも全てが自由なわけではない.その背景にはいまひとつの関係がある.それは漁師同士の関係である.この関係は非常事態の際に現れる.たとえば台風が襲来する前に協力して船を固定したり,船が転覆や遭難した際には一早く駆けつけてくれたりする.漁業という仕事における危険に対応する上で漁師同士の関係は必須なのである.したがって,漁師はその関係を保持し続けるために,全てが自由にできるわけではないのである. 4 考察  以上を踏まえると,海および漁師との関係を築くことが「漁師になる」ことだとわかる.漁師になるとは,自然との関係・社会関係のなかに再び取り込まれることを指す.注目したい点は,その関係には分断が見られないことである.賠償金や原発からの距離をめぐる軋轢は例外なく本地区にも存在する.しかし,漁師になった彼らからは,その社会的分断が見られない.すなわち,漁師になることの潜在的な意味に脱原発被災者もあげられる.これは原発事故後の社会的分断について考えていく上で重要な事柄になると考えられる.

報告番号42

津波被災地の復興過程における住民自治の展開(1) ――石巻市北上町ウィアーワン北上の10年と復興政策への応答
早稲田大学 西城戸誠

【目的】本報告では、東日本大震災後に結成された宮城県石巻市北上町における地域づくり団体「ウィアーワン北上(以下「W」)」の活動の変遷をたどり、活動内容の質的な変化を確認する。そして「W」が復興支援員、地域おこし協力隊といった「地域サポート人材」制度、「小規模多機能自治」「小さな拠点」づくりといった住民自治に関連した政策にどのように応答したのか分析し、被災地における住民自治施策の課題について考察する。 【方法】報告者らは東日本大震災後から北上町へのアクションリサーチを実施してきた。特に「W」が北上町における「住民自治」のための活動を本格化する際には、他の被災地や「小規模多機能自治」の先進事例の視察をともに行ってきた。本報告では「W」への参与観察および定期的な聞き取り調査の結果、発刊された記録をデータとして用いる。 【結果・結論】2011年6月に設立された「W」は、震災当初は買い物が不自由な地域住民が住む仮設住宅近くに店舗を作るといった「生活支援」の活動を主に行っていた。その後、国が被災地のコミュニティの再構築を図る目的で「復興支援員」を制度化(宮城県では「復興応援隊」)したが、「W」は復興応援隊として、高台集団移転の合意形成のサポートや石巻市が主導する「北上まちづくり委員会」の内容を「北上かわらばん」として発行、情報発信を担った。だが、同委員会で若い世代の意見が反映されていないと判断した「W」が、震災後に取り組んだ地元の海水浴場の再開に向けた活動を通じて知り合った若者が「北上インボルブ」というグループを作り(事務局は「W」)、「北上をおもしろくする提案書」という未来志向の復興計画を立案し、地域のイベントも行うようになった。さらに2018年からは地域支援員によるコミュニティ・ナース事業(第二報告)を開始した。このように「W」は「生活支援」、「コミュニティ支援」、地域の特徴を活かした「価値創造活動」へと展開したが、これらは中越地震発生後に導入された地域サポート人材の事業と共通している。  他方で「W」は「小規模多機能自治」「小さな拠点」といった国による地域活性化の施策について先進事例を学び、北上町への応用を図った。震災前から企図されていたが震災の影響で止まっていた、地域自治組織が地域自治を推進するための仕組みとされた「石巻市地域自治システム」が制度化され、2020年春に「みらいパートナーズ・きたかみ」が発足、「W」はそのサポート事務局を担った。だが、「小規模多機能自治」などによって紹介される地域課題の解決実践事例(買い物支援など)が必ずしも北上町にはフィットしないこと、石巻市の縦割り行政や予算措置の問題で「石巻市地域自治システム」が小規模多機能自治の先進事例のようには機能しないと「W」は判断しており、2021年春現在「みらいパートナーズ・きたかみ」の活動は棚上げ状態となっている。つまり、住民自治を目指すためのしくみを機能させるためには行政が民間団体をどう位置づけるかにかかっており、これは第2報告のコミュニティ・ナース事業にも共通している。また、震災復興・地域再生に関わる「ポンチ絵的なマニュアル」を批判的に検討し、順応的な対応をする「W」のような民間団体が地域再生には重要であり、先進事例の模倣ではなく、現場で実験し、適応しなければ無理に実施しないという制度への応答の態度が鍵となるだろう。

報告番号43

津波被災地の復興過程における住民自治の展開(2)――コミュニティ・ナース事業の活動の意味
岩手県立大学 庄司知恵子

【目的】本報告では、宮城県石巻市北上町の地域づくり団体「ウィーアーワン北上(以下「W」)」が、2018年から行った「コミュニティ・ナース(以下、「CN」)事業」の活動内容について確認する。そこから、被災地における地域づくり団体の活動の展開における、CNの存在意味を検討し、復興と住民自治における問題点について考察する。 【方法】報告者らは、震災後、北上町において実践的調査研究を行い、地域の様々なアクターとかかわりながら、北上町の復興過程をみてきた。そのアクターの一つがWである(詳細は第一報告)。本報告は、Wへの定期的な聞き取り調査とCN事業についての聞き取り調査で得たデータに基づき行う。 【結果】代表のSN氏は、活動する中で、北上での生活を継続する高齢者の存在から、「地域で健康に生活をする」ことへの支援が必要だと考えた。そこで、SN氏は大学で開講されたCSWスキルアッププログラムを受講したが、活動に結びつく視点は得られなかった。そのような中、地域自治組織の視察で訪れた島根県雲南市で、「コミュニティナーシング」にもとづいて地域づくりをするナースに出会った。その活動からヒントを得たSN氏は、2018年から3か年の予定で、北上町にてCN事業を行うことにし、大分県出身の看護師HA氏が着任した。 1年目は、地域づくりと看護の専門性との接続が見えずに悩んだHA氏だったが、2年目からは積極的に地域とかかわるようになった。その中で、これまでWが繋がれずにいた社会福祉協議会(以下、「社協」)と共同で集落ごとのお茶のみを開催した。HA氏が看護師であることについて、SN氏は「共通言語があることは強い」と話す。また、社協も地域共生型社会の実現に向け、Wとの接続を模索していたが「Wは産業振興の団体である」ことを理由に繋がれずにおり、HA氏を通し「福祉」の接点を持つことができた。 HA氏が、積極的に取り組んだ作業に「傾聴」がある。彼女が出会った住民は200人を超え、そのデータは「マップ」としてまとめられている。SN氏は、「森を見て木を見てこなかった」とCN事業前の活動について振り返り、HA氏が住民一人ひとりと向き合ったことで、次なる住民自治の展開への繋がりも見えてきたと話す。HA氏は、活動を通して終末期の支援がしたいと考え、任期半年を残し石巻市の訪問看護ステーションへ転職をしたが、それはCN活動の延長線上にあり、今後も北上町とかかわりっていく。 【結論】 被災地では、集落移転も終了し、地域づくり団体の活動は、自身の組織化やイベント重視の段階を過ぎ、「個人の支援」を組み込んだ地域づくりの段階に入っている。そこで求められる支援は、福祉領域に見られるニーズありきのものではなく、また死の地域化を目指した医療ベースのものでもない。平たく言えば、「そこで健康に生きていく」ことの支援である。Wがそのような活動を進めようとしたとき、保健師や社協職員とは繋がれずにいた。それは、行政や社協が「縦割り」であること、計画性重視であることに起因する。対して、右往左往しながら活動の在り方を模索できるWの活動は、住民のニーズにフィットするが、目に見える成果を収められるわけではない。大枠の計画の中で「回遊」するような民間の活動の在り方を、どこが支えていくのか。計画性重視の行政が、彼・彼女らをどう位置付けていくのか。地域の持続可能性の点からも検討が必要だろう。

報告番号44

大規模災害発生のリスク認知と原子力政策に対する態度の変化――JGSS-2008/2010/2012/2015/2018/2021の比較を通して
大阪商業大学 宍戸邦章

1 目的  日本版総合的社会調査(JGSS)では、再生可能エネルギーの利用・節電行動については2008年以降、環境汚染のリスク認知は2010年以降、大規模災害発生のリスク認知、災害時の地域の人々の協力、原子力発電所の事故への不安感、原子力政策の今後については2012年以降継続的に尋ねている。2021年1~2月に実施したJGSS-2021では、さらに、被災経験・時期・程度、避難場所、日頃の避難準備、復興政策の定期的な見直しの必要性についても尋ねている。 本報告では、災害リスク認知や原子力政策への態度について、JGSSを含む各種世論調査における回答の分布の変化を追う。また、JGSSのデータを基に、原発政策への態度に影響を与える要因、大規模災害発生のリスク認知や被災経験と日頃の被災準備との関係を考察する。 2 方法  資料としては、内閣府、新聞社、NHK、国立環境研究所、日本原子力学会、原子力文化振興財団などが実施した世論調査と報告者らが実施してきたJGSS-2008/2010/2012/2015/2018/2021の個票データを用いる。 3 結果(JGSS-2018まで、報告では、JGSS-2021の結果も加える) ① 東日本大震災後、各種の災害リスク認知は横ばいか低下傾向。熊本地震や地震予知報道、豪雨の影響などで、一部の地域では災害リスク認知が増加。 ② 2011年以降、大気、水質、土壌に対する汚染意識は低下傾向だが、元の水準には戻らず。とくに放射能による食品汚染意識は子育て中の家族を中心に残存。 ③ 福島第一原発からの地理的距離と環境汚染意識の関連は消滅。 ④ 各種世論調査によると、原発について「現状維持」が2割、「減らす」が7割。JGSS-2012と2018を比較すると、関東、中部、近畿、九州で、現状維持志向が増加。 ⑤ 原発から地理的に離れるほど、原発事故のリスク認知は低下傾向。原発近距離住民と遠距離住民のリスク認知ギャップが拡大。最寄の原発から70~80km圏内でリスク認知が高まる。 ⑥ 原発からの距離はリスク認知と強く関連するが、原発政策への態度には関連せず。受苦圏(原発立地周辺地域)と受益圏(原発から離れた電力消費地・都市部)といった単純な関係ではない。 ⑦原発からの距離と地震リスク認知の交互作用については、電源地域と消費地域によって原発政策への態度に与える影響要因が異なる。 ⑧汚染意識、リスク認知、原発政策のいずれにおいても、支持政党の影響が認められる。自民党支持層で、汚染意識やリスク認知が低く、原発縮小政策に反対する傾向。 [文献] 岩井・宍戸, 2013, 「東日本大震災・福島第一原子力発電所の事故が災害リスクの認知および原子力政策への態度に与えた影響」『社会学評論』64(3)420-438. 宍戸・岩井, 2019,「大規模自然災害・原発災害発生のリスク認知と原子力政策・エネルギー利用に関する意識の推移」第5回震災問題研究交流会 [Acknowledgment] 日本版 General Social Surveys(JGSS)は、大阪商業大学 JGSS 研究センター(文部科学大臣認定日本版総合的社会調査共同研究拠点)が、大阪商業大学の支援を得て実施している研究プロジェクトである。JGSS-2021 は、文部科学省特色ある共同研究拠点の整備の推進事業 JPMXP0620335833、JSPS 科研費 JP20H00089 の助成を受けて実施した。データの整備は、JSPS人文学・社会科学データインフラストラクチャー構築推進事業JPJS00218077184の支援を得た。

報告番号45

東日本大震災被災世帯による住宅再建への調整行動――岩手県釜石市A地区世帯への質的縦断調査より
東洋大学 西野淑美

【1.目的】 本研究の目的は、津波被災時に地理的に同一条件にあった住民がどのように異なる生活再建行動をとるのかを明らかにすることである。隣り合った世帯であっても、元の地区での自宅再建、他の地区での自宅再建、災害復興公営住宅への入居など、住宅再建の場所も形態も時期も異なる。本報告では、再建の「選択」の背景にあった各世帯の調整行動について、描写と解釈を試みる。 【2.方法】 本グループは、岩手県釜石市A町内会の東日本大震災時会員約220世帯のうち40数世帯に2012-2019年に原則毎年、計8回の聞き取り調査を行い、現在も質的な縦断調査を継続中である(東京大学社会科学研究所他編 2020)。また、2016年には聞き取り調査の対象以外の世帯に質問紙調査を行った。本報告では、縦断調査対象のうち42世帯と質問紙調査の有効回答57世帯の計99世帯の2016年までのデータと、2016年以降の聞き取りの情報を用いる。A地区は過半数の世帯が全壊・大規模半壊の被害を受けたが、町内に高低差があり、半壊、一部損壊、被害無しの住宅も存在した。また、震災前のA地区の半数以上の住宅があった範囲では盛土等を行う土地区画整理事業が実施されたが、事業区域に含まれない範囲も町内には存在する。そのため様々な条件の世帯間の比較が可能である。 【3.結果】 A地区の土地区画整理事業の換地引き渡し完了は2020年4月で、震災発生の2011年3月から9年が経っていた。調査対象世帯で完全流失・修理不能の被害を受けたが自宅を再建したケースのうち、まず区画整理の事業区域外に自宅があったケースは、盛土工事を待つ必要がないため、2013-2016年頃に自宅再建が完了した。事業区域内に自宅があったが、事業区域外に土地を得て再建したケースも、ほぼ同時期に再建が済んだ。しかし、事業完了を待って換地先に自宅再建をしたケースは、早くて2017年以降の再建となった。自宅再建を断念し復興公営住宅に入居したケースは、入居先の完成時期により2013年から2017年の入居となった。 【4.結論】 こうした再建形態の相違は、一見すると住宅再建資金の調達力で決まったようにみえる。A地区外に「家+土地」を確保する資金を調達できれば事業を待たずに早期再建が可能になる。「家」のみの資金しか確保できなければ事業完了を長く待って換地先に再建する。資金の確保が困難ならば復興公営住宅に入居する。大まかにはこのように分類できよう。しかし、詳しく分析すると、このような説明の整合性は結果的にとれたものと考えられる。住宅再建の資金+αの資源を多様な工夫により調達する行動が観察されるのである。例えば、震災前は別居だった親世帯と成人子世帯が二世帯の住宅再建支援金を持ち寄って「家+土地」を確保する、区域外の相続地を活用する、伝手を駆使して売地の情報を探すなどである。ライフコースの展望、世代間の住宅継承、地域社会の中での役割等が、行動に影響したと考えられる。もっとも、被災世帯の能動性だけを強調することも偏りがある。実際は復興事業の進捗や家族との綱引きのなかで、諦めながら選択肢が狭まっていった。制約のなかで、できるだけベターな選択だったと言えるようにするために各家族が様々な工夫を行った結果、制度に沿って「選択」が成立したように見えていると解釈しえよう。 文献 東京大学社会科学研究所他編, 2020,『地域の危機・釜石の対応』東京大学出版会.

報告番号46

日本若年層の「スマホゲーム」頻度にたいする、遺伝子一塩基多型rs4680 の看過しがたい効果――遺伝子社会学の試み:そのN
鹿児島大学 桜井芳生

1 目的  現代日本の青少年におけるいわゆるスマホゲームの遊戯頻度に、ある一つの遺伝子変数が看過しがたい影響をもっていることをみいだした(桜井芳生他)。また遺伝子変数を視野にいれた社会科学的リサーチの普及への一歩として、報告したい。 2 方法  以下の実験計画は大学倫理審査委員会の審査済みである。匿名の調査協力者さんたちから採集した遺伝子試料の解析結果と、その方々に回答いただいたスマホアンケートの結果との統計解析をおこなった。DNA採取キット Oragene® DNAをもちい、唾液ないし頬内側粘膜をいただいた。A大学とB大学、2018-19年度のいくつかの授業履修者さんたちが対象プール、スマホアンケートと遺伝子試料ご提供協力者161人。男性102人。女性59人。年齢平均19.17歳。リアルタイムPCR装置(StepOnePlus)で解析を行った  まず、以下要点となる遺伝子一塩基多型(SNP)rs4680について説明する。周知のようにDNAは、ねじれた縄ばしごが、父由来と母由来の二組ある形状をしている。この縄ばしごの一段に変異した多様性が見られる場合がある。この変異が集団内で1%以上の頻度で見られるとき、これを一塩基多型SNP(スニップ) : Single Nucleotide Polymorphismとよばれる。よく知られているように単位をなす塩基は、アデニン(A)、グアニン(G)、シトシン(C)、チミン(T)の四種類だけであるが、「アデニン(A)⇔チミン(T)」「グアニン(G)⇔シトシン(C)」という組み合わせでのみ相補対をなす。したがって、ターゲットとする座位が、「G」か「A」かのいずれかであるかがわかればその座位の値は完全にわかることとなる。ただし父由来と母由来の都合二つの値がありうるので、ある座位がもちうる値はG G(ホモ)、A G (ヘテロ)、A A (ホモ)の三通りとなる。当該の「rs4680」においては、G Gホモが、ワイルド=野生型(もともとの型)とされ、AはGからのミュータント(変異型)とされる。上記のようにGGが野生型であり、AキャリアであるAGとAA は、変異型とされるため、「Aキャリア」である(すなわち、AGとAA)か、いなかで、「rs4680Aキャリアダミー」という変数を作成した。 3 結果  重回帰分析の結果「GGタイプの人」ほど、「スマホゲームをし」、「外向性が高く(活発で、おとなしくなく)」、「協調性が低く」、「技能向上や新技術習得のための訓練の機会が多いことを重視する」となった。  4 結論  今回のリサーチは、新コロナ禍のためもあって、サンプル数が十二分とはいえない。広く、追試を呼びかけたい。GGタイプのヒトすなわち、23andMeにいうwarrior 勇士 型のヒトが、有意に、スマホゲームの頻度が高い、というのは絵に描いたような結果でわれわれ自身おどろいている。 いずれにせよ、今回われわれがみいだいしたことは、スマホゲームをめぐる社会(科)学的アプローチに一石を投じるものであることは否定し難いのでなないだろうか。 (1).登壇者、コレスポンディング・オーサー。yoshiosakuraig@gmail.com 参照 SNPedia:rs4680 https://www.snpedia.com/index.php/Rs4680 2020年8月12日閲覧 科研費挑戦的(開拓)20K20281「文化-ジーン共進化説のミクロ的確認とネットワーク社会学的展開」(代表:桜井芳生)による。 桜井芳生他2021『遺伝子社会学の試み 社会学的生物学嫌い(バイオフォビア)を超えて』日本評論社

報告番号47

大学教員や文系知識人は「革新」的か?――科学の政治化と職業による保革自己認知の違い
京都大学 太郎丸博

1 目的 大学教員や関連の職についている人が他の職業の人よりも革新的か明らかにする。このような分析を行う背景には、日本における科学の政治化がある。科学の政治化とは特定の政治的党派と科学が対立することをいう。科学の政治化は米国で1980年代頃から起きたと言われているが、日本でも自民党政権による南京大虐殺の忘却、安保関連法案審議時の違憲解釈の無視、日本学術会議会員の不承認、といった具合に、保守政治家と主に文系の研究者との対立が目につく。こういった状況の一因として、大学教員や知識人と呼ばれるような職業の人が革新的であるから、保守派と対立する、という指摘がある (N. Gross, 2013. Why Are Professors Liberal and Why Do Conservatives Care? Harvard University Press)。日本でも同じことが言えるのか検討する。 2 方法 Japanese General Social Surveys (JGSS) – 大阪商業大学 JGSS研究センター 2000-2012年をマージしたデータを用いる。二次分析に当たり、東京大学社会科学研究所附属社会調査・データアーカイブ研究センターSSJデータアーカイブから上記調査の個票データの提供を受けた。20~89歳の男女を対象とし、リストワイズで欠損値を含むケースは取り除いた。有効サンプルサイズは 20,080である。従属変数は保革自己認知で「政治的な考え方を、保守的から革新的まての5段階にわけるとしたら、あなたはとれにあてはまりますか」という質問に対する回答で、選択肢は5択で1~5の値をわりふった(数値が大きいほど革新的)。主な独立変数は職業で、「大学教員」、「文系研究者」、「理系研究者」、「小、中、高教員」、「記者・作家」、「その他の職業」の6カテゴリである。調査年、性別、年齢、学歴、従業上の地位、世帯収入も統制変数としてモデルに投入し、OLS で推定した。 3 結果 大学教員、文系研究者、記者作家はその他の職業よりも有意に革新的であることがわかった。学歴は効果がなく、年齢と収入は二乗の効果が有意であった。具体的には年齢は40歳ぐらいで最も革新的でそれより若い、あるいは高齢であるほど保守的であり、世帯収入800万円程度で最も革新的、それより収入が低い、あるいは高くなるほど保守的になる傾向が見られた。また無職の方が有職より保守的であったが、正規雇用、非正規雇用、自営のあいだに有意な違いは見られなかった。 4 結論 米国と類似の結果が得られたが、理系の場合はあてはまらない可能性も示唆された。

報告番号48

台湾ひまわり運動・香港雨傘運動における「対話」と「情動」
日本大学 陳怡禎

【1.目的】 本研究では、2014年に起きた台湾のひまわり運動や香港の雨傘運動という2つの社会運動に注目し、それらの社会運動の担い手である若者は、いかに「内向きの語り」を実践し、さらにその「語り」を通して、参加者の間の「対話」を実現し、「情動」を生成するかを考察する。 【2.方法】 まず、本研究は、台湾の学術研究機関「中央研究院」によって保存されるひまわり運動創作物アーカイブ『318公民運動文物紀錄典藏庫(318 Civil Movement Archive)』や、香港雨傘運動参加者有志が集めた現場の記録写真をデータとして、運動当時の風景を再現する試みを行う。中では特にそれらの社会運動空間の出現した二つの「対話」的空間に交わされた運動参加者の語りに焦点を当て、運動参加者はどのように社会運動に対する語りを交換していたか、さらにどのような情動を生成したのかを分析する。 【3.結果】 本研究は、以下のような調査結果を示す。 ひまわり運動や雨傘運動参加者の間に、多くの「感謝」や「激励」の言葉が交わされていたことが確認できる。それは、それら二つの運動の「ケア」や「気遣い」といった特質や、参加者同士の相互的視線から生じたものだと考えられる。 また、それらの運動の参加者による語りの中では、「台湾」や「香港」という言葉が多用されていることも確認できる。参加者たちは、「台湾」または「香港」について語ることを通して、台湾または香港に対する愛着を参加者の間で共有し、情動を生成したうえで、運動参加者、さらにいえば台湾人または香港人としての民族的アイデンティティや連帯感を構築していたと推察できる。 【4.結論】 本研究は、台湾のひまわり運動や香港の雨傘運動の参加者は、いかに社会運動空間において、さまざまな内向的語りを生み出し、さらにその語りを交換し続けることによって、「情動」を生成したかについて考察した。このような情動は、台湾や香港の社会運動参加者の社会運動に深く参入することを促し、長期化した社会運動のエネルギーを維持したと考えられる。中でも、運動の担い手である台湾や香港の若者たちは、相互的目線や語りを交換することを通して、「運動参加者」という連帯感を構築したうえで、社会運動の目的より「台湾」や「香港」への愛着を強調し、「台湾人」や「香港人」というアイデンティティを確かめていたと考えられる。このような「運動参加者」、さらに台湾、または香港「民族」の二重的アイデンティティの構築によって、台湾や香港の若者たちは、それらの社会運動に意味を付与し続けていると言えるだろう。

報告番号49

「小資」的雑誌のメディア機能――『上海壹週』の分析を中心に
京都大学大学院 呉江城

本稿は中国の都市新中間層の表象文化の一つである「小資」文化に着目するものである。「小資」文化の生産体制の中心部にあるマスメディアはどのような機能を発揮しているかという問いに立ち向かって、「小資」文化の文化的オムニボアの特質をメディア文化論の視点から究明し、都市新中間層の文化形成に与える影響を検討する。先行研究(呉 2020)では、「小資」文化の文化的オムニボアの特質を1980年代のエリート大学生世代の文化的教養習得という経路に従って考察した。本稿はその結論の延長線にある1990年代のメディア文化人を研究射程に加わる。市場化されたメディア業界で活躍した記者・雑誌編集者・メディア経営者といったメディア文化人は実際に1980年代のエリート大学生世代でもあった。こうした動きを背景として、本稿は最も「小資」的だと呼ばれた『上海壹週』の分析を中心に、2001年-2004年の間の「小資」的雑誌のメディア機能を明らかにする。  「小資」的雑誌のメディア機能を総合的に考察するため、送り手、誌面内容、受け手に対応する多様な研究方法を採用する。まず、送り手研究に関して、『上海壹週』の出版元ある上海文芸出版総社の出版文化、『上海壹週』の経営者・編集者である陳保平と徐沪生の理想と実践を、調査報告書、自伝などの文献資料によって整理する。次に、雑誌内容に関して、『上海壹週』の誌面構成に対する誌面分析、「読書」・「音楽」・「映画」というライフスタイル情報のコラムに対する量的・質的内容分析を行う。さらに、読者に関しては、読者投稿欄の記事を分析する。 結果として、『上海壹週』は上海文芸出版総社の「文化重視」という出版文化から強く影響を受け、エリート大学生世代に属する陳保平と徐沪生の文化的理想と文化的実践によって「小資」文化の重鎮を成していることがわかった。また、誌面分析からは『上海壹週』の「小市民文化」と「機関紙文化」の間にある位置付け、内容分析からは「小資」文化における外来文化、アカデミック文化、若者文化の混合が明らかになった。それは1980年代のエリート大学生の文化的教養と類似する文化的オムニボアの傾向をみせている。最後に、読者の投稿欄分析によって、読者も『上海壹週』の「小資」文化を都市新中間層の階層文化として受け止めていることが究明された。  以上の結果を踏まえて、本稿の結論は以下のようになっている。「小資」的雑誌メディアは、一つの文化装置(Mills、1963= 1971、323)として、①新たな「小資」文化の生産、②「小資」文化の「中間性」の規定、③「小資」文化の生産体制の自律性の維持という三つの機能を果たしている。「小資」文化の文化的オムニボアは、実は文化装置がもたらした結果だとも考えられる。 参考文献 1呉江城, 2020,「エリート大学生の文化的教養習得に対する歴史的考察:1980年代の「文化熱の分析を手がかりに」 『京都大学教育学研究科紀要』66:233-246. 2Mills, C.W. 1963 Power, People, Politics, ed. by I.L. Horowitz, Oxford University Press (= 1971,青井和夫・本間康平監訳『権力・政治・民衆』みすず書房).

報告番号50

英語支配の何が問題か――メディアとしての英語
東京農工大学 岡野一郎

グローバル化の時代において英語という外国語は重要さを増しているが,英語支配はさまざまな議論を呼び起こしている。しかし英語支配を批判する立場の間にも対立があり,明快な見取り図が見えず,多言語多文化を尊重すべしという誰もが同意するような主張と,英語の一元的支配の進行とが,ばらばらに併存しているのが現状である。そして同じ英語支配への批判派の間でも,英語支配を言語そのものの問題として見るか,社会的関係の反映として見るか等,論者によってかなり見解が異なっている。結局,英語支配の何が問題なのかが,未だに不明確なのである。  特に気を付けなければならないのは,グローバル化のとらえ方も多様であることだ。1990年代以降の変動が目を引くが,グローバル化自体は数世紀のスパンで生じている現象である。現在の英語による支配と,半世紀前のそれとでは,その位置づけはかなり違っていてもおかしくない。  そこで本報告では,故ウォーラースティーンが導入した「ヨーロッパ的普遍主義」の議論に依拠し,英語を近代世界システム及びそのイデオロギーとの関係で捉えることを試みる。ウォーラースティーンによれば,近代世界システムの展開の中で,それを正当化するイデオロギーは,「野蛮に対する普遍的価値(文明/未開)」「本質主義的個別主義(オリエンタリズム)」そして「科学的普遍主義」へと,重なり合いつつ変化してきた。  これを英語支配にあてはめるならば,オリエンタリズムはコロニアリズムを背景として,言語を本質主義的に序列化する。それに対して「科学」と「人文学」が差別化される科学的普遍主義においては,英語は科学の言語として特権化される。この観点からすると,英語は現在,もはやコロニアリズムを離れ,新自由主義的経済秩序とますます結びついている。いずれにしても背後にあるのは資本主義世界経済の論理であり,英語はそのようなイデオロギーを担ったメディアとして流通している。  これを日本の英語教育をめぐる状況に当てはめてみると,日本における英語教育をめぐる言説も,歴史的に変化してきた。戦前の内村鑑三以来の教養主義的英語教育観に対して,戦後の英語教育を導いたのは異文化コミュニケーションなどの国際理解教育であり,これは「文明の衝突」に代表されるオリエンタリズムの時代に該当する。それに対して1990年代以降のいわゆるグローバル化の時代において,英語はますます文化的要素を脱色され,人文学ではなく科学の言語となっていくのである。 したがって問題なのは,英語が国際語かどうかということではなく,何のための国際語かということであろう。新自由主義的国際秩序を乗り越えるために必要なのは,英語を市場と科学の結合から解き放つことであり,グローバルな平和運動,反差別,反核など,反システム運動との連帯のためのメディアとして英語を位置づけていくことである。

報告番号51

犯罪被害者を本格派推理小説の主題として描く方法――東野圭吾と貫井徳郎の作家活動に注目して
東京家政学院大学 岡村逸郎

1 目的  本報告の目的は,東野圭吾と貫井徳郎が犯罪被害者を本格派推理小説の主題として描く方法をいかにして形成したのかを,彼らが作品の執筆を通して行なった作家活動に注目して明らかにすることである. 2 方法  収集した資料は,東野が1985年から2009年にかけての時期に刊行した小説およびエッセイと,貫井が1993年から2009年にかけての時期に刊行した小説およびエッセイである.それらのなかから,犯罪の被害者ないしその遺族が物語の中心に位置づけられた作品を選び,内容の分析を行なった. 3 結果  東野と貫井は,本格派推理小説をあらたなかたちで展開する活動を新本格運動の影響のもとで行なった点と,犯罪被害者を本格派推理小説の枠組みのなかで描く方法を模索した点において,問題関心を共有した.しかし,犯罪被害者を活用した方法については,方向性を異にした.  東野は,「リアル」な人間描写を本格派推理小説の枠組みのなかで行なうことを課題とした.さらに,被害者遺族を探偵役にするとともに,被害者の内面に関する叙述トリックを用いることによって「涙」の結末を演出する方法を提示した(『秘密』).そして,加害者/被害者に関する描写を操作することによって人々の共感の対象が容易に移ろうことを,個々の作品の執筆と読者の反応の観察を通して主題化した(『手紙』『さまよう刃』『容疑者Xの献身』).  貫井は,本格派推理小説の物語を駆動するあらたな仕かけとして犯罪被害者に注目し,それを物語のプロットの核心におく方法を模索した.貫井は,被害者を叙述トリックの核心として用いる方法(『慟哭』『被害者は誰?』)や,謎の多重解決を被害者の内面に関する解釈の多面性の観点から描写する方法(『プリズム』)を提示した.そして,人々の日常生活におけるさまざまな些細な悪意を1つの事件が起きる伏線として描くことを通して,遺族の経験を本格派推理小説の枠組みのなかで描写する方法を提示した(『乱反射』). 4 結論  探偵小説や推理小説においては,物語の冒頭で誰かが死ぬことが求められ,被害者の死が物語を駆動するトリガーとして位置づけられてきた.こうした物語の構造上の制約があるにもかかわらず,なぜ推理小説のなかでもとくに形式の順守に重きをおく本格派推理小説を中心として,犯罪被害者を主題とする作品が書かれたのだろうか.この問いには,犯罪被害者に対する社会的な注目の高まりという,ジャンル外の要因のみによっては答えることができない.犯罪被害者を主題とする本格派推理小説の謎は,本報告で試みたように,複数の作品の執筆を通して犯罪被害者を描く方法を模索した作家の諸実践を分析することによって,解明することができる. [文献] 笠井潔,2011,『探偵小説と叙述トリック』東京創元社. Sapiro, Gisèle, 2014, La sociologie de la littérature, Paris: Éditions La Découverte.(鈴木智之・松下優一訳,2017,『文学社会学とはなにか』世界思想社.) 内田隆三,[2001] 2011,『探偵小説の社会学』岩波書店.

報告番号52

新型コロナ禍におけるイベントスペース利用者調査から(1)――ジャンルによる「自粛」の違いと代替スペースの分析
明星大学 小股遼

1.目的  本報告の目的は、新型コロナウィルス感染症の流行前後で、大都市における文化空間の利用が、施設やジャンルごとにどのように変化したかを明らかにすることである。  コロナ禍によって、対面が自律・他律に関わらず「自粛」され、集積・集合のための物理空間に再考を迫る状況が到来した。サードプレイス論における第1から第3の場の役割が、在宅や巣篭りの形で一箇所に集約された新しい生活様式が推奨された。注目すべきは、コロナ禍以前からの積極的なスペース利用者にとって、それがどのように受け入れられたのか否かである。本報告では、利用者の動向から、コロナ禍での「自粛」の影響が施設やジャンルに対しどのような示差的影響を及ぼしたのか、どのような場所が代替的に活用されたのかを明らかにする。このことを通じ、ポストコロナを含む今後の大都市の空間再編を検討する作業につなげたい。 2.方法  2021年3月に報告者らはオンラインアンケート調査「新型コロナ禍におけるイベントスペース利用者調査」(n=1244)を実施した。調査は、2019年以降、各種イベント(劇場・小劇場、コンサート・音楽ホール、クラブ・ライブハウス、ボランティア・講演会・シンポジウム、同人誌即売会・MAGイベント)への参加経験者を調査対象者(首都圏在住)とし、コロナ禍以前・以後でのイベントへの参加状況や、「自粛」下でのイベントへの対応などについて尋ねた。 3.結果・結論  全ての施設やイベントにおいてコロナ禍以前と以後では参加率が大きく異なり、多くは7割近く落ち込んだ。音楽系で利用者数最大 のコンサートホール・音楽ホール(n=615)では、コロナ禍以前の531(86.3%)から一回目の宣言中の64(10.4%)へと参加が落ち込み、宣言解除期も180(29.3%)と回復は限定的だった。他の屋内系の施設やイベントでも似た傾向がみられた(美術館等(n=574)は、コロナ禍以前504(87.8%)一回目の宣言中84(14.6%)、宣言解除期225(39.2%))。  他方で、広場・公園(n=650)は、コロナ禍以前576(88.6%)、一回目の宣言中371(57.1%)、宣言解除期440(67.7%)と推移し、落ち込みが少ない(3割程)。「自粛」下において買い物を除く数少ない外出先として機能していた。宣言解除期における利用者回復は1割程で、再開した施設やスペースへの人の戻り方と比べると相対的に低い。ここから公園が、「危機」時におけるレジリエンスを特徴とする代替空間としてあったことが浮かび上がった。  オンラインイベントへの参加度数は、美術館等では時期を問わずリアルな場所への参加度数の約半分であるのに対し、講演会等(n=210)ではそれが約3倍であった。アートでは生の臨場性に価値がおかれる一方、知識や情報の伝達が主体の講演会等ではオンラインによる代替ないし新規参加者の掘り起こしが起きた可能性が推測される。  コロナ禍によって都市の文化空間は大きな痛手を受けた。しかし、「自粛」要請を臨機応変に解釈する人びとによって、屋内・屋外の公共空間が特性に応じて使い分けられていた。またオンラインの利用分析からは、対面性や臨場性へのこだわりや代替可能性の違いに応じて、施設やジャンルごとに実験的状況が展開したことが明らかとなった。  本報告は科研費基盤研究(B)「『高さ』を疑う、『高さ』を背負う: 新しい都市ガバナンスの社会学」(19H01557)の成果の一部である。

報告番号53

新型コロナ禍におけるイベントスペース利用者調査から(2)――クラブ・ライブハウス利用者の支援行動とスペースの存続
一橋大学大学院 山内智瑛

1.目的 本報告の目的は、新型コロナウイルス感染拡大(以下、コロナ禍)におけるクラブ・ライブハウス利用者の支援行動にいかなる傾向が見られるのかを明らかにすることである。日本では「クラブやライブハウスといった音楽・ダンスの場、およびバー等の酒類を提供する娯楽場」(池田ほか 2017)であるナイトライフは長らく取締りの対象であったが、2015年の風営法改正以降、都市経済成長のための「夜間経済(night-time economy)」へと転換されつつあった(山内 2020)。しかしCOVID-19流行に伴う政府・地方行政からの営業自粛要請の結果、多くの店舗が閉店・休業・時短営業を余儀なくされ、再び存続の危機を迎えている。こうした中、ナイトライフ側は公的補償を求めつつ(e.g.「#SaveOurSpace」)、有/無観客イベント配信の収益化、クラウドファンディング、グッズ販売といった手段を用いて生き残りの道を模索している。本報告では、報告者らが実施したアンケート調査のデータにおけるクラブ・ライブハウス利用者の支援行動に焦点を当てながら、コロナ禍における「自助」や「共助」のあり様を検討する。 2.方法 報告者ら調査グループは、2021年3月にオンラインアンケート調査「新型コロナ禍におけるイベントスペース利用者調査」(n=1244)を実施した。その中で、本報告は2019年以降にクラブ・ライブハウスを訪れた人およびクラブ・ライブイベントに参加した人を「クラブ・ライブハウス利用者」(n=341)として分析の対象とし、オンライン配信視聴やクラウドファンディング参加の有無、コロナ禍でのスペース継続参加等の分析を行った。以上の調査は、科研費基盤研究(B)「『高さ』を疑う、『高さ』を背負う――新しい都市ガバナンスの社会学」(19H01557)の一部である。 3.結果・結論 クラブ・ライブハウスへの支援行動の有無には「ハコへの愛着」が大きく関与していた。報告者は2019年以降の施設利用・イベント参加の目的を尋ねた設問(選択肢:「特定の場所が個人的に好きで定期的に通っている」、「所属している組織や団体、サークルで特定の場所によくお世話になっている」、「イベント・出演者がおもな目的なので、特定の場所にこだわりはない」)を用いて「ハコ愛着スコア」を作成し、支援行動との関連を分析した。その結果、スコアが高い利用者ほどオンライン配信の視聴やクラウドファンディングへの参加、そしてコロナ禍でのスペース継続利用が多いことがわかった。クラブ・ライブハウスのジャンルには、ハコのその場の空気を売りにするもの(e.g.クラブ、ジャズ)とアーティストのパフォーマンスを売りにするもの(e.g.ロック)という2種類が存在するが、上記の結果は後者のタイプの店舗が支援を受けづらいことを示唆する。コロナ禍の「自助」・「共助」は、イベントスペースの維持をめぐる新たな「協働体」の生成という徴候を見せつつ、その内外で格差を助長するというアンビバレントさを抱えている。 文献 池田真利子・卯田卓矢・磯野巧・杉本興運・太田慧・小池拓矢・飯塚遼, 2017, 「東京におけるナイトライフ観光の特性――夜間音楽観光資源としてのクラブ・ライブハウスに着目して」『地理空間』10(3): 149-64. 山内智瑛, 2020, 「ナイトライフと『安全・安心なまちづくり』政策――大阪・アメリカ村におけるクラブ摘発問題の事例から」『年報社会学論集』33: 180-91.

報告番号54

新型コロナ禍におけるイベントスペース利用者調査から(3)――参加者の行動変容から再考するオタク文化と対面イベントの関係性
慶應義塾大学大学院 杉山怜美

1. 目的 本報告は,同人誌即売会を主要な事例として,新型コロナウイルス感染拡大(以下,コロナ禍)前後でのアニメ・マンガ・ゲーム等(以下,MAG)のオタク文化における対面イベントに対する参加者の行動変容を明らかにする.  2000年代以降,日本ではオタク文化の市場面への社会的認知が高まった(野村総合研究所 2005).しかし、そうした文化に親しむ人びとにとっては、メディアコンテンツの個人的消費だけでなく,コミックマーケットなどの同人誌即売会への参加もまた,数十年の歴史を持つ重要な活動としてある.2010年代以降はライブエンタテインメントも盛んになり,オタク文化と関連した対面イベントが興隆した.特に同人誌即売会では他の参加者との交流が重視されており(東 2013),同人誌の流通経路が多様化しても同人誌即売会に参加し続ける理由の一つと考えられてきた.  だが,2020年以降のコロナ禍によって多くのイベントが中止・延期を余儀なくされ,その後オンラインへの移行や規模縮小による再開を迫られた.イベントにおいて参加者は施設運営者やイベント主催者と並ぶ重要なアクターであり,感染症対策として推進された「自粛」要請に対して個別に判断・行動することが求められた.そこでは,どのような行動様式が出現したのか.本報告では,2020年3月以降をコロナ禍と位置づけ,首都圏におけるコロナ禍の参加者の行動変容から,オタク文化と対面イベントの関係性にどのような変化がみられたかを明らかにする. 2. 方法 報告者らは2021年3月にオンラインアンケート調査(「新型コロナ禍におけるイベントスペース利用者調査」(n=1244))を実施した.この調査で2019年以降に同人誌即売会やMAGイベントへの参加経験を持つと回答した人(n=197)を主な対象として,①参加継続型(2019年と2020年以降ともに参加(n=68)),②参加断念型(2019年のみ参加(n=99)),③コロナ禍のみ参加型(2020年以降のみ参加(n=30))でグループ分けした.その上で,コロナ禍以前の参加状況,感染リスクや感染症対策への意識と,コロナ禍でのリアル・オンライン別の参加状況や意識の変化を分析した. 3. 結果・結論 ①~③のグループの中では①がオンラインイベントの参加や支援行動の実施率が高かった.同時に①は他の参加者との同行率や,イベント会場やスタッフに知り合いがいる割合も高かった.一方で②はコロナ禍で同人誌即売会やMAGイベントにとって会場や施設が欠かせないと思うようになったと回答した割合が,他グループより低かった.  以上から,コロナ禍以前から継続して対面イベントに参加している人びとは,対面イベントや場所を維持するための行動に熱心で,その場での交流を重視していることから,対面イベントの継続に強い関心をもつ人びとだと推測できる.オタク文化に関するイベント参加者の分析からは,イベントのオンライン化も許容されながら,対面イベントへの根強い支持も存在していることが明らかになった.  なお,本報告は科研費基盤研究(B)「『高さ』を疑う,『高さ』を背負う: 新しい都市ガバナンスの社会学」(19H01557)の成果の一部である. 文献 東園子,2013,「紙の手ごたえ――女性たちの同人活動におけるメディアの機能分化」『マス・コミュニケーション研究』83: 31-45.野村総合研究所,2005,『オタク市場の研究』東洋経済新報社.

報告番号55

Go To トラベルとGo To イートの促進要因と抑制要因
近畿大学 辻竜平

【1.目的】  日本における新型コロナウイルス(COVID-19)の政府の対応は,一方で感染を抑えつつ,他方で経済活動を少しでも維持するという方針で行われた.その経済活動維持の中でも,特に経済的影響が深刻だった観光業や飲食業の回復のために打ち出されたのが,Go To トラベルやGo To イートといった政策であった.まだ感染が収まったわけでもない状況でこれらを利用しようとしたのは,どのような人々だったのかを実証的に明らかにする. 【2.方法】  マクロミル社のモニターを用いたインターネット調査(CAWI形式)を行った.対象年齢は,15~74歳.設計サンプルサイズは700で,4月7日に緊急事態宣言がなされた7都府県(大都市部)とそれ以外の県(地方部)に分け,大都市部/地方部 × 男/女の4カテゴリが同数となるように割り付けを行った. 【3.結果と考察】  Go ToトラベルとGo To イートの利用回数を従属変数としたポワソン回帰分析を行った.収入については,個人収入を投入する場合と世帯収入を投入する場合を検討した.主な結果は,次のとおりである.  心理変数について,相互協調的自己観にかかわる排除回避がGo To トラベルの利用を促進したのに対して,調和追求は利用を抑制した.調和追求は,相互協調的自己観の主要な側面であり,排除回避は集団主義的な側面であるとされる.まだ感染が収まったとは言えない状況にあって,調和追求の得点が高い人は,協調して感染を収束させるべきであると考えて,旅行を控えたのだろう.一方,旅行に出かける人が増えると,それまで排除回避のために旅行を控えていた人たちが,もはや人目を気にしなくてよいと考えて旅行に出かけたのだろう.  権威主義は,Go To イートを促進し,Go To トラベルを10%水準ながら促進する傾向が認められた.政府という権威が外出を認めたのだから,それに従って外食や旅行をしてもよいと考えたのだろう.  通勤通学時間が長かったり,公共交通機関を利用する人の方が,通勤通学途中でGo To イートを利用するのではないかと考えたが,そのような傾向は認められなかった.しかし,通勤通学時間が長い人は,Go To トラベルを利用した.長時間乗り物に乗ることに慣れていたからと考えられる.他方,通勤通学に公共交通機関を利用する人は,Go To トラベルを利用しなかった.ふだんから公共交通機関に乗っている人は,潜在的に感染不安を感じており,旅行することを回避したと考えられる.  収入については,2019年の世帯収入が,Go To トラベルの利用を有意に促進したが,個人収入は,10%有意にとどまった.旅行に関していえば,当初個人の出張でGo To トラベルを利用する人がいることが報じられ,それを後に抑制したことから,家族旅行により強い効果が現れたのだと考えられる.また,2019年の世帯収入と,2020年の前年度比での収入の増減見込みが,Go To イートを促進した.経済的に余裕のある人たちが出歩くきっかけになったものと考えられる.  雇用形態では,正規雇用に比べて,派遣・契約・嘱託のGo To トラベルおよびGo To イートの利用が低かった.職業間の差は若干あったが,全体としてはあまり大きな差はなかった.

報告番号56

新型コロナ禍における社会意識と強制への価値観
立教大学 村瀬洋一

1.目的  新型コロナ感染は経済活動の制限や、人々の行動の変更など、社会に大きな影響を及ぼした。日本社会における死者数は、先進国の中でも少ない方であり、政府による強制的な都市封鎖などの政策も少なかった。暴動や騒乱などの大きな混乱もなく、社会秩序も存在する。しかし、自粛といいつつ様々な施策を強制するなど、日本における対策には批判も存在する。本研究は、新型コロナ感染に関する各種の質問項目の規定因について、社会階層との関連に着目しつつ、独自の社会調査データを分析し、その規定因を解明する。 2.方法  立教大学社会学部が2019年に豊島区、2020年に世田谷区にて実施した「生活と防災についての意識調査」データを用いて計量分析を行う。予算の都合上、都心部と郊外を比較する形での調査実施となった。世田谷区調査では20歳以上の男女、確率比例抽出法により市内の90地点を抽出し(エリアサンプリング)、最終的に2700人の20歳以上の個人を対象とし1092人(回収率40%)の回答を得た。調査員が対象となるご家庭に封筒を配布し、回収は郵送で行った。回収のために調査員が対象者を訪問することは断念せざるを得なかった。 3.結果  「新型コロナウイルスの感染拡大前と、感染拡大後で、あなたの暮らしは変わりましたか」という問に対して「大きく変わった」と「ある程度変わった」を合わせて73%「あまり変わっていない」と「まったく変わっていない」を合わせて26%だった。変わったこと(複数回答)としては、経済状況23%、家族や友人、パートナーとの関係36%、働き方43%、自宅での過ごし方や食生活52%、健康・衛生に対する取り組み66%、人生設計15%、コミュニケーションの手段35%、政治への関心26%、特に変わったことはない8%だった。  「新型コロナウイルス特別措置法に基づく緊急事態宣言で、日本では都市封鎖のような強制力によらず、臨時休業や外出自粛の要請に国民の多くが応じました。あなたは、今回の日本社会の対応について、どう思いますか」という問に対して「評価する」と「ある程度評価する」を合わせて75%、「あまり評価しない」と「評価しない」を合わせて24%だった。  「自粛という方法を強制して押しつけるのは望ましいやり方ではない」という問に対して「そう思う」と「どちらかといえばそう思う」を合わせて42%、「どちらかといえばそうは思わない」と「そうは思わない」を合わせて58%だった。  これら社会認識や行動の変化を被説明変数として、重回帰分析を行った。生活の変化に関しては、年齢が低いほど、また教育年数が長いほど、変化があったと答える傾向があった。性別ダミー変数や自営業ダミー変数や階層帰属意識は有意な効果がなかった。日本社会の対応については、生活満足感が正の効果を持つが、他に有意な変数はなかった。自粛の強制については、性別ダミーと年齢が正の効果を持ち、男性ほど、また高齢者ほど、自粛を強制することに否定的な傾向があった。 4.結論  重回帰分析では、教育年数が有意なこともあったが、階層帰属意識などは効果がなかった。しかし、実際の収入の変化などは、職業によっても異なるだろうし、政策への評価には、様々な社会的地位や、価値観が影響しているだろう。これらについてはさらなる分析の必要がある。 注 本研究は立教大学学術推進特別重点資金(立教SFR)の助成を受けて実施した。

報告番号57

「社会意識の分断」という観点から見た現代日本の政治意識の構造
立命館大学 金澤悠介

【1.目的】  従来の政治社会学は社会経済的地位の分断に着目して人々の政治行動を説明してきたものの(e.g. Lipset and Rokkan 1967)、近年の日本の政治行動は従来の政治社会学の説明図式ではうまく説明できないものとなっている。第一に、現代日本において、社会経済的地位よりも経済的自由主義やナショナリズムといった社会意識のほうが政治行動に対する影響力が強い(田辺 2019)。第二に、世代間で「保守-革新」の意味合いが異なり、現在の若年層はこの対立軸に沿って政治を理解・評価していないことが経験的に明らかにされている(遠藤・ジョウ 2019)。  社会経済的地位の分断に着目する既存のアプローチに対し、本研究は人々の間に潜在する「社会意識の分断」からその政治行動を説明することを目指す。具体的にいえば、(ⅰ)「保守-リベラル」のように研究者が理論上想定する対立軸ではなく、理想とする社会像、選好する政策、政治志向性といった社会意識の類似性をもとに人々を類型化したうえで、(ⅱ)社会意識類型と政治行動の関連を検討する。 【2.方法】  本研究は2020年10月下旬に実施したweb調査(楽天インサイト株式会社の登録モニターを対象、20~69歳の男女、N = 6600)を分析対象とする。このweb調査では2019年10月1日時点の人口推計をもとに、性別、年代、居住地域について割付を行った。 【3.結果と結論】  理想とする社会像(将来世代への投資についての価値観、普遍主義的態度)、少子化政策への選好、従来的な保守-革新の対立軸(平和主義、日米安全保障条約、従軍慰安婦問題、天皇制)への評価、保守-リベラルについての自己認知をもとに、潜在クラス分析によって、回答者を分類したところ、新リベラル層(回答者の19%)、旧リベラル層(6%)、ライト保守層(23%)、コア保守層(7%)、非政治層(23%)、判断保留層(15%)、黙従傾向層(8%)という7つの類型が抽出された。しかし、判断保留層は「どちらともいえない」などの中間回答を、黙従傾向層は「そう思う」などの左端回答を主に選択していたため、以下の分析からは除外する。  各類型の回答を詳しく見てみると、イデオロギーについての自己認知と理想とする社会像が直線的なかたちで結合しておらず、人々の政治意識は複層的な構造を成していることがわかった。回答者のイデオロギーについていえば、旧リベラル層は従来のリベラル的価値観にコミットし、ライト保守層とコア保守層は従来の保守的価値観を重視している一方で、新リベラル層は自身をリベラルと位置づけながらも、従来的な保守-リベラルの対立軸については中立的な立場をとっている。理想とする社会像については、新リベラル層とライト保守層は将来世代への投資を重視し、普遍主義的態度を持つ点で共通している。一方、旧リベラル層は普遍主義的態度が強いものの、将来世代への投資をあまり重視していない。また、コア保守層は将来世代への投資をあまり重視していないし、普遍主義に対しては否定的な態度を持っている。  なお、当日の報告では、政治意識類型と社会経済的地位および政治行動(支持政党、政治参加、政治的アクターへの信頼)の関連についても議論する。

報告番号58

能力開発・教育政策と<自律><能動>――「市民」に必要な能力は何か (1)
中京大学 亀山俊朗

【1. 目的】  社会政策の規範的目標は市民の<自律>の実現であり、求められる資質は<能動性>であった。こうした認識に立ち、政策類型ごとの自律モデルと期待される能動性を跡づけたうえで、現代の社会政策の主流をなすアクティベーションとそれに伴う能力開発政策・教育政策における市民像、求められる能力や資質を明らかにする。 【2. 方法】 ・福祉国家政策、新自由主義政策、「第三の道」政策などの政策類型をシティズンシップ論をもとに検討し、市民の<自律>を実現するための政策枠組みとそこで市民にもとめられる<能動性>の特色を、先行研究をもとに明らかにする。 ・OECDの教育政策論等における主要概念(キー・コンピテンシー、エージェンシーなど)をめぐる議論を、その参照元の社会理論であるシティズンシップ論、ケイパビリティ・アプローチと比較検討し、その特徴を明らかにする。 ・日本における政策文書等において、上記のような諸概念をめぐる議論がどのように解釈され政策的に展開されようとしているかを検討する。 【3. 結果】 ・OECDなど「先進」諸国の議論では、知識社会・情報社会への転換を伴うアクティベーション政策が主流の一つとなっている。教育政策は従来の学校教育や職業訓練にとどまらない、社会全域にわたる能力開発として主要な政策的課題となっている。 ・OECD等の教育政策論の主要概念は抽象的に提起されており、であるがゆえにケイパビリティ・アプローチなどの社会理論とおおまかな方向性に決定的な齟齬はない。言い換えると、各国・各地域・各領域での実践に詳細は任されている。 ・日本の教育政策文書は、OECDなどがうたう教育理念は日本でもすでに十分に政策に反映されていると主張する。しかし、そのエージェンシーや自由などの諸概念は、ケイパビリティ・アプローチなどの社会理論の想定から外れがちである。さらに、具体的な学校等での教育実践においては中央政府(文部科学省)の理念であっても共有されているとはいいがたく、実現できるような環境や資源も欠けている。 【4. 結論】  従来的な狭義の教育・能力政策だけでは<自律><能動性>の実現や涵養は難しいし、社会政策だけでエージェンシーや自由の拡大は無理である。それらは社会運動のような葛藤的な側面も含めた日々の実践により実現されるものである。これは、制度的なシティズンシップの埒外での「生きられたシティズンシップ」の生成というシティズンシップ理論の現代的課題とも即応している。  <自律><能動性>の確立、あるいは自由の拡大やエージェンシーの涵養には、適切な所得や社会保障、住宅といった社会的な諸資源が不可欠であることは、OECDなどの政策論も認める。20世紀の福祉国家政策のマーシャル的パラダイムには、社会的シティズンシップによる文明的生活の保障が市場での能力主義的競争を相対化し無意味化する、そうであるがゆえに競争的市場が存続しうるという逆説的な含意があった。同様に、社会的シティズンシップの内実を保障しなければ知識化・情報化に対応した持続可能な市場は形成されえない、という問題意識を現代の能力開発・教育政策に見て取ることは可能かもしれない。こうした問題意識は、一方の主流である新自由主義的な政策傾向とときに葛藤し、時に整合していくことになろう。  付記 本報告は、JSPS科研費19H01647の助成を受けたものである。

報告番号59

社会政策におけるエージェンシーと自律――「市民」に必要な能力は何か (2)
目白大学 平野寛弥

【1.目的】  本報告は,社会政策において社会的シティズンシップの保障が個人のエージェンシー(agency)の発揮を通じた善き生の実現(=自律(autonomy))を可能にするための装置として機能してきたという理解を前提としたうえで,近年の社会政策の転換が個人のエージェンシーと自律に与えた影響を検討する.あわせて,リバタリアン・パターナリズム(以下,LP)が社会政策にも導入されつつあることも考慮に入れ,現代社会における個人のエージェンシーや自律のあり方を考察し,それを実現するための新たなシティズンシップを構想する. 【2.方法】  まず,福祉国家再編の政治において支配的となった個人の「能動性(activeness)」を重視する言説と,それを受けて転換を遂げた近年の社会政策を検討し,それらが個人のエージェンシーと自律を大きく制約していることを示す.さらに,LPの代表的論者であるC.サンスティーンの主張(Sunstein 2015; 同 2016)を検討し,LPが個人のエージェンシーと自律にもたらす問題を指摘したうえでその打開策を考察する.最後に,以上の議論を踏まえて,多様なエージェンシーの発揮を許容しつつ自律を実現するための新たなシティズンシップの構想を提示する. 【3.結果】  個人の「能動性(activeness)」を重視する言説は,市場や市民社会,コミュニティへの参加の重要性を強調し,それに向けた具体的な行動(behavior)を求める方向へと社会政策を転換させた.ここに至ってエージェンシーとは,政府の規範的要請に追従する形で自らの「能動性」を発揮することの謂いであり,その意味では個人のエージェンシーも,またエージェンシーを発揮することで達成できると想定されている自律も大きな制約を受けている.  他方でLPは,個人のウェルビーイングの達成に寄与するが,エージェンシーが立脚する当人の意思の所在が不明なほか,エージェンシーの発揮の目的がウェルビーイングの達成に縮減される懸念がある.このような懸念を払拭するには,サンスティーン自身も推奨するようにLPを「能動的選択を促すためのアーキテクチャ」として利用することが望ましい.そのための賭金となるのは拒否権の保証である.それはエージェンシーの発揮を実質的に担保するだけでなく,ウェルビーイングの達成に限らないエージェンシーの発揮(A.センのいう「エージェンシー的自由」)を可能にし,まさしく能動的に自律の実現を図るうえで重要な意味を持つ.またこうした方向性でのLPの利用は,いまやLPや類似の手法が無数に展開され,「相互ナッジのネットワーク」(那須 2020)化した社会における,個人のエージェンシーの涵養という側面からも必要である. 【4.結論】  新たなシティズンシップに求められるのは,多様なエージェンシーのありかたを許容し,それらの発揮を通じた自律の実現を保障するとともに,ナッジが張り巡らされた現代社会において個人がエージェンシーを発揮するための能力を強化することである.具体的には,特定の「能動性」に紐付けされない生活保障の提供や多様な生の承認とともに,現代社会を深く理解し,熟慮や内省を行いつつ適切な選択を下す能力を備えた個人の育成(シティズンシップ教育)が要請されることになる.  (注)本報告はJSPS科研費19H01647の助成を受けたものである.

報告番号60

自律の社会的決定要因と認知資源配分――「市民」に必要な能力は何か(3)
金沢大学 村上慎司

【1.目的】  グローバル化や知識経済化を背景としたシティズンシップ教育について、OECDは日本の教育の文脈では「主体性」に類似した生徒エージェンシーを提唱した。これは自律概念とも密接に関連すると考えられる。多様な分野でエージェンシー概念は言及されるが、自律との概念的関係の見解は統一されておらず、また、シティズンシップ教育への応用を念頭に置いた自律の理論的彫琢が待望されている。  本報告の目的は、ケイパビリティ・アプローチとリバタリアン・パターナリズムにおける自律に関連する議論を社会的決定要因と認知資源配分の側面から検討し、そのシティズンシップ教育への理論的含意を示すことである。 【2.方法】  本報告の研究方法は自律概念に関連する文献読解を通じた理論的考察である。主に、ケイパビリティの概念的サブカテゴリーであるエージェンシー的自由並びにメタ・ランキングを論じたA. センの文献、そして、リバタリアン・パターナリズムの手法であるナッジ・選択アーキテクチャーの理論的議論を提供しているC. サンスティーンの文献を検討する。 【3.結果】  第一に、自律の社会的決定要因を論じる。本報告はエージェンシー的自由として自律を解釈する。この理論的利点は、各人の内的・外的賦与、社会環境の状況、そして、他者からの支援等に応じたエージェンシー的自由の在り方が把握可能となり、こうしたエージェンシー的自由が一定の水準に達しない当該個人に対して関連権利の要求を認め、適切な社会環境の設計や政策介入の必要性を主張できる。そのため、エージェンシー的自由としての自律は、ナイーブな自律の解釈である自己責任論的発想とは異なる。つまり、エージェンシー的自由としての自律は、社会的決定要因という当該個人の外的要因に強く左右されることに留意する。  第二に、本報告は自律の認知資源配分を論じる。本報告は、認知資源というタームを、時間と記憶容量に加えて、注意力や情報処理能力といった認知能力を規定する要因といった包括的な意味で使用する。そして、リバタリアン・パターナリズムが想定する認知資源配分に着眼することで、センのエージェンシー的自由の議論での不備を補う可能性を示す。つまり、現在の情報過多化社会において、認知資源を節約するために何らかのナッジ・選択アーキテクチャーの利用は不可避的であり、これらを活用することでエージェンシー的自由としての自律が実効的に行使されると論じる。  第三に、エージェンシー的自由の行使における認知資源を集中的に配分する対象の優先順位づけを論じる。本報告は、J. ドォーキンとH. フランクファートの自律・人格の議論、そして、センのメタ・ランキングに着想を得て、能動的な選択とナッジ・選択アーキテクチャーへの委託との間で選択する階層的自律が有益であると主張する。 【4.結論】  センとサンスティーンの議論の相補性から、本報告は、社会的決定要因と認知資源配分に基づくエージェンシー的自由と階層性によって、自律概念を特徴づけた。ここから、シティズンシップ教育において、各人の社会的・認知的要因を考慮し、「選択することと選択しないこと」それ自体を選択できる能力開発が重要であるという理論的含意が導かれる。 付記 本報告は、JSPS科研費19H01647、19K02156の助成を受けたものである。

報告番号61

シティズンシップ教育と3つの政治――「市民」に必要な能力は何か (4)
学習院女子大学 時安邦治

【1. 目的】  本発表は,自由主義的シティズンシップと市民共和主義的シティズンシップを調停不可能な対立関係にあるものと捉えず,相補的に理解しようとする.自由主義的シティズンシップをめぐっては,再分配の政治と承認の政治が緊張関係を生じることがあるが,それらを調停する役割を市民共和主義的シティズンシップの主要素である共通善の政治に求める.そうして,市民の諸権利の追求と共通善を両立させるような市民の態度(市民的徳)を涵養することがシティズンシップ教育の重要な課題であり,具体的に市民にどのような能力が必要とされるのかを社会哲学的に考察する. 【2. 方法】  [1] I. M. ヤングの「政治体と集団の差異」および『正義と差異の政治』と,N. フレイザーのヤングに対する批判を主要テクストとして,フレイザーに依拠しながら再分配の政治と承認の政治の緊張関係を明らかにする.また,再分配と承認を求める自由主義的シティズンシップは,時に共通善の実現を目標とする市民共和主義的シティズンシップと緊張関係を生む.  [2] 再分配と承認と共通善という三者の葛藤を調停するために市民的徳が要請されることを論じる.この場合,市民的徳は,参加民主主義を成り立たせる能動的な政治参加と社会全体への配慮を含む.市民的徳の涵養こそシティズンシップ教育の中心的な目標である.  [3] そうした目標をもつシティズンシップ教育は,能動的な市民となるべき児童・生徒・学生にいかなる能力を身につけさせるべきなのかを考察する. 【3. 結果】  シティズンシップ教育を三つの要素に分けたい.再分配の政治に対応するのは政治・経済・社会に関する教育であり,公民分野の科目が中心的な役割を果たす.承認の政治に対応するのは歴史意識と文化的アイデンティティに関する教育であり,地理歴史科が重要な役割を果たす.共通善の政治に対応するのは道徳教育や新設の公共(の一部)および倫理であろう.  公民科教育(政治・経済ほか)は政治経済的平等を主要な価値とし,政治と社会の仕組みを理解するための諸概念と,それらを用いて社会生活を送る術の習得を目指す.地理歴史科教育はアイデンティティの承認を主要な価値とし,諸集団の文化的差異についての感受性を育みながら,1つの政治的共同体として協働していく必要性を学ぶ.道徳関連の教育は基本的人権と共通善の追求を主要な価値とし,市民的徳の涵養を最大の目標とする.  これらの要請をすべてシティズンシップ教育に求めることは,教育への期待が大きすぎるのかもしれない.もちろん教育は万能ではなく,何かにつけ予期せざる結果を生む.しかし,リバタリアン・パターナリズムによって仕組まれたナッジのネットワークの中での「選択」ではない市民の自律の可能性を考えるとすれば,教育に期待するしかないのが現状であろう. 【4. 結論】  こうしたシティズンシップ教育の構想は,市民共和主義的リベラリズムと呼びうるだろう.これは撞着語法かもしれない.しかし,たとえばSDGsが目指されるべき目標として掲げられるような社会では,そのための協働が不可欠であり,個人の尊重と社会的目標の両立を求めるようなゆるやかな共通善の政治が望ましいだろう. 付記 本報告はJSPS科研費(19H01647)の助成を受けた共同研究の成果である.

報告番号62

OECD型グローバル・コンピテンスと多文化シティズンシップ――「市民」に必要な能力は何か (5)
長崎大学 寺田晋

1.目的  2000年代中盤以降、UNESCOや欧州評議会によって、マイノリティ集団の承認を課題とする多文化主義(multiculturalism)から、文化の垣根を越えた交流の意義を強調する間文化主義(interculturalism)への転換がはかられてきた。この転換は個人の問題解決能力の涵養を目指す近年の教育政策とも軌を一にしている。文化的多様性の理解をターゲットのひとつとしたSDGsの策定をうけて、2018年度からOECDの「生徒の学習到達度調査」の評価項目に加えられたグローバル・コンピテンス(global competence、以下、GC)は、文化的多様性を文化間の交流にかかわる課題として捉える点で、国際機関が主唱する間文化主義のアプローチと一致している。本報告では、こうした間文化的な教育へと向かうグローバルな教育政策の動向がもつ意義を明らかにするために、多文化主義にもとづく多文化シティズンシップの観点からGC概念を分析する。両者の比較を通じてそれぞれの特徴と教育政策上の含意を明確にすることが本報告の課題である。 2.方法  OECDのGC概念とそれが参照するUNESCOや欧州評議会の文書の言説分析を行う。さらに、シティズンシップ研究における多文化シティズンシップの概念とGCを比較する。 3.結果  コンピテンスは、知識、技能、態度、価値を組み合わせて具体的な状況に適応し、問題を解決する能力であり、GCは、「ローカルでグローバルな、文化間の問題」を解決する能力とされている。注目されるのは、民族やエスニシティだけでなく職業、世代、家族なども文化をもつ集団であり、それゆえ、すべての個人は複数の文化集団に所属し、文化との関係において独自の立場に立つとされている点である。  これに対し、多文化シティズンシップは、公的な場で通用する多数派文化とそうではない少数派文化との不平等を問題とし、平等の実現には少数派文化の公的制度への反映が必要であるとする。このため、多文化シティズンシップは、国家に焦点を置き、公的な承認を必要とする文化とそうではない文化とを区別する。  GCが文化の間に設ける区別は別の形をとる。グローバルな評価基準であるGCはケイパビリティの概念に依拠し、人間の尊厳を人類共通の価値とすることで、尊重すべき文化の線引きを行う。GCの涵養が直接寄与すると捉えられているのは不平等ではなく差別の克服である。  以上から、平等という観点から多数派と少数派の文化を区別する多文化シティズンシップに対し、GCは人間の尊厳という観点から尊重すべき文化とそうでない文化を区別するということ、マイノリティ集団の承認を目標とする前者と、文化との関係における個人の独自性を強調する後者とでは、集団ごとの差異に適合した教育を志向するのか、個人の多様性を重視した教育を志向するのかという違いがあることが明らかとなった。 4.結論  本報告の分析結果は、文化的多様性をめぐって方向性の異なる教育政策を提唱する多文化主義と間文化主義を総合する必要性があることを示唆する。しかし、多文化シティズンシップとGCは、相互に補完する可能性もあれば、対立する可能性もある。この点で、両者にどのような関係が成り立つのかは、今後実証的に検討されるべきである。 付記 本報告は、JSPS科研費19H01647の助成を受けた共同研究の成果である。

報告番号63

フリースクールの実践からみた子どもの自由と市民/非市民の境界――「市民」に必要な能力は何か(6)
中京大学 森田次朗

【1.目的】  1990 年代以降、日本の教育学の領域では、シティズンシップ教育をいかに構想するかが注目を集めている。それはグローバル化の進展に伴い、市民/非市民の境界が流動化するなかで、社会の構成員として備えるべき資質や能力の獲得機会をいかに公的に保障するかが、教育政策上の課題となっているからである。  だが、シティズンシップ教育をめぐる昨今の研究動向には課題がある。それは、市民/非市民の境界に注目が集まる一方で、市民の内部における排除や分断の問題、すなわち部分的市民の教育問題が看過されているという課題である。たとえば、ここでいう部分的市民とは、女性や障碍者、高齢者、教育機関の中途退学者のように国籍という地位資格の点では同じシティズンシップを持っていても「二級市民扱い」され、十全な権利を享受することができない人々のことである。そのうえで先行研究では、部分的市民のシティズンシップを教育制度の枠内でいかに保障するかという論点が軽視されている。 そこで本研究では、先行研究に多くを負いつつも、市民の内部における教育機会をめぐる重層的な排除/包摂の問題に注目しながら、従来のシティズンシップ教育の知見を再考することを目的として論じていく。 【2.方法】  上記の目的を遂行する際に、本研究は日本で不登校児童生徒の学習保障を目指す「フリースクール」の事例を取りあげる。フリースクールとは、欧米社会にルーツをもつとされる子ども中心主義を掲げる教育形態であり、日本では1980 年代以降、学校に行かない子どもに居場所を提供することを掲げて登場してきた学校外の実践である。このようなフリースクールに注目する理由は、フリースクールとは、学校に通わないことで「問題児」というスティグマを受ける子どもたちと、その保護者や元教員をはじめとする「市民」が、子どもの自由や自主性を掲げて立ち上げた「オルタナティブ」な学び場だという点で、シティズンシップ教育の典型と呼びうる実践であるにもかかわらず、先行研究ではその実態が十分に位置づけられていないからである。  以上の関心のもと、本研究はフリースクールをシティズンシップ教育の実践例として位置づけたうえで、近年、欧米圏で子どもの自由やその社会的条件を分析する際の枠組みとして注目を集めているケイパビリティ・アプローチの視点から、そこで掲げられている市民像と学びの関係性について考察する。 【3.結果】  分析の結果、1)フリースクールでは、子どもたちが既存のあるべき人間像に同一化することを目指して特定の知識や技能を獲得することよりも、自由時間やミーティングを通して学校に行かない「ありのままの自分」を肯定し、そこから自らが進むべき生き方を想像し模索している姿、すなわち「迷う市民像」が重要視されていること、2)こうした市民像を肯定することは、「何もしないこと」(参加の自由、休息権)や「居場所の複数性」が保障されることで可能となることが明らかになった。 【4.結論】  以上の議論から、市民の内部における排除/包摂をめぐる教育問題を考察する際に、子どもの参加と同時に、「市民」になることを強制されない自由に注目することの重要性が明らかになった。今後は、市民像の複数性に着目したシティズンシップ教育研究が重要になる。 付記 本報告は、JSPS科研費19H01647の助成を受けたものです。

報告番号64

非行少年からみた不登校と学校社会――沖縄を中心として
佛教大学 作田誠一郎

少年非行は、「社会を映す鏡」と評される。それは、社会の変化を敏感に反映する青少年が行為主体となり、新たなシステムや対人関係の変容に先んじて犯罪や非行を実行する点にある。現状の少年非行の状況をみてみると、2020年の非行少年の検挙人員(刑法犯・危険運転致死傷・過失運転致死傷等)は、戦後最大の検挙人員を記録した1983(昭和58)年の317,438人とくらべて37,193人と大きく減少している(『令和2年版犯罪白書』)。しかし、「再犯防止推進法」(2016)が施行され、再非行・再犯をどのように防止していくのかが今後の課題となっている。一方、本報告では、沖縄の非行少年を中心に取りあげる。沖縄県は、学校に関してみてみると、学校の管理化における暴力行為やいじめ、長期欠席や高等学校中途退学率が全国的にも高い値を示している(『令和元年度児童生徒の問題行動・不登校等生徒指導上の諸課題に関する調査結果について』)。また沖縄県が実施した「子どもの貧困実態調査」(2016)では、沖縄県内の子どもの貧困率は29.9%であった。この結果を受けて、同年に沖縄県は、「沖縄県子どもの貧困対策計画」(2016-2022)を策定し、その支援に注力することを明言している。  本報告では、非行少年に対するインタビューを通じて沖縄の不登校と学校社会について分析する。インタビュー対象者は、少年院に入院している沖縄県出身の5人の少年少女である。各少年たちは、年齢も家族構成もさまざまである。しかし、この5人が共通していることは、不登校を経験していることである。この不登校の端緒やその背景にある要因について明らかにし、学校の教師がどのような対応や反応を示してきたのかを含めて分析する。これまでにあまり分析の進んでいない非行少年の不登校経験を通じて、不登校の背景にある諸要因と学校の教師の対応について明らかにする。  結果として、非行少年の不登校の端緒をみると、虐待経験や両親の別居、学外のプライベートな遊びなど、多様な端緒が存在した。そのなかで、不登校の状況を維持するような働く場の提供も他ではみられない沖縄社会の特徴として指摘される。このような沖縄社会のネットワークは、学校の人間関係よりも広く深いことが明らかとなった。つまり、一般的な学校区以外のつながりが、非行少年の不登校の要因となる遊びや働く場の提供に関連しているようである。  また非行少年と教師の関係をみると、教師が力で管理するような関係性や教師がレッテル張りをするようなマイナスの経験が認められた。一方で、しっかりと関係性を築いていこうとする教師からの対応も認められたが、教師の言動が非行少年の学校生活に対するイメージや不登校へと向かわせることも指摘される。  今後は、教師に対するインタビュー調査を含めて、非行少年の視点から不登校に対してさらなる分析を進めていきたい。

報告番号65

児童虐待防止政策と家族の統治
九州大学 藤田智子

【1. 目的】  日本では1990年代以降、児童虐待が社会問題化され、政策的課題として認識されるようになった。2000年には児童虐待防止法が施行されたが、その後も児童相談所による虐待相談対応件数の増加や「重大」な児童虐待事案の発生を背景に、種々の法改正が行われてきた。最近では2019年に、児童福祉法および関連法令が改正され、親権者による体罰の禁止が明記されるとともに、他機関との連携を含む児童相談所の体制が強化されるなど、政策的対応が推進されている。  本報告の目的は、近年の児童虐待防止政策について、ジェンダーや家族に関する規範に着目して考察することにある。 【2. 方法】  第二波フェミニズムは、ジェンダー非対称性にもとづく公私二元論が、「家族」内部の暴力を放置してきたことを批判した。そこでは家族を不可侵の領域とするイデオロギーが問題となっている。児童虐待への政策的対応の推進は、こうした公私二元論を再編するものとして理解することも可能である。しかしながら、公権力による家族への介入は、既存の社会規範を再生産・強化してしまう危険性もある(大貫 2014: 第6章)。実際、児童虐待のリスク・アセスメントが、ひとり親家族や貧困の家庭への偏見にもとづいていることも指摘されている(上野 2016)。  以上の視点から本報告では、2000年以降の動向をふまえつつ、近年の児童虐待防止政策が、何を「問題」とし、どのような論理で正当化されたのか、そしてそれは家族や親、とくに母親に対してどのような影響をもたらすのかを検討する。そのため、国会での法案をめぐる議論、政策の運用に関する行政文書(通知、マニュアル等)などの言説を分析する。 【3. 結果】  児童虐待の背景として、近代家族を前提とする諸制度やイデオロギー、社会のジェンダー構造等が指摘されてきたが、その防止政策もまたそうした家族モデルにもとづいている。すなわち、「子どもの権利」「子どもの保護」という名目で、児童虐待概念の拡大・明確化、児童相談所と警察の連携、DV対策と児童虐待防止の連携などが進められるなかで、家族への統治の網の目は拡大してきた。そこでは、「理想的な家族」でないことは虐待の「兆候」であり、「理想的な家族」を営むことが親の義務となっている。 【4. 結論】  児童虐待に関わる政策は、子どもを持つすべての家族を統治の網で取り囲みながら、「理想的な家族」の規範を押し付け、そこから「逸脱」する親への介入を可能としてきた。それによって、家族の理想像やジェンダー規範は再生産・強化されている。 【文献】 大貫挙学,2014,『性的主体化と社会空間――バトラーのパフォーマティヴィティ概念をめぐって』インパクト出版会. 上野加代子,2016,「『児童福祉から児童保護へ』の陥穽――ネオリベラルなリスク社会と児童虐待問題」『犯罪社会学研究』41: 62-78.

報告番号66

旧南洋群島の植民地支配とキリスト教――それは植民地支配を支えたのか
上智大学 松本いく子

【1. 目的】  グアムとハワイの間に広がる旧南洋群島・現ミクロネシア地域は、人口に占める日系人の割合が2割を超える島もあり、日本の近代史と深く関わった。戦前はスペイン・ドイツ・日本、そして戦後はアメリカによる植民地支配が1世紀にわたって重層した。太平洋戦争終戦までは大日本帝国の地図上あったミクロネシア島嶼では、今日も植民地支配や戦争の記憶が語り継がれている。しかし、今日の日本ではその記憶や認識は風化し、戦後日本の記憶から抜け落ちた地域と指摘する研究者もいる。  本研究は、1850年代に宣教が始まったキリスト教が、植民地統治時代を経て島の社会に土着化した過程や、戦前・戦中にキリスト教会(カトリック・プロテスタント)が果たした、植民地支配に対する社会正義における役割などを歴史社会学的視点で考察する。外来宗教のキリスト教会が、島社会のソーシャルキャピタルに変容した過程を明らかにする。【2. 方法】 【2. 方法】  本研究は、ミクロネシア地域のマーシャル諸島の事例に焦点を当てる。植民地政策に関連する歴史資料のみならず、当時の変動する統治体制のなかで実際に現地と関わった日本人や、現地で活動したドイツ人・スペイン人宣教師らが残した資料の分析することを通して、キリスト教が島民社会とどのような関係を構築してきたのかを明らかにする。 【3. 結果】  1850年代にアメリカのプロテスタント宣教団が始めたミクロネシアへのキリスト教宣教は、ドイツによる植民地化(1880年代)に先行していたが、キリスト教は植民地支配者側と協働関係にあったと見られることが多い。これは、植民地統治時代に活発化したカトリック・プロテスタント教会の社会事業活動を「文明化の使命」と捉える視点ともいえるだろう。  しかし、当時の日本政府職員や宣教師による手記などの歴史資料は異なる視点も示す。例えば、1914年にミクロネシア地域を軍事占領して一般ドイツ人やドイツ人宣教師を追放した新統治国日本に対して、多くの島の島民が懇願した最優先事項は、日本政府が提示した道路や橋ではなく宣教師の再来であった。  宣教開始後半世紀余りで、キリスト教がミクロネシア地域の島民社会に深く定着していたことを示すこの事実は、日本の南洋群島統治政策におけるキリスト教会(カトリック・プロテスタント)の位置付けに強く影響した。キリスト教会は土着化し、植民地支配に対する島民の社会正義の媒体としても機能していた。 【4.結論】  従来、植民地支配において支配者の側にあったとされがちなキリスト教教会が、ミクロネシア地域・マーシャル諸島においては土着化やインカルチュレーションを経て、島民と連帯する教会に転化したことが明らかになった。外来宗教が島民や彼らの文化と相互関係を構築しながら土着化し文化創造する過程で、島嶼社会のソーシャルキャピタルに変容したとも考えられる。これは、今後この地域の脱植民地化への過程においてキリスト教会がもたらした影響を考察する宗教社会学的な研究の意義にもつながるだろう。

報告番号67

戦災犠牲死の検証にもとづく文化財とするために――戦災で犠牲となった沖縄県渡嘉敷村民調査から
岩手大学 麥倉哲

1.目的 本報告の目的は、戦争による犠牲死について、犠牲死者一人ひとりの死の状況を検証し、解明された事実を住民目線で記録化し、伝承記録とするものである。これはまた、戦災地住民による伝承活動の拠り所とする文化財化の取り組みの一環である。報告者は東日本大震災犠牲者一人ひとりの生きた証を記録し、語り継ぐ活動を継続中である。沖縄戦における犠牲死者のことが忘れ去られ戦災が風化しないために、災害検証と伝承活動の視点を投入することが有効と思われるのである。戦争災害の記録化については、多様な専門分野による取り組みがなされているが、社会学的な貢献も重要である。証言者と信頼関係を構築した上での聴き取りにより、これまで証言をしてこなかった多数の方々から聴き取りができたからである。 2.方法 太平洋戦争中の沖縄県渡嘉敷村民の犠牲死者は500名を上回る。この犠牲死について、沖縄県平和の礎(いしじ)関係資料、渡嘉敷村関係資料、各種先行研究資料、新聞記事等を精査し、また戦中に渡嘉敷村民であった住民と元住民およびその遺族への聴き取り調査をした結果を報告する。聴き取り調査は、2012年から2021年の約10年間に約100名の方々の協力をえたものである。①対象者自身及び家族の戦争体験、②家族や近親者・知人の死の状況、③死をもたらす様々な要因、④戦後の人生の歩み等について聞いている。 3.結果 集団自決(強制集団死)以外にも多様な犠牲死があること、戦時下の島(渡嘉敷島)の秩序において強烈な命の格差があったこと、集団自決において第一の場と第二の場では様相が異なること、自決場へ向かう(追い立てられる)ルートは複数あったこと、自決により犠牲となった人と犠牲とならなかった人との差異がみられること、自決場に行く前に止められた人など住民の間でも犠牲における差異がみられたなどが明らかとなった。日本軍の中で、将校と下士官、防衛隊などの序列があり、住民、慰安婦、朝鮮人軍夫などの上下関係がみられることも分かった。 4.結論 沖縄県渡嘉敷村の戦争犠牲死者については、集団自決において軍命があったかどうかで、島内を超えて、激しい議論がなされた。この軍命をめぐっては、教科書における記述問題が発生し、裁判で争われることになった。こうした経過の中での村民の聴き取りは困難が予想されたが、非常に多くの体験者、遺族、関係者の方々の協力がえられ、戦争体験を文化財としてのこすというスタンスで村内外の協力をえることができた。 こうした経緯から、①戦時下において著しい命の格差があったこと、②その中で犠牲となった村民は死を望んだものではなかったこと、とくに子供たちは生きたいと切望したこと、③それでも捕虜になるまえに死ななければならないと思想教育され、そこから逃れようのない状況をつくられたこと、④自決ではなく軍による他殺によって犠牲となった人が少なくないこと、⑤子どもの死が多いこと、⑥自決場へ向かわせないような差配をする軍関係者もいたこと、⑦投降するのも命がけであったことなどが明らかとなった。こうした知見を、時間的、空間的に整理し、犠牲死を類型化し分析した結果を示したい。また住民目線の戦災伝承記録誌の成果の一端を示したい。

報告番号68

コロナ禍のフランスにおける緊急事態宣言がもたらしたもの:医療状況と医療者・インターン学生の過労――
千葉経済大学 佐藤典子

【1.目的】 コロナ禍においては、世界中の多くの国が緊急事態宣言などにより、人の移動はもとより、経済的な流れも止まってしまった。とりわけ、フランスでは、緊急事態宣言時において、エマニュエル・マクロン大統領が「これは戦争である」と発言したことにより、アカデミズムの世界やリベラルな層を中心に反発や大きな危惧がもたれた。そこで、この緊急事態宣言による数か月のロックダウンによって、フランスの状況とりわけ、医療状況や医療者の行動・意識はその後、どのように変化したのかについて、統計、先行研究、インタビューなどを通して考察する。 【2.方法】 各種統計、研究会での発表結果、また、医療スタッフ、看護学生、助産師学生、インターン学生への現地でのインタビューを通して、第一波からの第三波までの医療の状況や働き方、それにおける問題点などを抽出する。 【3.結果】 フランスの第一波においては、①感染経路などが不明であり、文化的にマスクなどの感染予防が不徹底であったことなどにより、高齢者中心に多くの死亡者が出たこと。これに対し、②フランス政府は「緊急事態」を宣言し、大統領が「戦争状態にある」と述べ、憲法学者などを中心にその是非が問われたが、医療の現場では、特例的に従来、行えなった治療方法、例えば、看護学生などが医療行為を行うことや蘇生行為を行う資格のない医師が短期研修により、行えるようになることで多くの救命行為が可能となったということである。 【4.結論】 統計資料、研究発表、医師、薬剤師、看護師、看護学生へのインタビューにより、以下のことが分かった。2020年10月末、2021年3月末のロックダウン時は、第一波と異なり、まだ医療現場には余裕がある(しかし、このままの状況では危険な状況つまり、ICUの病床等が不足する)と述べていた。というのも、第一波の時と異なり、①医療消費資材:マスクやカザック(医療ガウン)が十分にあり、②医療を行う医師・看護師の確保およびその練達:症状を早期に分類し、治療手順も定まっている、③フランスの特徴的制度であるHAD:L’Hospitalisation A Domicile(在宅入院制度:亜急性期の医療行為を医療機器やスタッフ全て入院時同様に病室を家庭で再現)により、従前であれば入院すべき、がん化学療法や周産期医療また中度のコロナ治療を在宅で行うことで、病院のベッドを空けることができ、また医療スタッフも最小限に抑えることができるようになったからだという。2021年6月16日の首相発表では、感染者数は5千人を下回り(ロックダウン前は4万人を超えていた)、昨年8月より最も低い水準であり、ロックダウンを6月20日に前倒しするという。事実、飲食店を含め、商店は再開し、公共施設の利用も行われており、それらはワクチンの効果と言われている(人口の四分の一が完全接種を終え、50歳以上、基礎疾患を持つ者の85%が第一回の接種は受けている)。その一方、スタッフの過労はピークに達し、特に、学徒動員された医療系学生の過労、自殺の増加、孤立また、教育の不十分さへの不安、不満が問題となっていることが分かった。

報告番号69

COVID-19対策の日英比較――ワクチン接種に焦点を当てて
防衛医科大学校 金子雅彦

【1.目的】  ワクチン接種は一次予防内の特殊予防である。2020年からCOVID-19パンデミックが世界中で起きているが、ワクチンが開発された後、どのようにして多くの人に素早くワクチン接種するかが各国で課題となっている。それには、ワクチンを注射する人の確保が重要である。本報告はワクチンの打ち手の確保に焦点を当てて、日英比較を行う。 【2.方法】  日本とイギリスの公的資料やマスコミ報道などを分析資料として用いる。 【3.結果】  イギリスでは、ワクチンが開発途上の段階から、ワクチン接種を素早く行うための法律改正が議論されていた。2020年10月16日にヒト用医薬品規制2012が改正され(ヒト用医薬品[コロナウイルスとインフルエンザ][改正]規制2020)、ワクチンの打ち手の範囲が大幅に拡大された。従来の法律で認められていた医師や看護師以外の医療資格を持つ保健医療職者や、さらには保健医療資格を持たない者にまで拡大された。12月8日、ワクチン接種が開始された。ワクチンの打ち手は、医師、看護師、救急隊員、理学療法士、研修を受けた保健医療資格を持たないボランティアなどが担う。こうした対応の結果、世界でもトップクラスのワクチン接種スピードとなった。  他方日本では、2021年2月17日にワクチン接種が開始された。イギリスと同様に従来の法律上では、医師と看護師(関連職種)のみがワクチンの打ち手として認められていた。4月26日に厚労省の事務連絡が出された。法律は改正せず、一定の条件付きで集団接種の特設会場においてのみという、時限的・特例的な取扱いとされた。5月18日、神奈川県大和市で歯科医師が初めて集団接種に参加した。5月31日現在、打ち手の資格範囲を拡大することが検討されている。 【4.考察と結論】  ワクチン接種のスピードや打ち手の範囲に関する日本とイギリスのこうした違いの原因は何に由来するだろうか。一つは、人口当たりCOVID-19感染者数・死亡者数が違うことである。2020年12月31日時点で、イギリスの100万人当たり累積感染者数は36771人、100万人当たり累積死亡者数は1084人。それに比べると、日本はそれぞれ1864人、26人とかなり少なかった(Our World in Data: https://github.com/owid/covid-19-data/tree/master/public/data 2021年5月28日アクセス)。この点から日本の「ワクチン敗戦」の立場には与しない。  もう一つは、法システムが違うことである。イギリスは英米法(common law)系である。英米法は伝統的に判例を第一次的な法源とし、事実に即した具体的帰納的思考をする。したがって、法律改正も素早かったし、その内容も大胆だった。他方、日本はヨーロッパ大陸法(civil law)系である。大陸法は制定法、とくに法典を第一次的法源とし、法典の抽象的規範からの演繹的操作によって結論を出そうとする。法システム全体の整合性に気を配るため、法律改正にはかなりの手続きを要する。結局法律改正せず、医師・看護師以外の打ち手は集団接種の特設会場における時限的・特例的な取扱いにとどまっている。 ※本報告はJSPS科研費19K02064の研究成果の一部である。

報告番号70

新型コロナウイルス感染症による日本人の生活と意識への影響――JGSS-2021調査の結果概要
大阪商業大学 吉野智美

【1.目的】 本報告の目的は、新型コロナウイルス感染症(COVID-19)が日本人の意識や行動に与えた影響について明らかにすることである。COVID-19の感染拡大は人々の日常生活を大きく変化させた。感染拡大防止のために必要とされる行動や移動の規制により、職場、学校や家庭などでさまざまな影響を及ぼし、人々の意識にも変化をもたらした。COVID-19の感染拡大に鑑みて、日本版総合的社会調査(JGSS)では、2021年に実施した調査(JGSS-2021)に、COVID-19に関する16の設問を組み込んだ。感染に対する不安、感染脆弱意識、予防行動、行動自粛、感染についての責任と情報源、生活や仕事への影響、ワクチン接種の希望、政府 の対応への評価など、幅広いテーマについて尋ねている。  JGSS研究センターが2003年から中国、台湾、韓国と共同で取り組んできたEast Asian Social Survey (EASS)に加えて、ヨーロッパ諸国との比較可能なデータを収集するために、European Social Survey(ESS)と連携して、「政府が感染拡大対策で優先するべき事項」に関する意識について、経済対策とのバランスや移動制限などを含む4つの設問を入れた。  シカゴ大学が2020年5月に実施した調査によると、アメリカ人の幸福度は過去50年間でもっとも低く、孤立していると感じている人の割合も感染拡大前に比べて増加した(NORC, 2020)。本報告では、COVID-19についての意識や感染拡大による日常生活への影響を明らかにし、コロナ禍において日本人の主観的健康感や生活満足度にどのような変化があったかを考察する。 【2.方法】 本報告では,2021年1月から3月に実施されたJGSS-2021を主に用いる。この調査は、全国440地点で20歳~89歳の男女6,600人を対象に留置法で実施され、回収率は約53%であった。JGSSは、2000年からおよそ2年に一度実施されている反復横断調査で、主観的健康感、精神的健康や生活満足度などを継続して尋ねている。20年以上にわたり13回実施したデータを比較することで、人々の生活や意識がコロナ禍でどのように変化したかを検証する。 【3. 結果と結論】 COVID-19感染拡大は、あらゆる側面で人々の意識や行動に影響を与えた。感染拡大による影響の全体像を把握するとともに、性別、年齢、雇用状況や世帯構造などの属性別に検証することで、感染拡大の影響を強く受けた層を確認すると同時に、感染拡大以前から経済基盤が脆弱な層や社会的孤立につながりやすいとされている人々の生活にどのような影響を与えたかを明らかにする。 【参考文献】 NORC, June 2020, Issue Brief: Historic shift in American’s happiness amid pandemic, https://www.norc.org/PDFs/COVID%20Response%20Tracking%20Study/Historic%20Shift%20in%20Americans%20Happiness%20Amid%20Pandemic.pdf 【謝辞】 日本版General Social Surveys(JGSS)は、大阪商業大学 JGSS 研究センター(文部科学大臣認定日本 版総合的社会調査共同研究拠点)が、大阪商業大学の支援を得て実施している研究プロジェクトである。JGSS-2021は、京都大学大学院教育学研究科教育社会学講座の協力を得て実施し、文部科学省特色ある共同研究拠点の整備の推進事業JPMXP0620335833、JSPS科研費JP20H00089の助成を受けて実施した。データの整備は、JSPS人文学・社会科学データインフラストラクチャー構築推進事業JPJS00218077184の支援を得た。

報告番号71

コロナ禍における自助組織の新たな展開
九州看護福祉大学 竹中健

【目的】  コロナ禍において医療・福祉分野に展開する自助組織の活動は、実施が困難な状況にある。当事者とそこにかかわる専門職スタッフは、どのようにこの現実と向い合っているのかを明らかにし、共有可能な新たな交流の仕組み、グループ活動のあり方を模索することを本研究の目的とする。 【方法】  熊本県に展開する脳卒中を患った人びとによる自助組織Xに着目する。組織形成の歴史、活動内容の変遷、担われてきた社会的役割の変化に目を向けて考察した。組織の立ち上げに深くかかわり、その活動を築いてきた支援者にインタビューをおこなった。コロナ禍における活動の現状を聞き、今後の組織や活動のあり方を当事者や支援者らとともに模索した。 【結果】  対面的な活動は、コロナ禍の影響により2021年6月の現時点ではほぼすべての活動が中断されており実施されていないが、会報による会員間の情報発信は継続している。今後SNSによるネットワーク創りが期待されるが、高齢者にとっての機器の扱いと操作をどのように支援できるのかが課題である。  これまでXが担ってきた生活支援や就労支援は、2000年以降、徐々に介護保険や就労支援の制度的支援に置き換わり、近年では自助組織が提供する役割としてはウエートが小さくなった。こうしたことから、会員数は減少し、現会員の高齢化がすすんだ。Xの賛助会員を除く正会員の2021年6月現在の会員数は20名ほどである。しかしながら当事者間の情報交換や交流という最も自発的な相互行為とそこから提案される様ざまな楽しみのための活動や感情的な支援は、こうした自助組織の会のなかでしかその展開を期待できないようにもみえる。今後どのような形で、会員相互の連携や共同性が展開していくかは、引き続き注目される。とくに若手の会員の獲得と、彼ら彼女らの期待に見合うような活動の展開が今後の課題である。 【結論】  Zoom等を用いた情報発信や交流は、これまで地域ごとに展開してきた各支部のコアメンバーたちが物理的距離を越えて容易につながり、情報の発信や共有ができる可能性を秘めている。これまで自助組織Xが30年以上にわたって築き上げてきた当事者による当事者のための自助活動は、介護保険法や障害者自立支援法による社会的支援の制度化に大きく貢献してきた。今後は、当事者間のよりパーソナルなつながりをその会の目的としてメンバーに共有され、交流が展開・促進していくことが予想される。

報告番号72

「給付金のたかり屋」言説にみる社会的排除のカテゴリーとその変容――
石巻専修大学 野島那津子

1 目的  イギリスではとりわけ緊縮財政以降、「給付金のたかり屋」等の言葉を用いて、なんらかの給付金を「不正に」受給していると思われる人びとを非難する言説空間が形成されてきた。2012年当時の財務大臣ジョージ・オズボーンは、公的支援は「働き者」や「勤勉な家族」のためにあると主張し、給付金の受給者を「(働かない)怠け者」と暗に非難した。メディアではそうした人びとを「給付金のたかり屋」や「給付金の不正受給者」として取り上げ、社会保障費を食いつぶすとされる「怠け者」に対して厳しい視線が注がれ続けている。こうした言説によって、給付金受給者のスティグマが強化され、必要な支援を受けられない人びとが増加することが指摘されており(Garthwaite 2016; Patrick 2016)、社会的排除の加速化や不可視化が懸念される。本報告では、こうした「給付金のたかり屋」言説において、具体的にどのような人びとが「給付金のたかり屋(benefit scroungers)」として指し示されてきたのかを明らかにし、社会的排除のカテゴリーやその変容について考察する。 2 方法  「給付金のたかり屋」に関するイギリス発行の新聞・雑誌記事のテキストデータを収集し、データの整理・分析を行う。本報告では、どのような人びとが「給付金のたかり屋」やその類語によって呼びあらわされ、どのように説明されてきたかについて、新聞記事のテキストデータを中心に検討する。 3 結果  「給付金のたかり屋」やその類の言葉は、総じて障害者との関連で用いられる傾向にあるが、とりわけ緊縮財政以降頻繁に関連づけて論じられる。また、外国人、移民と結びつけたものも多く、シングルマザーや肥満者への言及も散見される。こうした人びとの「特徴」として、しばしば「怠け病」や「働きたくない病」などという言葉とともに、労働を忌避する「態度」の説明が加わる。 4 結論  総体的なトレンドを把握するにはさらなる検討が必要であるが、「給付金のたかり屋」言説における社会的排除のカテゴリーは、その複層化ないし排除のさらなる「内部化」への移行傾向を示唆している。こうした排除カテゴリーの変容は、社会保障改革や移民政策の影響をかなりの程度受けていると考えられるが、誰を「怠け者」や「給付金のたかり屋」とみなすかが社会的に選別されるプロセスも考慮に入れる必要がある。 文献 Garthwaite, K., 2016, “Stigma, Shame and ‘People Like Us’: An Ethnographic Study of Foodbank Use in the UK,” Journal of Poverty and Social Justice, 24(3): 277-89. Patrick, R., 2016, “Living with and Responding to the ‘Scrounger’ Narrative in the UK: Exploring Everyday Strategies of Acceptance, Resistance and Deflection,” Journal of Poverty and Social Justice, 24(3): 245-59.

報告番号73

医療保護入院を巡る「中空構造」――「家族化」「商品化」された「残余」モデルを越えるために
兵庫県立大学 竹端寛

【目的】  社会的入院・入所は、福祉おける「家族化」「商品化」が温存された「残余的」福祉モデルの典型例である(竹端2018)。その中でも、家族等の同意に基づく強制入院という「医療保護入院」は、その残余的性質が色濃く反映されている形態であり、その形態の入院は10万人を超える。本研究ではこの医療保護入院の論理構造を分析し、本制度に象徴される「家族からの排除/家族への排除」の実態や課題を明らかにしたい。 【方法】  本研究では医療保護入院における家族責任の論点を検討すると共に、国家責任が免責される構造がなぜ温存されているのか、も検討する。 【結果】 医療保護入院の起源を辿った後藤基行(2020)は、以前の同意入院時代には「家族をケアから「解放」するメカニズムとして機能していた」と指摘するが、少なくとも現行法においては別の現実がある。なぜならば、医療保護入院とは、上野千鶴子(2011)の言う「ケアされることを強制されない権利」が対象者にとって侵害されるだけでなく、入退院の責任を家族等が担い続けるという点で、依然として家族にとっては「ケアすることを強制されない権利」の侵害が続いているからである。「当事者は地域生活をしたいが、家族が入院を望む」という二項対立の図式ではなく、サービス提供体制の不十分さが家族責任という形で転嫁され、「家族丸抱え」の状態のみながらず、「施設丸投げ」の是非の判断も家族に課せられ、家族責任を抱え続けるからである。そして、この状態では家族は当事者と利益相反関係であり、当事者の「代弁者」にもなり得ない。 日本では精神科病院の9割が民間病院であるが、私人である民間病院の指定医が診断し、家族等という私人が同意すれば、強制的に入院させることができる(姜2020)。この構造は公的責任の放棄であり、強制医療における「商品化」「家族化」の温存である。強制入院を私人である家族や民間病院の指定医に委ねること自体、国家が自らの責任能力や判断能力、事理弁別能力を放棄しているとは言えないか。これは、臨床心理学者の河合隼雄(1999)の「中空構造」概念を援用するならば、強制医療における「中空構造」の温存でもある、といえる。 【結論】  「家族からの排除/家族への排除」を真正面から問い直すには、この「残余」的福祉の「中空構造」を再考する必要がある。行政責任を回避し、家族や民間病院などの私人に支援・判断の責任を「家族化」「商品化」し続ける理由は何か。そこにどのような社会的抑圧の放置が黙認されているのか。誰がどのように責任を取るべきか。これは家族責任のとらえ直しであり、国家や医療・福祉の専門職が取るべき責任と取れるはずのない責任の峻別でもある。この視点から、医療保護入院が機能し続ける日本社会のケア体制の矛盾や、それに繋がる家族からの排除・家族への排除に関して考察したい。 <文献> 河合隼雄 1999 『中空構造日本の深層』中公文庫 後藤基行 2020 「精神衛生法下の同意入院と現行医療保護入院 —ケア義務からの「解放」という論点」『精神医療』 (97) 83 – 93 姜文江 2020 「医療保護入院制度を廃止しなければならない理由」『精神医療』(97), 53-6 竹端寛 2018 「「家族丸抱え」から「施設丸投げ」へ─日本型”残余”福祉形成史」『学術の動向』23(9),34-39 上野千鶴子 2011 『ケアの社会学』太田出版

報告番号74

社会的孤立と社会的処方の実践――南医療生協のおたがいさま運動を事例にして
名古屋大学 河村則行

1.研究の背景  現在、経済格差が拡大するなかで、社会的孤立や貧困などの社会環境が健康格差を生み出していることが社会問題になっている。M・マーモットは、「人々が生まれ、育ち、暮らし、働き、老いる社会環境(健康の社会的決定要因)こそが、乳幼児期、青壮年期、老年期の健康と健康の不平等に深く影響している」と論じている。貧困・社会的孤立は、自分の生活をコントロールできない(無力化disempowerment)ことから、健康リスクを高める要因であり、健康格差を回避するには、社会環境の改善、エンパワーメントが求められる。そして、健康格差の問題解決の一つの方法として、社会的処方が注目されている。それは、薬の代わりに社会的なつながりを処方することであり、住民の健康や生活に影響を与える社会的な背景要因に気づき、本人のニーズに合わせた社会資源等につなぎ、伴走的で継続的な支援を行うことで、より健康で幸せな生活(ウェルビーイング)を送ることができるとされる。そこでは、生活の問題を抱えていても助けての声を上げられない人を見つけ、ボランティア団体などの地域の社会資源とつなぐ「リンクワーカー」の役割が重要になる。しかし、分断化、個人化が進み、地域コミュニティの衰退が言われるなかで、地域に埋め込まれたネットワークが存在するのかが、問題となる。 2.調査対象と分析枠組み  本報告では、名古屋市南部の不利地域で社会的処方の実践に取り組んでいる、南医療生協のおたがいさま運動の事例をとりあげる。これは、「困っている」人に対して地域の組合員が生活支援(草刈り、家具の移動、ゴミ出し、散髪など)を実施する活動であり、実際に、地域で、いろいろな理由で社会的に排除されている人々を継続的に支援し、生活習慣の行動変容をもたらし、その人のウェルビーイングを高めている。そして、以下の点に焦点をあてて、その社会的処方が成功している要因を分析する。第1に、制度にもとづくフォーマルなサービスと自発的なインフォーマルなサービスとをどのようにつないでいるのか。誰がリンクワーカーとなっているのか、その能力の特徴はなにか。第2に、なぜ、社会的に排除された人々の生活習慣の変容、行動変容は起きたのか。第3に、地域にどのようなソーシャルキャピタルが蓄積されているのか、どのようにして信頼と集合的効力感が醸成されたのか。 3.研究の意義 この地域の事例は、国の政策として進められている社会的処方、地域包括ケアシステムの課題を考えるゆえで参考になるだけでなく、地域のソーシャルキャピタル、集合的効力感の一つのタイプとして研究の意義があると考える。なお、本研究は、基盤研究B(名古屋都市圏の「見えない格差」:何が地域社会のウェルビーイングを規定するのか)の成果である。

報告番号75

Embodied difference in manhood: Positioning disabled Xhosa men subjectivities――
Rhodes University, South Africa ThokoSipungu

“The goal of this study is to explore the ways in which Xhosa men with physical disabilities construct and define and also negotiate Xhosa manhood masculinity for themselves. This goal stems from the claim that a man’s physical body in the Xhosa culture and context is enmeshed in a web of social obligations. Therefore, this paper seeks to explore how the participants respond to cultural and social demands of manhood which place an emphasis on an able-bodied corporeality. This paper begins by highlighting the distinct ways in which the two groups of participants– injury pre-initiation, and injury post-initiation – perceive their disabilities in relation to initiation, and in subsequent manhood. I interviewed men who acquired the impairments before initiation and those who acquired them post-initiation not only as a method to broaden the scope of a “geography of disability” but also to enable comparative analysis in terms of the impact of disability on the participants’ subjectivities in constructing, defining, and negotiating Xhosa manhood identities.

The main goal of this paper is underpinned by an interactionist concern as, at the core, it seeks to study these men’s social selves and further investigate how they negotiate, on a daily basis, their disabled selves amongst other men and women within their communities. Therefore, Cooley’s (1902) theoretical notion of the looking-glass self, Bourdieu’s (1990) concept of capital, and Shilling’s (2005; 2012) corporeality, and the notion of personhood are employed to frame the participants’ modalities of negotiation.

I begin by exploring the experiences of the men who acquired their physical impairments after initiation. The main observation from this group is that they regard the acquisition of impairments and the subsequent disability as a second initiation because it set them back to ‘square one’ in terms of their manhood duties and responsibilities and the fact that they had to figure out anew how to be a different kind of man, in this instance disabled men. The paper proceeds to explore the alternative masculinities that emerge from these men as a result of the impairment. In this paper, I make a distinction between being ‘a man after disability’, and ‘becoming a man with a disability’. The former group are the ones who acquired the impairments after initiation while the latter acquired the impairments before initiation. I make this distinction because their routes and transitions to manhood are markedly different and therefore necessitate exploring separately.

The paper proceeds to explore the experiences of men who acquired their impairments before initiation and their views in relation to becoming disabled men. The main observation in this section is that this group of men viewed initiation as an entry into full personhood due to the existing stigma surrounding not only being a boy (inkwenkwe) in this context but also disability. From their accounts, I gather that they could not wait to become men as they saw it as the only way to acquire respectable personhood within their communities.

This paper also explores the participants’ views regarding the impact of being disabled on their manhood identities. This is in line with the literature that claims that disability negatively impacts upon men’s masculine identities . The main observation in this section is that both groups of men (acquisition of impairment before and after initiation) do not see disability as impacting their manhood identities in any form and therefore, no loss of masculinity. There is a neat separation between manhood and disability which does not allow for the two identities to intersect. It is for this reason that the men do not speak about their impaired bodies when they speak about manhood. Lastly, while the participants do not conceive of any threats on their manhood subjectivities resulting from their impairments, I discover that they instead see their social status as impacted. The last section is therefore an exploration of the ways in which they conceive of their status as impacted.


報告番号76

Could the reason for kodokushi be interpreted in a different way? A new approach analyzing the death of aged men living alone in Japan in accordance with the disappearance of business life roles.――
Çankırı Karatekin University TanerSABANCI

“Traditional Japanese ‘ie’ family structure, that three generations live together, has begun to replace with the nuclear family with the acceleration of industrialization and urbanization after the II. World War. The elderly, who are in a reputable position in the traditional structure, had started to become lonely with these transformations. The nuclear family structure, which became widespread as of this period, turned into a form with few children and even without children, and the increase in the rate of never-married or divorced individuals resulted in a further shrinking of the family structure. It is observed that the number of Japanese living alone has increased due to the high rate of the elderly population and the transformations in the family structure. Thus, a new household type consisting of ‘old women and men, living alone’ has been formed. This formation has caused the phenomenon of ‘kodokushi’, which is placed at the center of this research, to exist as a social problem. As the concept, kodokushi is expressed as those elderly people living alone, especially elderly men, die in their homes and their deaths are noticed after days, weeks or even months. The status of the elderly living alone and the increasing number of kodokushi cases each year have been limitedly reviewed in the studies evaluating the demographic and family structure of the country. In addition to this researches, there are also studies evaluating the country’s urbanization, industrialization and economic situation as the contributing factors to the problem. Moreover, the situation of the elderly living alone has also been the subject of health and social work researches. In this study, the kodokushi phenomenon and the condition of elderly men living alone will be evaluated in the context of Japan’s business life, unlike previous researches that have limited the issue. The transformations in the family structure show that the roles and bonds to be formed by the family are weakened. Contrary to this, in Japan, where the young population is decreasing, the need for the elderly workforce increases and the characteristics and innovations of the business life show that the elderly have a different connection/relationship/role. These indicators refer to the related community where the elderly live. The disengagement of this role and bond, after retirement, are the indicators of the unrelated society. This unrelated society briefly states the disappearance of the business life bond that the elderly have for many years and the disappearance of their business roles. In addition to being away from the familial roles of the elderly living alone, the disappearance of business life belongingness in the unrelated society brings more loneliness. The point emphasized here is that, unlike other studies, the formation of kodokushi and the current situation of the elderly male population can be explained with the concept of bond, role, performance, and stage.
In this study, which aims to examine the subject with the concepts of bond, role and belonging that consequence of business life, the qualitative research method will be used in order to reveal more clearly the opinions of the participants about the research topic. In the research that was regionally limited to Tokyo, it was decided to collect the main data of the research through semi-structured interviews with three participant groups determined with different samples. After the research area that was restricted to Tokyo, it was decided to use the purposeful maximum diversity sampling for the first group of this study. In this direction, it is aimed to reveal the effect of business life on the bond, belonging, and role formation of individuals through male participants between the ages of 20-59, labeled as “”the working group before retirement age””. Thus, it will be possible to evaluate the common perception among the “”working group before retirement age”” of the concepts of bond, belonging, and role on the basis of business life. Thus, it is aimed to clarify the background of the stage expressed as the related society. It has been decided to use the purposeful homogeneous sample for the group at the age of 60 and above, which constitutes the second working group and is labeled as “”the group working after retirement age””. Thus, it is aimed to reach participants who are 60 years old and over, working and with distinctive characteristics such as creating single-person households. It is aimed to reveal the position of the person, how he makes sense of himself in the related society and what are the factors that shape his current lives, through the participants in the “”group working after retirement age”” label. Thus, the reflections of the common perception, which will be evaluated through the first group, on the second group will also be analyzed. The third group of the study is labeled as ‘retired group’.The relevant group consists of men who are retired at the age of 60 and form one-person households. This group has a feature that will be difficult to reach, unlike other groups that we label as “”working before retirement age”” and “”working after retirement age””. For this reason, it was decided to use purposeful snowball sampling for the third group of the research. In this direction, in order to detect kodokushi cases as quickly as possible, the “”patrol system”” formations created by social service volunteers will be contacted. The patrol system is included in some apartment complexes with a high proportion of elderly people and they periodically control the elderly living alone (Ahlmark, 2017). Accordingly, it is aimed to reach the participants who form a “”retired group””.Through the third group, it is aimed to evaluate the longitudinal difference and the nature of the unrelated society mentioned earlier. Thus, the situation of old men living alone and the kodokushi phenomenon will be examined by revealing the effect of business life that shapes the selves, unlike previous studies that partially addressed the subject on the basis of family and demography.”

報告番号77

性的アイデンティティの形成とコミュニティ ――女性同性愛者のライフ・ストーリーに着目して
筑波大学大学院 呉丹

1目的  本稿の目的は,女性同性愛者のライフ・ストーリーに着目しながら,1990年代以降のセクシュアルマイノリティの性的アイデンティティがいかに形成しているのかを明らかにすることにある.この時期の日本社会においては,「性の多様化や差異の承認は,ジェンダー・セクシュアリティ研究や当事者運動において最も重要な要素の一つであった」(石井2018:232).この影響によって,当事者像には大きな変化が生じている.「ゲイ」や「レズビアン」などのカテゴリーを前に打ち出すことの代わりに,カテゴリーと距離を取りながら性的アイデンティティを語る当事者が浮かび上がってくる. 2方法  性的アイデンティティの形成を考察するさいに,本稿は歴史,文化などの資源に根ざした,時には政治的基盤となる当事者コミュニティとの相互作用に注目する.ケン・プラマー(1997)のセクシュアル・ストーリー論を参照し言い換えれば,本稿は①個人の「物語」だけでなく,②「解釈共同体」のなかである種集合性を持つ「物語」の形成にも注目する.非対称的ジェンダー,セクシュアリティ制度や脱構築的傾向などの構造的条件を踏まえ,2つの物語が形成する際の困難及び可能性を中心にして記述していく. 3結果  分析の結果は次のようにまとめることができる. 女性同性愛者の場合,家族内の再生産役割の引き受け,「女性」や「レズビアン」などカテゴリーに対する違和感が性的アイデンティティの形成及びコミュニティの参加に困難をもたらす. 可能性に関しては,学校,職場などの空間において,ジェンダー的非対称性や近年システムの改革によって,個人の性的アイデンティティ(段階的,特例的なものとみなされる)が形成可能である.この段階性と特例性を超えるため,コミュニティへの参加は動機付けられている.しかし,既存コミュニティの場合,とくに女性の性的アイデンティティの可能性を限定する限界もある.ゆえに,新たなコミュニティが作られてきた.ここではトランスジェンダーを含む多様な主体は,再構築されたカテゴリーのもとで(「L+」,「ちゃんとしている」など)集まっている. 4結論  女性同性愛者の性的アイデンティティ形成プロセスの検討から,女性同性愛者の場合,「私的領域」(個人)の多様な可能性を認識している姿勢が特徴的なものであり,「公的領域」(コミュニティ&より広い市民社会)への接続は選択しながら行っていることが示唆された.これはWU(2021)でも指摘されるように,公と私の間のバランスを重視する「半公共圏」(Ilse・Lenz 2013:284)が形成していると言える. 参考文献 石井由香里,2018,『トランスジェンダーと現代社会――多様化する性とあいまいな自己像をもつ人たちの生活世界』明石書店. Ilse,Lenz,2013,「フェミニズムにおける「私」と「公」のダイナミクス : ドイツと日本」古谷野郁, 左海陽子訳,落合恵美子編  『親密圏と公共圏の再編成 : アジア近代からの問い』京都大学学術出版会. Plummer,Ken,1995,Telling Sexual Stories. Power, Change and Social Worlds.London: Routledge(桜井厚・好井裕明・  小林多寿子,1998,『セクシュアル・ストーリーの時代―語りのポリティクス』新曜社.) WUDAN,2021,「『私は私』の社会的可能性:女性同性愛者コミュニティのグループディスカッションに着目して」『社会学ジャー  ナル』46:1-19.

報告番号78

身体的実存を等しく扱う法の構想 ――コーネルとセンの自由概念の統合をもとに
東京理科大学 長野慎一

【1.目的】  近年、性的少数者に属す当事者たちを中心に、SOGI(sexual orientation, gender identity)に関わる平等の法制化を求める活動が活発化している。他方、差別の是正に歯止めがかけられる社会状況も依然続いている。それを象徴的に示す出来事として、いわゆるLGBT理解増進法案が、自由民主党の抵抗により、国会で審議されることなく葬り去られた事案(2021年6月16日現在)を挙げることができる。この状況は、性(sex)と法の関係を再度問うべきことを要請している。  そこで、本報告では、性に関わる身体的経験の根源的様態とその自由な現われを等しく扱う法のあり方について、理論的な整理を行う。そのために、D.コーネルとA.センの各々の自由概念を架橋する。前者は、文化的に強制された身体像を練り直す営みに、後者は、実質的な選択の可能性の条件である「であること」と「なすこと」の組み合わせのあり様に焦点を当て、ともに自由に関わる重要な側面を論じている。だが、各々の議論領域の接点は必ずしも明らかではい。 【2. 方法】  M.メルロ=ポンティの身体論を媒介項として位置づけ、両理論の統合を試みる。メルロ=ポンティの身体論は、規範理論ではないものの、身体としての人間の根源を論じようとする点で、身体の何が法的に保護されるべきなのかを論じるにあたり、参照されるべき議論である。  第一に、メルロ=ポンティの身体論から(1)人間の主体性の根源に、生物学的事実には還元できない「身体的実存」がある点と(2)その実存の自己実現を左右する条件として社会的に生み出される「間身体性」がある点を指摘する。  第二に、コーネルの「イマジナリーな領域」とセンの「ケイパビリティ」が各々に措定する自由概念を、M.メルロ=ポンティがいう「身体的実存」をめぐる議論として再解釈すべきことを示したうえで、両者の理論の統合を図る。   【3.結果】  メルロ=ポンティを介したコーネルとセンの理論の統合からは、(1)性に関わる自由とは、身体的実存として、性と自己の生の関係を想像し、これを実際の行動へと結びつける活動の自由であること、(2)かかる自由の平等が法的に保障されるべきことが導き出せる。 【4.結論】  この構想に照らすとき、日本社会において、性的少数者は、身体的実存としての自由を不当に制限されており、その扱いは不法状態にあると言うべきである。 【文献】 Cornell, D., 1995, The Imaginary Domain: Abortion, Pornography and Sexual Harassment, New York& London: Routledge. Merleau-Ponty, M., 1945, Phénoménologie de la perception, Paris: Gallimard. ――――, 1964, Le visible et l’invisible, Paris: Gallimard. 長野慎一,2021, 「異性愛主義的秩序における身体の再帰性――バトラーによるメルロ=ポンティの交差配列論の受容の検討を中心に」『現代社会学理論研究』15: 124-36 Sen, A., 1992, Inequality Reexamined, Oxford: Oxford University Press.

報告番号79

「腐女子」とはいかなる人びとか――女性のBL読者/非読者へのWEB調査から(1)
東洋学園大学、淑徳大学、亀田医療大学、東京医療学院大学、千葉中央看護専門学校、大宮医師会看護専門学校 佐藤麻衣

【1.目的】 日本の社会学界においては、特に1990年代以降、さまざまな「BL」研究および「腐女子」研究が行われてきた。しかし、その研究の多くはドキュメント分析やインタビューといった質的な研究によって占められており、腐女子の実態を明らかにするような量的調査はごく少数である。それらの量的調査の成果は十分に評価されて良いと考えるが、その一方で以下のような問題点が見受けられる。 1)既存の腐女子に関する量的調査の多くは大学生を対象としたものであり、一般の人びとを対象としたものはほとんど見られないこと。 2)大学生以外を対象とした調査もわずかながら見られるが、それらの対象者もやはり10〜20代といった若年層に偏っており、「腐女子」の標本数は50名前後とごく少数であること。 1)・2)の結果として、「一般的な腐女子」の年齢・学歴・職業といった基本属性の分布は未だよくわかっていない。こうした既存の研究の問題点を踏まえ、本報告においては、調査会社のモニタに対して行った2本の調査を元に、腐女子の基本属性の分布、およびBLの楽しみ方やジェンダー観などについて報告する。 【2.方法】 使用するデータは以下のとおりである。 予備調査:2020年2月21日〜2月25日、有効回答9348票 本調査:2021年2月19日〜2月24日、有効回答1524票 いずれも、全国の20〜59歳の女性を対象としたモニタ型WEB調査。本調査では、調査会社のモニタに対しあらかじめスクリーニングを行うことで、腐女子とそうでない人、それぞれ800人を目標にサンプリングを行った。 【3.結果】 まず、予備調査の結果から、腐女子の出現率は質問の仕方によって4〜15%ほどの間で変化することがわかった。腐女子はやはり若年層に多く見られるが、40-50代であっても3-10%ほどは腐女子とみなして良い人びとがいることがわかった。なお、「BL」や「腐女子」という言葉を知らない人は25%ほどであった。 次に、本調査の結果から、対象者を「腐女子」「BL接触経験者」「BL非接触者」の3群に分けて分析をした結果、腐女子は他の群に比べ、年齢がやや低いこと、未婚者が多いこと、子どもがいない人が多いこと、ヘテロセクシュアル以外のセクシュアル・アイデンティティをもつ人が多いことが示された。また、腐女子は他の2群と比較し固定的なジェンダー観が弱いことが示された。なお、BL接触経験者と非接触者との間ではジェンダー観に違いは見られなかった。さらに、BLを楽しむ媒体としてマンガを好む人は9割にのぼるのに対し、小説・映画・ドラマ・アニメを好む人は3割、ゲームを好む人は1割であった。 【4.結論】 本報告では、今までほとんど明らかにされてこなかった「腐女子」の基本データを明らかにした。すでにセクシュアル・マイノリティ男性を対象とした同じ調査も行っているため、今後は、そちらのデータも合わせて分析を進め、最終的には、BLを楽しむ人びととセクシュアル・マイノリティが共に歩んでいける可能性を探っていく予定である。 *本報告は科研費19H04388「BL等の表現の国際的な広がりと、各国での現実のLGBTとの社会的関係の国際比較」(研究代表者:藤本由香里)の研究成果の一部である。

報告番号80

性的マイノリティと共に生きようとしているのは誰か ――女性のBL読者/非読者へのWEB調査から(2)
明治学院大学 石田仁

【1.目的】  女性を主たる想定読者として描かれてきた男性同士の恋愛(ボーイズラブ、以下BL)は、長い歴史と大きな市場を日本で得てきた。90年代以降、BLにおける“男性同性愛”の表象をめぐる議論はたびたび起こり、同性愛嫌悪的な描写が批判されたり、批判の妥当性が議論されたりした*1。しかし2000年代中盤より、主人公の少なくとも一方がゲイであることを明示したBL作品が増え*2、社会に存する異性愛主義や性別二元制を乗り越えようとする主人公たちを描いたBL作品も研究の過程で注目されだした*3。いくつかの動向から、日本におけるBLとゲイ(あるいは性的マイノリティ)の距離はかつてより近づいたとされる*4が、研究者や識者にとどまらないBL読者が、ゲイ男性や性的マイノリティをどう受容しているのか、あるいは逆に、ゲイ男性がBLやジェンダー平等をどうとらえているかについての十分な数量的研究は存在しない。 【2.方法】  上記の問題を克服するために、WEB調査という方法論的限界は踏まえつつも、次の2つの調査を企画・実施した*5。調査1:1社のモニタの中で女性として登録している20-59歳を対象としたクローズド型WEB調査。調査2:ゲイ・バイセクシュアル男性用マッチングアプリのユーザーを対象としたオープン型WEB調査。  本報告の内容となる調査1は、スクリーニングの段階でBL読者か否かをたずね、20-30代/40-50代、読者/非読者の4セルについて、有効回答数=各400ssを目標として割り付けた。2021年2月に実施し、有効回答総数は1524票だった。調査項目には、性的マイノリティの受容、同性愛・ゲイのイメージ、ジェンダー平等観、社会運動観、基本属性等の33項目が含まれる。 【3.結果】  BL読者/非読者、あるいは他の属性・意識別に見た場合の性的マイノリティの受容の差を調べた。調査では「同性の恋人と一生を共にすること」「性別を変えた人と一生を共にすること」を「子どもにしてほしくない生き方」だと捉えているか否かについて尋ねており、この項目を性的マイノリティと共に生きていこうとする「性的マイノリティの受容」変数とみなし、二変量の分析を行った。「BL読者は非読者に比べ、性的マイノリティを受容している」等の仮説が検証された。 【4.結論】  本報告は調査1の分析にとどまるが、後日行う調査2の分析と併せて、BL(コンテンツ・担い手)と性的マイノリティとの接近性を明らかにする研究*6の一角に位置づけられる。 <註> *1 佐藤雅樹1992『CHIISIR』(20)に続く「やおい論争」;溝口彰子2000『QUEER JAPAN』(2);石田仁2007『ユリイカ』39(16);堀あきこ2010好井編『セクシュアリティの多様性と排除』明石書店;堀あきこ2012『ユリイカ』44(15)ほか. *2 藤本純子2007『ユリイカ』39(16). *3 溝口彰子2015『BL進化論』太田出版. *4 溝口 前掲;前川直哉2020堀あきこ・守如子編『BLの教科書』有斐閣. *5 科研費基盤研究(B) 19H04388「BL等の表現の国際的な広がりと、各国での現実のLGBTとの社会的関係の国際比較」(研究代表者:藤本由香里・明治大学)、国内調査班(石田仁・佐藤麻衣). *6 例えばJ. ウェルカー編『BLが開く扉』青土社.

報告番号81

新型コロナウィルス感染症のLGBTコミュニティへの影響――自助団体/支援団体の資料の分析から
奈良女子大学研究院 三部倫子

1.目的  新型コロナウィルスの感染拡大は、日本社会、とくに「マイノリティ」とされる人々に甚大な影響を及ぼしている。本研究は、未だ収束の兆しのみえない新型コロナウィルス感染症が、LGBTコミュニティにもたらした影響の解明を目指している。  国内での感染拡大から1年以上経過し、生物医学的見地に留まらない、パンデミックの社会的影響が論じられている。危機への対応は概して性的指向、性自認、性表現(SOGIE)を含んだ交差性(intersectionality)が考慮されない形でなされるため、LGBTなどの性的マイリティはより一層脆弱な立場に置かれていると推察される。  異性愛の家父長的家族からの解放がゲイ・アイデンティを醸成したように、地縁・血縁に基づかないLGBTコミュニティは、感染症拡大以前よりLGBTにとっての生命線となってきた。しかし、感染症対策のもと要請される移動制限、LGBT支援団体や自助団体の活動場となる公共施設の利用制限、LGBTを客層とする飲食店の営業「自粛」は、対面的相互行為によりアイデンティティを承認しあうLGBTコミュニティの存続を危うくしているとみられる。  新型コロナウィルス感染症は、LGBTコミュニティにどのような影響を及ぼしているのだろうか。パンデミックの只中では対面を前提とする従前の調査の実施は困難であり、緊急の課題が学術的に集約されていない。 2.方法  感染症拡大のなかさまざまな団体がオンラインへの活動に移行し、報告書、インターネット上の研究会や自助団体/支援団体によるウェブ調査等が実施されてきた。ネット上の資料は図書館などに収録されるものではないため、研究者が体系的に収集しなければ散逸し、アフターコロナの調査や運動に向けた経験知の共有を難しくする。本研究ではLGBTの自助団体ないしは支援団体が発行したネット上の資料を網羅的に収集し、各団体がどのような課題にみまわれ、どのような対処をしてきたかを分析し、コミュニティの経験知を共有知とすることを目指す。  具体的には報告者がこれまでの調査で培ってきたネットワークにて資料の提供を依頼したり、SNSを通してLGBT自助団体/支援団体に研究協力を呼びかけ、紹介をうけた資料を収集、分析をする。公開され個人で入手可能なものであれば非匿名とし、されていない物は協力団体と相談の上、匿名か非匿名にて扱う。 3.結果と結論  いまだ収集の途上であるため、具体的な結果と結論は報告にて詳述するが、現段階では以下のような見通しを立てている。諸団体が見舞われた困難として1)対面活動の中断、2)感染拡大地域とそうではない地域間の地域差(匿名性を担保しやすい都市部での対面活動の再開vs匿名性が担保されにくい地方での活動中断の継続)、3)感染拡大防止(クラスターを追うため)のための個人情報収集のジレンマ(セクシュアリティの匿名性を保つvs保健所などから要請される個人情報の把握)、対処として1)遠隔イベントの工夫(オンライン会議システムの活用)、2)参加者の状況を把握するためのウェブ当事者調査の実施があげられる。団体による調査からは、LGBT内部の格差(異性愛家族と同居しているのか否か、セクシュアリティの開示状況、経済状況、性自認と性的指向などとの交差性)の一端を垣間見ることができると考えられる。

報告番号82

性的少数者と生活困難――聞き取り調査からの考察
早稲田大学 志田哲之

【1.目的】  性的少数者が抱える問題として、これまで主に指摘されてきたのは、カップル関係の制度保障の欠如や就業、いじめ・自殺などの生きづらさに関する事柄であった。同時に「LGBT」の名を冠して展開される様々な動きは、概して華やかで経済的豊かな印象を与えがちである。しかしながら性的少数者の経済的な生活困難はかつてより指摘され、また日本ではおよそ10年間これに取り組む当事者団体がいくつか発足したが、この問題についての国内の研究領域ではあまり取り上げられることがなく今日に至っている(志田,ibid.,2020)。当事者団体のひとつである「LGBTハウジングファーストを考える会・東京」の取り組みを考察した金井は、「セクシュアルマイノリティ当事者が生活困窮状態になるとき、特有の事情が背景にあることが少なくない」と論じている(金井,2021)。その一方で性的少数者の中でも経済的な成功を収めている者もいることから、性的少数者という経験と生活困窮には関係はないとする当事者からの声もある。とはいえ、具体的なデータに基づく議論は未だ行われていないというのが実情だといえる。  本報告では英語圏での研究などを参照しつつ、日本における性的少数者の生活困難についての聞き取り調査を実施し、実態を明らかにしたい。 【2.方法】  調査時において経済的生活困難を抱える、ないしはこれまで生活困難を経験した性的少数者数名を対象者とし、半構造化聞き取り調査を実施した。  いじめや差別の経験、定位家族・友人・パートナー等の対人関係、居住形態、学歴、就業や所得、健康状態、社会的支援との関わりについて主に質問を行った。録音した音声データを書き起こし、秘匿化したのち対象者から承諾を得た上で分析、考察を行った。 【3.結果と結論】 「性的少数者」であるから生活困難に陥るといった、直接的な図式を描くことは困難であった。様々なきっかけや経緯があり、それらが複雑に絡み合って生活困難という状況が生成されると考えられた。なかでも定位家族との関係やパートナー関係といった対人関係や、居住形態、ジェンダーといった事柄が大きく関与しているように見受けられた。同時に個人として生きていけるような制度的保障が生活困難への抑止効果をもたらすことも考えられた。  本研究をさらに進展させるためには、より多くの事例を集める必要があるだろう。同時に、生活困難に陥ることのない当事者や、生活困難にある/あった非性的少数者の事例との比較検討も視野に入れる必要があると考える。 【参考文献】 金井聡,2021「LGBTハウジングファースト −セクシュアル・マイノリティの貧困という視点から」『賃金と社会保障』,No.1769・70,賃社編集室,旬報社,pp.10-17 志田哲之,2020,「性的少数者の貧困研究についての考察 -海外研究から-」,第93回日本社会学会大会

報告番号83

大学生の生活と意識2020(1)――親学歴は子の大学ランクに影響を与えるのか?
同志社大学 妹尾麻美

【1.目的】  本研究の目的は、第1に青少年研究会が2020年秋に実施した調査票調査の概要を説明すること、第2にそのデータを用いて入学難易度の規定要因を確認すること、第3にこの入学難易度変数とコロナウィルス禍に関する意識との分析結果を紹介することにある。  青少年研究会は、2010年に若者の文化活動、メディア利用、人間関係(友人、家族、恋愛)、自己および社会意識に関する大学生に調査票調査を行った。これらの意識を再度検証するため、改めて本研究会によって2020年秋に調査が実施された。本報告は、この概要を説明する。  2000年代以降、若者文化研究のなかでは、若者文化を消費文化とみなし階層性を欠落させた研究に批判が寄せられてきた(中西・高山編2009)。18歳の半数以上が大学に進学し、大学生が若者文化の一端を担っているという前提を置くならば、大学生内部における階層差について確認しておくことは重要な課題だと考えられる。その課題の1つとして、2020年時点における親学歴と入学難易度の関連について確認する。また、本調査はコロナ禍に実施されたことから、関連する意識についても若干ではあるものの確認しておきたい。 【2.方法】  本研究会は、2020年秋に国公立・私立19大学にて社会学系授業を受講している大学生を対象に調査票調査を実施した(オンラインアンケートフォームを用いた集合式調査(一部質問紙を配布)、有効回答数1059)。有効回答数のうち、大学所在地は関東45.3%、関西24.9%、その他29.8%、大学種別は私立75.9%、国公立24.1%となっている。本報告はこのデータを分析に用いる。なお、2010年に実施した先述のデータの分析結果も参考として示す。19大学の入学難易度は河合塾Kai-Net大学検索システムの入試難易度を用いた。平均53.89、標準偏差7.37である。詳細は、当日報告を行う。 【3.結果】  入学難易度を従属変数とし、親の教育年数を独立変数に投入した重回帰分析の結果、決定係数は小さいものの、父親の教育年数、母親の教育年数が高ければ、子が入学難易度の高い大学に進学する傾向にある。とりわけ、父親の学歴の効果よりも母親の学歴の効果が大きい。親の教育年数によって、進学する大学の選抜性に差があると考えられる。加えて、こうした入学難易度によってオンライン授業またはリモートワークへの親和性に違いが確認された。 【4.結論】  分析結果から、大学生内部においてもペアレントクラシー(ブラウン1995=2005)の浸透、定着が想定される。こうした基礎的な分析をふまえた上で、入学難易度とその他の要因が社会意識にどのような影響を及ぼしているのか、どのような若者文化を形成しているのか、続く報告で確認していく。 参考文献 中西新太郎・高山智樹編、2009、『ノンエリート青年の社会空間―働くこと、生きること、「大人になる」ということ』大月書店。 フィリップ・ブラウン、1995=2005、「文化資本と社会的排除」A・H・ハルゼー他編(住田ほか編訳)『教育社会学 第三のソリューション』九州大学出版会、597-622。

報告番号84

大学生の生活と意識2020(2)――再帰性はアイデンティティ資本になりうるのか?
大妻女子大学 牧野智和

【1.目的】 自己に対するリフレクシブな態度が、現代における適応的な自己のモードであるという見方について、アンソニー・ギデンズら後期近代論者が主張していることはよく知られているが、アンソニー・エリオットなどの自己論の専門家もまた、今日に至るまで概ねその見方を採用している。だが、そのようなモードは実際のところ、現代を生きる人々にとってどの程度支配的なものであるのだろうか。また、そのようなモードは現代人にとってどのような効果をもたらしているのだろうか。本報告ではジェームズ・コテの「アイデンティティ資本」についての議論を分析枠組みとして、自己に対するリフレクシブな態度が、自己意識のあり方にどう関係しているのかを検討する。 【2.方法】 分析は、2020年9~11月に「青少年研究会」が行った全国19大学における大学生調査のデータを用いる(オンラインアンケートフォームを用いた集合式調査(一部質問紙を配布)、有効回答数1059)。自己に対するリフレクシブな態度(以下、これをreflexivity=再帰性として表現する)に関する質問への単純集計結果は、「自分のふるまい方が正しいかどうかをふりかえることがある」(95%)、「これからの社会で生きていくために必要とされる力が、自分にあるかどうか心配になる」(88%)、「自分自身についてじっくり考えることがある」(83%)、「今日は当たり前であることが、明日もそうだとは限らないと感じる」(81%)、「大切なことを決めるときに、自分の中に複数の基準があって困ることがある」(80%)といずれも高い肯定回答率となっている。これらの項目の主成分得点を算出し、それが自己肯定感、将来展望、自己変革志向、同調志向といった自己意識の諸側面にどう関係していくのかを、他の変数を統制したうえで重回帰分析を行う。 【3.結果】 自己肯定感については、自己の再帰性は負の効果をもたらす。「今のままの自分でいいと思う」という項目に対しても、同様に負の効果がみられる。その意味で、自己に対するリフレクシブな態度は、現代の若者にとって単純に適応的な効果をもたらすものではないと考えられる。だが、「他人とは違った、自分らしさを出すことが好きだ」「自分の外見が変われば、人生も変わると思う」「なりたい自分になるために努力することが大切だ」「これからの人生で大事にしたいことが定まっている」といった項目については、自己の再帰性は正の効果をもたらしている。 【4.結論】 自己の再帰性は、本分析において言える限りでは、自己意識のコンサマトリーな状態に対してはプラスの効果をもたらさない。その意味で、短期的には適応をもたらす資本にはなっていないと考えられる。だが一方で、自分自身がこれから変わっていくこと、自分自身が将来何をしていくかということについては、プラスの効果をもたらしているといえる。その意味では、自己の再帰性は、将来において適応をもたらす資本にはなっていると考えられる。ただ、今回用いたデータは大学生を対象とした調査によるものであったため、これが現代人全般に対しても同様にいえるのかは、引き続き検討する必要がある。

報告番号85

大学生の生活と意識2020(3)――「努力」のコスパを重視するのはどのような大学生か?
茨城大学 寺地幹人

【1.目的】  本報告の目的は、「努力」に対する考え方に注目して、近年の大学生の状況を考察することである。近年の日本の大学においては、3ポリシーの明確化とそれに基づいた成果の検証などが一層求められている。そうした状況において、大学で学ぶことに実利的な価値や短期的な成果を求める学生がいるならば、「努力」に対するそのコスパ重視の価値観は、親和的である(ただし、「努力」の対象が学びに限らないことは、例えば本田由紀の先行研究で論じられており、その点は留意が必要である)。前述のような近年の大学への要請に対して対抗的な価値観に親和的な「努力」を支持する学生や「努力」から降りる学生たちも含め、「努力」に対する再帰性という観点から、近年の大学生の状況をみていく。 【2.方法】  分析には、青少年研究会が2020年9~11月に全国19の大学で行った、オンラインサーベイ(一部質問紙使用。有効回答数1059)のデータを用いる。本報告では、「物事に真剣に取り組まないのはカッコ悪い」(Q44-4)と「一生懸命に物事に取り組んでも成果に結びつかないと意味がない」(Q44-5)という2つの質問項目に着目する。回答選択肢はいずれも4件法だが、それぞれ肯定回答と否定回答に分けて両者のクロス分析を行った結果をもとに、対象者を3つのタイプ(【3.結果】に記載)に区分し、本人属性や各種意識項目などとの関連を分析する。 【3.結果】  Q44-4否定×Q44-5肯定・否定を「非努力志向」(17.0%)、Q44-4肯定×q44-5否定を「旧努力志向」(34.9%)、Q44-4肯定×q44-5肯定を「新努力志向」(48.1%)と定義した。このタイプと、性別、大学難易度(2区分)、入試形態(一般選抜か否か)、父・母学歴(短大以上か未満か)、暮らし向きといった属性項目のクロス分析の結果では、性別のみ統計的に有意な関連(χ2乗検定5%)を確認できた。残差分析の結果をみると、「旧努力志向」が女性で有意に多い傾向にあった。更に、3つのタイプと性別の関連を、先に挙げたほかの変数で統制した3重クロス分析の結果では、低入学難易度の大学生、親が短大卒以上の大学生において、3つのタイプと性別との間に統計的に有意な関連(χ2乗検定5%)を確認できた。残差分析の結果をみると、「旧努力志向」が低入学難易度の女性で、「新努力志向」が低入学難易度の男性で、有意に多い傾向にあった。また、「非努力志向」が高学歴の父ないし母をもつ男性で、「旧努力志向」が高学歴の父をもつ女性で、有意に多い傾向にあった。 【4.結論】  今回調査した大学生全体としては、「努力」のコスパ認識にジェンダーが関連していたものの、そのなかでも特定の大学生においてその関連が確認できることがわかった。すなわち、「努力」に対する再帰性のあり方へのジェンダーの効果には、大学の入学難易度や親の学歴といった変数を考慮する必要がある。当日の報告では、ここに挙げた以外の変数(大学所在地など)との関連も踏まえ、かつ各変数間の効果の統制を行った多変量解析や、3つのタイプと各種社会意識・自己意識などとの関係についても、説明する予定である。

報告番号86

大学生の生活と意識2020(4)――趣味は情報選択に違いをもたらすのか?
桃山学院大学 木島由晶

【1.目的】 本報告の目的は、2020年9~11月に全国の大学生を対象とした調査(主としてオンラインアンケートフォームを用いた集合式調査:有効回答数1059)の結果に基づき、若者たちの文化的な趣味の実態を明らかにすることである。とくに、若者が趣味を再帰的に構築していくさいの情報選択の違いに注目する。また、先行研究からは「おたく/オタク」に対する否定的なイメージが薄れてアニメやゲームに接することがありふれた趣味になったこと、ファストファッションの浸透にともない他者との差異化のためにお洒落をする感覚が薄れたことが指摘されているが、こうした意味での若者文化のフラット化は今日の大学生に対してどの程度当てはまるのか。本報告ではファッション、音楽、ACG(アニメ・コミック・ゲーム)に関心のある3つのグループを比較することで、以上の問いを検討したい。 【2.方法】 目的変数は、全12のポピュラー文化ジャンルのうち、最も関心のあるものとして、音楽(327名)、ファッション(153名)、アニメ・マンガ・ゲーム(223名)をそれぞれ選んだかどうかである。説明変数には、音楽を知る際に重んじる情報(ヒットチャート・AIのおすすめ・ネットの「口コミ」など)と、大切なことを決める際の判断材料に関する質問項目を用いる。他には、好きなアーティストの自由記述、友人の数、一日におけるLINEの送信数、そして趣味がアイデンティティや他者との差異化・卓越化に影響するのかどうかを尋ねた項目などとの関連も報告する予定である。 【3.結果】 おおむね先行研究を支持する結果となった。すなわち、関心のある趣味と友人数との間には強い影響関係はみられず、またファッションが自分らしさを表すものだとしても、他者との差異化が強く意識されているわけではないと解釈しうる結果がえられた。そして趣味ごとの特徴の違い、すなわちアニメが数ある趣味の中で自律した傾向を持つのに対して、音楽は趣味としての特徴が薄い(誰もが「好き」と気軽に口にするぶん特徴がみえにくい)といった先行研究に関しても、それを間接的に支持する結果がえられた。さらに、新しい音楽を知る際に重んじる情報として、ファッションに関心のある人は口コミ、音楽の場合は自分の好きなアーティスト、ACGの場合は友人の情報が重んじられており、それぞれの趣味を再帰的に構築していくさいの判断基準に差があることが示唆された。 【4.結論】 得られた結果から解釈できるのは、今日の大学生一般において、音楽に関心をもつことは自分の「推し」を応援することに近い意味をもっていること、第二に、音楽に関心をもつことはそれだけでは友人形成の契機になりにくいが、アニメの場合はその効果が期待しうるということである。もっとも本分析では、直接的に趣味と情報選択との関係を尋ねたわけではないので、結果の解釈には様々な留保が必要である。また「趣味」といっても、本調査ではそれらを網羅的にあつかったわけではなく、あくまでコンテンツの消費にかぎった意味での趣味である点にも十分に注意すべきだろう。

報告番号87

大学生の生活と意識2020(5)――「メディアの再帰的利用」を行うのは、どんな大学生か?
清泉女学院短期大学 二方龍紀

【1.目的】 本報告の目的は、SNSにおける「再帰的利用」を行う学生の傾向を諸属性や生活意識・行動の分析を通じて明らかにすることである。青少年研究会では、1992年以降、10年ごとに青少年の生活と意識に関する調査を行っている。2012年調査以降、メディア分野では、青少年の間で、SNS利用が本格的に普及した(利用率2012年76.0 %(青少年研究会調査若年票)、2020年94.5%(青少年研究会大学生調査))。こうしたSNSを前提とした情報環境が普及する中で、SNSが「自己の行動を他者の視線から見て、その振る舞いを修正していく」ためのツールになっている現象が見られる。ここでは、2020年に行われた大学生対象の調査を分析し、こうしたSNS利用を意識的に行う学生には、どのような特徴が見られるのかを明らかにしたい。 【2.方法】 青少年研究会が、2020年9~11月に、全国の19大学で実施した調査(オンラインアンケートフォームを用いた集合式調査。一部質問紙を配布)をもとに分析を行う。具体的には、「SNS上での反応を意識して行動する」とした学生にどのような特徴が見られるのかを、諸属性や「生活意識・行動」「文化行動」「メディア利用」「友人関係」「恋愛・家族意識」「自己意識」「社会意識」などの側面から分析する。 【3.結果】 まず、「SNS上での反応を意識して行動する」とした学生は、全体の47.5%だった。こうした学生は、入学難易度が比較的高い大学の学生に多い傾向が見られた。また、「友人関係意識」においては、「誰とでも仲良くなれる」という一方、「周りからの視線への不安」も見られた。「恋愛・家族意識」については、コミュニケーション力の高さも見られる一方、「家族と仲が悪いと周りから思われたくない」という意識も見られた。「メディア利用」に関しても、SNS上での情報の管理や演出は特に意識しているが、「メディアの使い過ぎ」には不安も感じている。また、「自己意識」に関しては、自分を振り返ったり、自分自身について、じっくり考えるという特徴が見られた。「社会意識」については、他者への信頼が薄く、政治や将来への関心が薄いという傾向も見られた。 【4.結論】 現代の大学生は、スマホやSNSによるコミュニケーションがごく自然な情報環境となった世界の中で育ってきている。こうした世界では、SNSを通じて、他者の視線がごく自然に個人の内面や行動に反映する。そうした情報環境を当然のものとして活用しながら、不安や悩みを抱えている部分もある。当日は、こうした学生の生活や意識、行動について、諸属性との関係やデータ分析の図表を示し、報告する。

報告番号88

大学生の生活と意識2020(6)――再帰的な友人関係は「濃密」で「希薄」か?
昭和女子大学 小川豊武

【1.目的】  本報告の目的は,1990~2000年代に盛んに議論された、若者の友人関係希薄化論について、全国の大学生を対象とした最新の質問紙調査のデータを用いて再検証することにある。2018年7月、NHKの「クローズアップ現代+」で「“つながり孤独” 若者の心を探って…」が放送された。SNS上で多くの人たちとつながっているにもかかわらず、孤独を感じている若者たちの実態を取材したものである。同様の指摘は、「Alone Together」(シェリー・タークル)など海外の若者研究にも見られる。日本では携帯電話の普及が進んだ1990年代から、若者の友人関係の希薄化を指摘する議論が数多くなされてきた。社会学やその隣接領域における実証研究ではおおむね希薄化論は反証され、若者の友人関係の選択化、多元化、濃密化といった指摘がされてきた。こうした中で「つながり孤独」という指摘は、一見すると、希薄化論の再興のようにも思えるが、「友人関係の希薄化」が“年長者の認識=希薄化/若者の認識≠希薄化”という事態であったのに対し、「つながり孤独」は“年長者の認識=濃密化/若者の認識=希薄化”という事態であり、新たな現象が生じているといえる。印象論と批判されてきた希薄化論から約20年の時を経て、若者の友人関係はついに当人たちの認識のレベルで「希薄化」してきたのだろうか。 【2.方法】  「青少年研究会」が2020年に実施した全国大学生調査(19大学対象、WEB調査主体、有効回答数1059)を用いて、若者の友人関係およびそれに関連した質問項目の計量的な分析を行う。具体的には,友人数や連絡頻度等を尋ねた事実質問、友人関係に関する態度等を尋ねた意識質問の分析に加えて、親しい友人との関係について尋ねるネームジェネレータ式質問を用いた分析も行い、現代の若者の友人関係における「希薄化」の内実について明らかにする。 【3.分析結果】  現時点での分析結果として、第一に、友人数の質問において「親友」「仲のよい友だち」に変化が見られなかったのに対し、「知り合い程度の友だち」が増加していることが明らかになった。第二に、最も親しい友人の大半が大学時代ではなく、高校までに知り合った同じ年の友人であることが分かった。第三に、このような結果の一方、最も親しい友人との間においても、自己や内面の開示を行わない、年長者から見れば「希薄」にも見えるような関係が観察できた。第四に、対人関係スキルに苦手意識を持っている者は友人関係に不安や孤独を抱いていることが明らかになった。このような結果から見えてくるのは、「濃密化した知り合い程度の友人たち」のマネジメントに困難を抱え、そこからの退避場所のようなものとして「希薄な親友」を求める若者たちの姿である。 【4.結論】  以上のような、現代の若者の「濃密」で「希薄」な友人関係が今後どのような帰結を生むのかについては更なる研究が必要となる。考えられることは、年長世代と同じように、「親友とは互いに自己や内面の開示を行うべきである」という規範意識を持っている若者にとっては、このような友人関係は時として「つながり孤独」のような耐え難いものになりうるということである。次に求められる作業は、論者の理論構築のために用いる概念としてではなく、当事者である若者が日常生活の様々な実践のために用いる概念としての「友人」の分析であろう。

報告番号89

大学生の生活と意識2020(7)――友人関係の構造と恋愛交際経歴は関連するか?
日本大学 久保田裕之

【1,目的】 近代が結婚を様々な慣習・制度から解放し配偶者選択を個人の恋愛的な選択に委ねたことは、「私は本当にこの人を好きなのか」といった自らの内面に問いかけるほかない選択を通じて「恋愛的自己」(Illouz 2012)を生み出した。同時に、恋愛関係の中に友人関係的な特質が求められるなど友人関係がますます重要になるとともに、とりわけ同性の友人は、恋愛に関する相談や忠告を通じてどのような恋愛や交際相手がdesirable(望ましいか/欲望可能か)の基準を提供し、恋愛的自己のモニタリングに対して重要な役割を持つようになったことが指摘されているChambers 2006)。 そこで、本報告の目的は、現代の大学生における恋愛関係と友人関係がどのように関連しているのかを、ジェンダーの観点から検討することである。とりわけ、友人数のみならず友人関係の持ち方の構造と、現時点での恋愛交際の有無のみならず恋愛交際人数や最長交際期間の長さといった恋愛交際経歴から得られる恋愛関係構築のパタンに着目して検討した。 【2,方法】  2020年9月~11月に、青少年研究会が全国の19大学で実施した調査(オンラインアンケートフォーム/一部質問紙:有効回答数1059)を用いた分析を行う。具体的には、親の教育経済階層や、本人の居住形態・アルバイト経験・大学の偏差値帯など恋愛交際に関係しそうな変数を統制したうえで、同性および異性の「親友」「親しい友人(友人)」と「知り合い程度の友人(知人)」の人数が、どのように恋愛交際人数および最長恋愛交際期間の長さと関連するのかを検討した。 【3,結果】 全体として友人数が多いほど恋愛交際人数が増える傾向にあったが、最長恋愛交際期間とは関連を持たなかった。男性では、異性の親友・友人が多いほど恋愛交際人数が増えるが、同性の親友の数は逆に恋愛交際人数を減らしていた。男性にとっての友人関係は、最長恋愛交際期間とは関連しなかった。女性では、異性の親友・友人・知人が多いほど恋愛交際人数は増えるが、同性の親友・友人が多いことは逆に恋愛交際人数を減らしていた。女性にとっての友人関係も、最長恋愛交際期間とはほとんど関連していなかった。 【4.結論】 分析からは、同性の親しい友人関係が恋人とうまくやるための相談(愚痴)やアドバイスを提供し恋愛関係を長続きさせる側面はみられなかった一方で、友人関係が恋愛関係を常に監視・相対化し、コンフリクトや相性の悪さや倦怠期などに機敏に反応して、恋愛関係を終結させ次の恋愛関係へと移行させる側面が観察されるとともに、異性の親しい友人関係が恋人予備軍として機能していることが示唆された。当日は、詳細なデータ分析のレジュメを配布して報告する。

報告番号90

日本における中国人留学生の曖昧な生きづらさ――「沈黙」を選ぶ理由
東北大学大学院 陳黙

1.目的 本研究の目的は、日本における中国人留学生を例にとって、日常的な「被傷」経験の事例を取り出し、その経験についてクレーム申し立てしない理由を検討することである。 2.方法 筆者のアルバイト先などからの機縁法により、インタビューを実施した。すべての協力者に対して共通の設問項目と質問文を準備したが、実際の聴取場面では、自由な会話に近づくよう配慮し工夫した(半構造化インタビュー)。 被傷経験については、生活の各場面での「嫌な経験」を切り口に話題をひろげた。最初から特定カテゴリーの浮上をねらって接近すれば、曖昧な「被傷」経験の多くを見逃してしまう危険がある。実際、当事者たちが傷つけられたと語り出す内容は、多岐にわたっていた。本研究にいう「被傷」経験とは、どれか特定の内容をもった経験を特定するためのものというよりも、こうした語りにできる限り広く網を掛けようとするためのものである。 得られた回答については、グラウンデッド・セオリー・アプローチを参考にしてそのデータ間の相互比較により、類型化を試みた。いわゆるオープン・コーディングに近いが、先行研究からの解釈(いわゆる理論的解釈)もおこなっている。 3.結果 複数の場面間の被傷経験を比較検討しながら考察し、クレームとして申し立てをしない理由・事情について検討する。  アルバイト場面・大学場面・公共場面における沈黙を比較すると、異議を申し立てない理由については、次のことが言えよう。  1)アルバイト場面に特有な沈黙は客との文化的齟齬や「他より弱い従業員」への配慮などである。  2)大学場面に特有な沈黙は学校の相談機関への失望である。  3)公共空間での沈黙はアルバイト場面および大学場面と重複している。  4)複数の場面・沈黙に、型として共通すると思われるものは、日本語表現の難しさ、コミュニケーション・チャネルの不在、結果に関する不安などである。 4.結論 本研究において得られたのは「被傷」経験の普遍性である。アルバイト先にしても、学校にしても、ほかの公共の場面にしても、広く「被傷」が経験されている。言い換えれば、自文化と異文化の挟みの中で生きている中国人留学生にとって、生きづらさがかなり広く普遍性を持っているとも言えるだろう。しかし、その経験について、当事者の中でクレーム申し立てをする人はわずかである。多くの人は「沈黙」を選んだ。大多数の人に対してごく当たり前のこと、とくに「常識」的な「問題」として考えられるとき、クレーム申し立てをしようとしても容易ではない。さらに、クレームを申し立てるための言葉があっても、必ずしもクレーム申し立てを行うわけではない。いかに申し立てるか、どんな言葉を使うか、どんな状況なら語れるか、誰とともに言うか、などの問題が現れる。逆に、語れないなら、その原因は何だろうか。これらが、クレームに関する問題群である。本研究によって、各カテゴリーを分析し、「問題」として分節化することも困難な中で、「沈黙」を選んだ当事者たちが他者に「問題」を表明しない原因を明らかにする。

報告番号91

台湾総統選におけるストラテジーとしての多言語の使い分け――スピーチライターの役割と選挙演説を中心に 
関西学院大学大学院 齋藤幸世

台湾社会では,1990年以降,主に台湾男性との婚姻のため主に東南アジア7カ国(ベトナム、タイ、インドネシア、マレーシア、ミャンマー、カンボジア、フィリピン)や中華圏等からの「新住民」が急増した.それに伴い,その東南アジア7カ国の言語が,台湾の従来のエスニック・グループである本省人(台語・閩南語)・外省人(中国語)・客家人(客家語)・先住民族(16族諸言語)の社会に加わった.その後,2018年「国家言語発展法」可決により,台湾のあらゆるエスニック・グループが使用する言語が「国家言語」であり,平等に扱われると謳われた.また,同年,国籍法改正により,「新住民」の投票権取得条件が大幅に緩和され、その有権者層の人数も言語の種類も増加した.その一方で,同年,台湾社会が中国語と英語による「2030年バイリンガル国家」を実現するという政策を閣議決定した.そして,2019年より小学校から高校までの「12年国民基本教育制度」という多文化教育が開始された.その言語領域カリキュラムには,「新住民」二世の就学者が大幅に増加したことも関係し,従来の言語教育に加え,特に東南アジア7カ国言語も新たに母語教育や第二外国語科目に導入された.  そこで,本研究ではこのような台湾社会の変化を起点に,上記の法案が「2020台湾総統選」のストラテジーにも何らかの影響を与えたのではないかと仮定し,総統の多言語の使い分け場面として,2020年の台湾総統選候補者の選挙活動と演説に焦点をあて分析する.そして,本研究の目的は,その候補者が場所と対象者により多言語を使い分けるのかを検討し,選挙戦でのストラテジーを明確にすることである.  本研究の方法として,まず,2019年現在の民進党(民主進歩党)蔡英文総統スピーチライターに筆者が実施したインタビューを参考に,2012年、2016年及び2020年の総統選における言語の使い分けをその選挙演説をYouTubeやSNSの動画を中心に文字起こしし,比較し分析した.また,蔡総統の総選挙における手法の変化として,新たに強化したSNSの活用にも着目し,その取り入れ方の特徴の分析も試みた.加えて,1996年の直接選挙制実施以降の歴代総統に関しても,文献や映像から同様に分析した. その結果,国民党(中国国民党)と民進党との選挙の単なる二項対立構図だけではなく,各党内における選挙戦の変容や差異が如実になり,政党の思想や主義とは別に,台湾の多言語多民族社会に時の政府がいかに対応あるいは適応してきたかも解明できた.  結論として,現蔡英文政権の選挙ストラテジーとしては,多様化した台湾社会を束ねるため,まずスピーチライターを起用することで,「言葉(word)」のレトリックのみならず,「言語(language)」の面で,選挙対策チームが多言語の使い分けやSNSを活用した選挙手法の創意工夫に注力したことが勝因に繋がったと考察できる.加えて,候補者の演説は,その場所と対象者ごとに単に多言語を使い分けしているだけでなく,台湾全域の選挙戦を組み合わせて見ることで、台湾アイデンティティを再認識させ団結心を促すストラテジーの効力が明らかとなった.このような状況は,将来日本においても多言語多民族社会やダイバーシティ社会への対応の参考になると示唆される.

報告番号92

フランスにおける中国系移民第二世代の統合とアイデンティティ――ルーツを探求する高学歴の若者に着目して
東洋大学 村上一基

【1.目的】  フランスにおける移民第二世代をめぐる問題として、主にマグレブ系移民やサブサハラ系移民などの子孫の統合と排除が議論されてきた。それに対して、アジア系移民(特に中国系移民)の子孫は特殊な位置付けがなされている。すなわちかれらは「モデル・マイノリティ」として捉えられ、学校不適応や非行の問題などには直面せず、むしろ社会的に「成功」しているとされてきた。その一方で、1990年代以降に流入した中国系新移民の子どもには学校での困難がみられ、さらに近年ではアジア系市民に対する差別や暴力なども顕在化し、とりわけコロナ禍において差別や人種主義的暴力は重大な問題となった。  本報告では、このように「モデル」とされつつも、統合の困難を抱えたり、人種主義などに直面している中国系移民第二世代(本報告では1.5世を含む)が、どのようにフランス社会に統合されているのか、そしてそのなかでどのように中国系としてのルーツを探求しているのかを、とりわけ統合の「成功」とされる高学歴の若者に着目して考察する。 【2. 方法】  本報告では、2017年2月からパリ市と周辺自治体で実施している調査結果を用いる。具体的には、18歳から30歳までの若者17名 、自治体やアソシエーション・宗教施設の関係者12名、学校教員、公立・私立学校で教える中国教師などにインタビューを行った。 【3.結果】  フランスで育った中国系移民第二世代は、フランスの公立学校に通い、いわゆる名門高校や名門大学に通うものも少なくない。そこでは学校教員の勧めによりバカロレア試験の第二外国語として中国語を選択することもしばしばみられる。また大半の若者が幼少期にアソシエーションなどが開く中国語教室に通っていた。その背景には中国語や中国ルーツに対して、中国の経済成長も相まって社会において積極的な意味づけがなされていることがある。  だが、かれらは、青年期に入っていくなかで、自らのルーツを探求するようになり、中国語や中国文化を学び直す。例えば、2009年に中国系のとりわけ高学歴の若者を中心に、「フランスの中国系青少年アソシエーション(AJCF)」が設立された。この団体は、当初中国系の若者同士で連帯し、下の世代を助けることを目的としていた。だが、アジア系市民に対する人種主義的暴力が顕在化するなかで、反人種主義運動を活発に行い、メディアでも着目されるようになった。2020年コロナ禍における人種差別の問題に対しても積極的な活動を行っている。若者は社会において、さまざまな差別やまなざしに直面したり、人種主義の問題に向き合うなかで、自らのルーツを探求し、同じルーツを持つ若者同士で連帯する必要性を感じるようになっている。 【4.結論】  移民の子孫の教育達成については、その国の教育制度やエスニックコミュニティの存在が影響を及ぼすことが論じられてきたが(Portes and Rumbaut 2001;Crul 2010)、中国系移民第二世代の社会統合には、その文化や言語の社会における地位が強く影響し、それがしばしば積極的な意味をもたらしている。その一方で、社会のなかで他者化される経験や、アジア系の人びとに対する人種主義の問題が、かれらにアイデンティティを強く意識させ、ルーツを探求させるきっかけとなる。このように、中国系移民第二世代にとってそのルーツは二重の意味を持つものである。

報告番号93

華僑・華人のトランスナショナル起業とその特徴に関する考察
福岡県立大学人間社会学部 陸麗君

1.目的  本報告の目的は、1990年代以降に来日した新華僑・華人を中心に彼らの起業にどのような特徴があるのか?在留外国人のトランスナショナル起業が彼らの社会移動にもたらした影響はどうであった(である)のかを解明することにある。また、ポストコロナに向けた外国人の起業に関する問題も視野に入れて究明したい。 2.方法  主に関西地域の華僑・華人の経営者を対象に聞き取り調査の方法で行う。具体的には大阪市とその周辺都市、そして兵庫県加古川市などで企業を経営している華僑・華人の経営者に対しインタビュー調査を行い、事例から華僑・華人の起業者たちの特徴と起業が彼らにもたらした影響を把握する。 3.結果  事例調査から以下の3点を中心に華僑・華人のトランスナショナル起業とその特徴を解明したい。 (1)「華僑・華人経営者の来日および起業の経緯」   まず、事例調査から1990年代以降に来日し経営者たちの来日当初の状況を把握する。また起業の経緯については、留学(就学)を終え、日本の企業での就職や飲食店でのアルバイト経験を積んでから、独立したケースが多いことが指摘できる。さらに起業に際しては彼らたちの社会関係資本がどのように動員されるのかを解明する。 (2)「華僑・華人経営活動の状況」   華僑・華人経営者の経営業種、経営活動に際して華僑・華人間のネットワークの役割、パンデミックに伴い、特に打撃を受けているインバウンド関連の事業(民泊・旅行業・ドラッグストア・飲食業など)の経営実態を把握し、コロナウイルス蔓延による就労と経済活動への具体的な影響を把握した上で、ポストコロナにおける在留外国人の経済活動の見通しを明らかにする。 (3)「華僑・華人の起業がもたらした影響」   これは華僑・華人経営者の社会移動に対する影響の側面から把握する。 4.結論  1990年代以降に来日した華僑・華人経営者は、彼らの来日当初の在留資格の多くは留学(就学)であったが、一部は家族滞在と配偶者ビザもあった。一方、近年は経営者ビザでの来日のケースも見られる。起業とその後の経営において、彼らの中国と日本にある社会関係資本が総動員されていること、インターネットの発達によって、グローバルな華人世界のネットワークも活用されていることが指摘できる。またコロナ禍が特にインバウンド関連、飲食関連の華僑・華人企業に大きなダメージを与えたが、これらの企業が日本政府の各種の支援を受けている実態も指摘できる。  最後に華僑・華人の起業は社会移動の側面に与える影響に関して、当事者である起業者の世代内移動にプラスに働いていること、また子世代への教育投資などを通して、起業者とその子世代の世代間移動にも有利に働いている傾向がみられる。

報告番号94

高度経済成長期日本の地域社会とミリタリー・カルチャー――石川県小松市のジェット機基地開設と防衛博覧会を中心に
神戸学院大学 松田ヒロ子

【1.目的】 1950年代後半から60年代―いわゆる高度経済成長期―は、一般に戦後平和主義が浸透する中で、自衛隊に対しても否定的な風潮が強かった時期であると理解されている。そのような歴史理解の一方で、近年の軍事と大衆文化の関係性、そして自衛隊の広報活動における大衆文化の積極的利用が注目され、ミリタリー・カルチャー研究の活性化につながっている。しかしながら、今日理解されているほど、高度経済成長期の自衛隊/防衛庁は、広報活動に大衆文化を利用することに抑制的だったのだろうか。また、学校や地域社会も、戦争や軍事組織に対して、批判的な態度を堅持していたのだろうか。本報告は、1950年代から80年代まで全国各地で開催された「防衛博覧会」に注目することにより、高度経済成長期日本のミリタリー・カルチャーを再考することを目的とする。 【2.方法】 本報告では、事例として1961年に自衛隊と民間航空が共用する小松飛行場/基地の基地開設を記念して1962年に開催された「伸びゆく日本 産業と防衛大博覧会」(以下、「小松防衛博」と略)を取り上げる。主催者の小松市・小松博覧会協会事務局が編纂した『博覧会記念誌』や『北國新聞』などの文書資料をもとに事例研究を行う。 【3.結果】 戦時中に建設され、海軍特攻隊の基地としても使用された小松飛行場は、終戦直後は米軍と民間航空会社によって共有利用されていた。だが、米軍から返還されることを見越して、防衛庁は航空自衛隊のジェット機基地の設置計画を発表した。市議会や商工会議所が推進した自衛隊基地誘致に対して、多くの住民が反対の意思を示し激しい反対運動も展開された。 住民の意見が二分されるなか、基地が完成した後の1962年に催された小松防衛博には、45日間の会期中にのべ51万人以上が来場した。その多くが小松市と金沢市を中心とする北陸地方からの来場者で小中高校生の団体入場者も少なくなかった。建設前は基地建設に反対の声も多く、また防衛博の開催についても反対意見が出されたにもかかわらず、結果的には自衛隊を主役にした小松防衛博は広範囲の住民の支持を獲得して催されたといえる。小松防衛博の成功の鍵となったのは、<地方博覧会>と<防衛博覧会>という二面性である。すなわち<地方博覧会>としての小松防衛博は、当時の世界の先進性の象徴だった航空機を地域の新たなシンボルにして、後進的な「裏日本」という地域アイデンティティから脱却するビジョンを示すことにより、地域の住民や企業の支持を獲得した。同時に<防衛博覧会>として自衛隊の装備品や兵器を暴力装置としてではなく、科学技術製品として電化製品や農業機械製品と並列させることにより、それらを市民の消費生活にとっても、地域や国の発展にとっても「明るい未来」を約束するものとして展示した。小松防衛博は、そのようにして軍事を高度経済成長期の消費主義に接合させることにより、軍事をエンターテイメント化し、地域社会の軍事化に寄与したのである。 【4.結論】 高度経済成長期日本には、「ゆるキャラ」もアニメキャラクターを使った自衛隊ポスターもなかったかもしれない。だが本事例は、近年とは異なるかたちで大衆文化が日本の地域社会の軍事化に重要な役割を果たしていたことを示唆するものといえる。

報告番号95

自衛隊退職者による当事者運動の二重性――1990年代の海外派遣論争と55年体制の崩壊時期を中心に
京都大学 津田壮章

【1.目的】  本報告は、1990年代に生じた自衛隊の海外派遣論争及び、長期政権政党であった自民党の衰退と社会党による自衛隊合憲化という政策的転換期において、その政策の当事者を自認しながらも、既に退職して文民である自衛隊退職者が自衛隊に関連する政策へどのような視線で関与してきたかの解明を目的とする。  兵士の戦後や旧軍の外郭団体を扱う研究は多いものの、戦後の在郷軍人会といえる自衛隊退職者団体隊友会は、戦後長い間、等閑視されてきた。しかしながら、隊友会は自衛隊の政策に関する社会的、政治的運動を続けており、その影響力は無視できない。自衛官はその性質から政治への厳しい制限がかけられており、当事者である自衛官による運動はおこなえない。このため、自衛隊に近い立場で自衛官の代弁が可能な層の存在が、政策に影響を与えるアクターとして重要な意味を持つ。 【2.方法】  隊友会の月刊機関紙『隊友』に掲載された防衛政策への見解や議論の内容を分析した。防衛政策の中でも、海外派遣論争及び55年体制崩壊時期の議論を中心に、記事の中で見え隠れする自衛隊退職者としての要求と「自衛官の代弁者」としての要求の差異に着目した。 【3.結果】  隊友会は元防衛庁長官の自民党所属国会議員を会長に据え、『隊友』で防衛政策に対する見解を共有してきたが、自民党の防衛政策への批判も数多く掲載されていた。そこで語られる防衛政策への批判や要求には、自衛隊退職時の十分な再就職先の確保や叙勲対象の拡大、予備自衛官の待遇改善等、自衛隊退職者という当事者性に基づくものと、自衛官定員の増加や防衛予算の増額、有事法制の制定、海外派遣時の武器使用基準緩和等、現職自衛官が抱くであろう見解を代弁するものに分けられ、『隊友』には後者に該当する自衛隊退職者の意見が目立つ。  1990年前後には、防衛政策への不満を持つ隊友会員有志が中心となり、各地で政治連盟が結成されていく。そこには、自民党の衰退と保守政党の乱立に加え、社会党が自衛隊を合憲と認識したことで、それまでのように自民党一択ではなく、「自衛官の代弁者」として影響を与えるべき政党が増えたことが影響していた。 【4.結論】  隊友会は1990年代の政治状況の中で、自衛隊退職者と「自衛官の代弁者」という二種類の当事者性をもって、自衛隊に有利な政策を推進するための政治的関与を模索しながら、二種類の当事者運動を同時に推進する社会運動団体の性質を有していた。隊友会の正会員は自衛隊を退職した文民であるため厳密には自衛隊の当事者とはいえないが、自衛隊経験を有することで当事者性を自認し、自衛隊退職者としての要求だけでなく、「自衛官の代弁者」として防衛政策に関する防衛庁への要望書の提出や自衛隊出身政治家の輩出、改憲を求める署名運動等をおこなってきた。こうした当事者運動の二重性を有することで、自衛隊退職者の待遇改善を求めるだけでなく、現職自衛官の待遇改善や自衛隊に有利な政策推進を求める運動体として、他の自衛隊外郭団体には無い位置を占めていた。

報告番号96

戦前期東京・都市公園の「浮浪者」対策――「社会」を語り始める造園家とその管理諸実践の分析
東京大学大学院 前田一歩

研究目的・方法 近代日本社会では1910年代後半から20年代以降に、近代化と産業化の諸矛盾が噴出し、さまざまな社会事業と社会調査が実施されだす。既存研究では、近代日本における感化・救済事業と社会事業が、たんなる補助ではなく、被救済者の習慣と考え方を善導するような教化的性格を持つことや、国家による強い統制が存在することが繰り返し論じられてきた(仁平 2011; 副田 2018)。なかでも貧困と居住形態の関係は関心を集め「都市下層」やスラムについての調査と介入の実践が積み重ねられた。本研究が扱う1930年代は、昭和恐慌にともなって増加した「浮浪者」「ルンペン」と呼ばれる、都市オープンスペースに居住する路上生活者がとくに問題化された。都市オープンスペースのなかでも、公園は路上生活者と切っても切り離すことができない。その問題は、東京市全域に中小規模の公園設置が進展しつつある大正・昭和戦前期においても現れた。 本研究は、1930年代の東京の都市公園には、路上生活者を排除しようとする管理方針と、路上生活者を受け入れようとする管理方針が同時に存在したことに注目し、その両義的な浮浪者管理が何によってもたらされたかを分析する。以上を通して、社会一般に存在した、強権的・教化的な浮浪者対策とは異質な公園管理の理念が語られ、それに基づいた実践が行われていたことを提示する。本研究は次の2つの分析からなる。第1に、公園の浮浪者問題に関連して「社会」を語る東京市保健局公園課の造園家たちの言説を分析する。第2に、東京市社会局/保健局等により実施された「浮浪者調査」報告書の読解と、東京市公園課による管理実践の分析をする。 結果・結論 分析の結果、既存研究が示してきた社会一般に存在する浮浪者対策と同様に、浮浪者を強権的に逮捕し、矯正に向かわせる「浮浪者狩り」の舞台となるような公園の浮浪者管理が存在することが明らかになる。しかし、その一方で、公園管理に参加する造園家の言説には、利用者の安全と快適さを確保しつつ浮浪者を受け入れようとする方針が存在し、その方針に基づいた公園管理が実施されたことを提示する。 造園家で東京市公園課長をつとめた井下清は、浮浪者が公園に居住することによる、他の利用者への支障を懸念しつつも、浮浪者にも公園を利用する権利があることを強調する。また「浮浪者問題」は、社会や産業構造の矛盾が都市公園に現れたものであり、公園行政は、その対策をする責務があることを述べる(井下 1932, 1936)。こうした造園家の認識のもと、公園管理に求められる姿勢と、設置されるべき公園の物的な設備が議論され、実施・整備されていくことを示す。公園管理の姿勢とは、当時、井下がスローガンとして掲げた「叱られない公園」をめざすことである。これは、公園利用の支障となりそうな要素を事前に除去し、管理者による直接的・抑圧的な利用者の管理を避けようとするものである。さらに、この方針と関連するものとして、公園で実施された浮浪者調査と、浮浪者の利用を特別に想定することで、飲用水洗と休憩所を設置した小公園「常盤公園」の設置を事例に分析する。このように、公園利用者一般の利便性と安全性を最大化させると同時に、公園という場所を介して社会問題を解決しようする造園家の姿勢が、「浮浪者」を排除しつつ受け入れる、公園の両義的な浮浪者対策を成立させたのである。

報告番号97

原爆被爆者たちの〈生〉と死者との関係に関する検討
武蔵大学 徳久美生子

1. 本報告の目的 本報告では、原爆や周囲の大人たちから〈生〉を否定され、自分が生き残った意味を見出せない体験をした原爆被爆者たちを〈生き抜く〉方向へと向かわせた動因を、死者たちとの関係という観点から明らかにする。 2. 方法 2011年から行ってきた原爆被爆者たちの生活史を分析する。特に継続的に話を聞いている建物疎開の作業に駆り出されていた多くの友人たちを亡くした、被爆時に現在の中学1年だった7名の女性たちの生活史をもとに検討する。 3. 分析結果 生活史調査を分析した結果、彼女たちは、敗戦後の苦難、原爆体験に関する他者からの無理解、日常生活を困難にする原爆の心身への影響、人間関係の困難といった〈生〉を否定する要素を、〈生〉へと向かう方向へと反転させて生き抜いてきていたことがわかった。いわば反転の積み重ねが、〈生〉を支えてきており、現在がある。この〈生〉へ向かう反転には、死者たちとの関係が3つの動因として関与していた。 (1)死者の苦難との比較  彼女たちには、自らの苦難を常に低く見積もる傾向がある。彼女たちにとって一番可哀想なのは、死者たちであり、死者たちの悲惨と比較すると、自分たちが直面する困難は低く見積もられる。だが死者たちの苦難は、彼女たちが自らの苦難を超えて〈生〉へと向かう動因となった。 (2)死へと向かう〈生〉  彼女たちにはそれぞれ特別な死者である友だちがいる。このかつて時間と空間を共有していた死者の不在は、自分もまた「死ぬであろうこと、そして自分が死んだ後も世界は永続していくこと」(Schutz and Luckmann 1973=2015: 122)を彼女たちに認識させた。実際に、原爆の影響は続き、彼女たちにとって死は常に身近だ。けれども彼女たちは、自身の死を自覚し、死が身近であったがゆえに〈生〉を志向した。 (3)生きる責任・生き抜く意思 彼女たちは、なぜ自分が原爆から生き残ったのかがわからないだけでなく、なぜ特別な死者である友人たちが死ななければならなかったのかもわからない。だが確かなことはある。原爆から「生きる・逃げる」だけを考えた自分と同じように、死者となった友人たちも、生きたかったのである。そのため、彼女たちには、生きたかった友人たちが生きられなかった時間を生きる責任が生じる。他方で、未来にくる自らの死は、死者たちとの再会のときでもある。再会のときに死者に恥じないようにと、彼女たちは、生き抜くのである。 以上3つの動因は、彼女たちが他の苦難を反転させ生き抜く基盤となったと考えられる。 4. 結論  生活史調査に協力してもらった7名の原爆被爆者たちは、死者との関係を基盤に、苦難を生きる方向へと反転させ、原爆によって否定された〈生〉を生き抜いた。彼女たちの〈生き様〉は、生き抜くことが、原爆という死の兵器に対する〈生〉からの抵抗となることを示唆している。 参考文献 Schutz, A. and Luckmann,T,1973=2015那須壽監訳『生活世界の構造』ちくま学芸文庫.

報告番号98

戦後「混血児問題」における<反人種差別>の論理――「混血児」概念の文脈と用法に着目して
東京大学大学院 有賀ゆうアニース

【1.目的】 戦後日本では米軍の占領を背景として進駐軍兵士と日本人女性の間に大量の児童が出生し、彼ら「混血児」をどのように処遇すべきかという観点から「混血児問題」が社会問題化した。先行研究ではレイシズム・ナショナリズム批判の観点から「混血児問題」が考察され、「単一民族神話」(Roebuck 2015)や「日本人化」(下地 2018)の力学によって「混血児」への人種差別が正当化・不可視化されたことが論じられてきた。しかし、先行研究では、①「混血児問題」をめぐる当時の政府の公的指針において人種差別・偏見そのものは問題として想定されていたという点(Fish 2009)、②こうした文脈において問題にかかわる当時の人々たち自身がどのような歴史的・局所的な知識・基準を参照することで人種差別を問題として同定していたのかという点が見落とされている。この課題にとりくむために、本研究では戦後日本社会で「混血児」をめぐる「反人種差別」の論理がいかにして構築され展開したのかを明らかにすることをめざす。 【2.方法】 以上の目的を遂行するために、欧米諸国における反人種差別政策の形成過程に関する歴史社会学的研究を参考に(Bleich 2003, Skrentny 2006)、①「混血児」カテゴリーがどのような制度的文脈でどのように使用されているのか、そして②そうした文脈と用法におうじてどのような社会的活動・認識が編成されるのか、という2つの観点から分析を進めた。 【3.結果】 「混血児」概念が参照されるにあたっては「子どもとしての混血児」という知識が前提されることで、学校教育や児童福祉などが「混血児問題」の制度的文脈として発動されていた。また「混血児」は、こうした文脈にかかわるアクターの活動を通じて、日本人による差別・偏見の被害者としても理解されるようになった。それにより、日本人の責任を追究し、人種差別を是正するための施策を肯定することが可能になった。他方、こうした反差別規範は、「子ども」という人生段階カテゴリーに依存していたがゆえに、児童福祉や学校教育という制度的枠組に制約されるという限界を内包していた。 【4.結論】 戦前までの日本で人種差別が公的言説の中で語られる場合、日本(人)はその被害者と想定されることが一般的であり、差別の加害者として日本人を位置づけ自己批判する言説が顕在化するのは反戦運動や反民族差別運動が発達した1970年以降のことであることが先行研究では示唆されている(小熊 1998; 2009)。これに対して本研究によれば、人種差別を公共的な問題として記述し、その責任を日本人たる自己に帰属する論理は、1950年代の時点で―きわめて微弱であったとはいえ―かつ一定の公的施策に結びついていた。このように本研究は、戦後日本における反(人種)差別規範についての既存の歴史的説明を補完する点に意義を有する。

報告番号99

「空気」を社会学で問うために――太平洋戦争・沖縄水上特攻の事例分析から
東京大学大学院 猪又梓

【1. 目的】 本報告は、日本社会が「空気」と一般に呼ぶ状況とはいかなる社会現象であるか、またいかにしてこの現象が成立するのかを、ひとつの歴史的事件の事例解釈を手掛かりに探索する社会学的考察である。 【2. 方法】 検討する事例は日本海軍の天一号作戦である。天一号作戦とは、1945年3月以降、連合国軍の沖縄進攻に対処するため組織された艦隊特攻作戦である。その「経緯」として語られるのは、これが作戦として不成立なのは論理的に明らかであるにもかかわらず、連合艦隊司令部では着々と計画が進行し、その一方で合理的な理由から作戦に反対してきたはずの実働部隊において、その責任者たる長官が、手のひらを返すように計画をあっさりと受け入れた、という空虚な内容である。この事件は「空気」を社会問題として問うた山本七平の著書『「空気」の研究』において繰り返し言及され、その後の「空気」論の系譜においても重要な研究対象とみなされてきた事例である。山本は、立案・実行役の連合艦隊司令部の証言に、「空気」が自分たちを実行へと駆り立てた、という不可解な論理が存在することに着目していた。この不可解な論理は、彼自身が太平洋戦争で経験した不条理な軍隊の行動論理にみられるだけではなく、〈過程が不透明で非合理的にも思える意思決定〉のあり方が、戦後の平和な社会の日常生活においても随所に観察しうる、という経験的事実から得た確信であった。したがって本報告では、「空気」という証言が指す具体的状況とは何かを、まず天一号作戦の事例から読み解く作業を行う。当時を語る資料を収集し、時系列順に再構築する作業を行うことで、当事者の意味世界に分け入り、彼らの目線に近づいて意思決定過程の内実を分析する。分析視点としては、つぎの3つの視点から議論を行う。第一に、特攻作戦の命令を受諾した第二艦隊司令部、第二に、意思決定過程の周縁にいた部下および周縁組織としての連合艦隊司令部、呉鎮守府、第三に、当時作戦立案・命令の最高責任者であった連合艦隊司令長官の各証言を取り上げる。 【3.結果】 第一の視点における結果としては、第二艦隊は最終的には受諾し、特攻命令は実行されたが、少なくとも沖縄戦の前年に起きた捷一号作戦後から、連合艦隊司令部の戦略を批判しており、連合艦隊司令部と意見対立が起こっていたことである。ただしこれまでの第二艦隊が行ってきた反対の経緯に照らすと、この命令の受諾はたんなる状況への順応ではないということを発見した。第二の視点における結果は、彼ら周辺組織、部下たちが中心的意思決定過程に接続し、その過程を促進し「一億総特攻」思想の拡大が生じていたことである。第三の視点における結果についての1つめは、すり替えの論理である。2つ目は、独特の「確率論」的表現である。3つめは「フリート・イン・ビーイング(艦隊温存)」への戒めという用兵思想である。 【4. 結論】 山本は「空気」をすべてを拘束する強大な力とする。しかし今回の分析では、「空気」の決定の拘束力が限定的であり、「空気」の意思決定は、重層的で複合的な過程を経てできあがるものだということがわかった。「空気」の力は、そこで形成された意思決定過程の中で局所的に正当化されているため、その中にあるものを強制的に従わせるような力が働いているようにも思えるが、これは閉じた論理においての限定的な拘束力である。

Back >>